Professional Responsibility Midterm Question Bank

Réussis tes devoirs et examens dès maintenant avec Quizwiz!

An attorney represented a municipality for several years, in accordance with a contract for legal services. The contract term ended, and the municipality published a new request for proposals, and in the end chose a different lawyer to provide legal services for the next several years. The municipality requested that the attorney (the one whose contract expired) provide the municipality's new counsel with all files - open and closed. The municipality has already paid the attorney in full for all his legal work. Which of the following would the attorney NOT have to provide to the municipality?

A general assessment of the municipality or the municipality's matter

An attorney was an associate at Big Firm. In his first year there, as a recent law school graduate, the attorney had a twenty-minute conversation with a more senior associate about research strategies involving a narrow issue of venue in federal court. The research was part of the representation of Big Bank, in the case of Developer v. Big Bank. The attorney's time sheets from the time clearly document the length of the conversation and its subject matter. The entire conversation focused on the facts pleaded in the complaint and answer; the attorney learned no confidential information about the matter. Eventually, the attorney left Big Firm to become an associate at Boutique Firm. Eighteen month later, a partner assigned the attorney to represent the same Developer against Big Bank in a matter that overlapped on many points with the matter in which Big Firm had represented Big Bank. Lawyers at Big Firm still represent Big Bank, and they inform the bank's officials that the attorney who worked for them is now working at Boutique Firm, representing the Developer. Big Bank instructs the lawyers at Big Firm to seek the disqualification of the entire Boutique Firm from representing the Developer in the matter. How could Boutique firm avoid the imputation of a conflict of interest to its lawyers?

At most, Boutique Firm would need to screen the attorney from the matter and have other lawyers represent the Developer, but even this may be unnecessary, because the attorney learned no confidential information about Big Bank at his previous firm.

A prospective client met with an attorney at Boutique Firm for an initial consultation about a personal injury lawsuit over injuries the prospective client had sustained. The attorney declined the representation because he thought the client's case was unwinnable and would therefore generate no fees. During the consultation, the attorney asked some probing questions about the incident, and the client admitted facts indicating an unreasonable assumption of foreseeable risks beforehand, as well as the client's own intoxication at the time, which in the case would constitute contributory negligence. Furthermore, the client had failed to take obvious measures afterward to mitigate the damages. The attorney was certain that all these unfavorable facts would come out during discovery, and the client's claim would become laughable at trial. Two months later, another client came in for a consultation with another lawyer at Boutique Firm. This prospective client had was service of process in a new personal injury lawsuit, and he was the named defendant. The plaintiff in the lawsuit was the same individual who had met with the first attorney for a consultation a few weeks before. Boutique firm agreed to take the case and represent the defendant in the litigation, and it has no measures in place to screen the attorney who consulted with the prospective client from participating in the matter. Which of the following is true, according to the MRPC?

Boutique Firm cannot represent the defendant in the case because an attorney there learned confidential information from the opposing party as a prospective client during an initial consultation two months ago, unless Boutique Firm obtains informed consent in writing from both the defendant and the opposing party, who was a prospective client during a one-time consultation.

A certain client hired an attorney to represent her at trial. After voir dire, the attorney wanted to learn as much as possible about each of the jurors, such as their views on political and social issues that might be relevant to issues in the case, so the attorney found each juror's social media accounts and reviewed their postings and comments. Was it permissible for the attorney to review all the social media posts and comments by the jurors, even back to their high school days, if the attorney has no other communication with the juror?

It is permissible for a lawyer to review a juror's Internet presence, which may include postings by the juror or potential juror in advance of and during a trial.

An attorney had her own firm, and she employed a paralegal who had previously worked for another firm. The attorney agreed to represent two new clients: a plaintiff in a personal injury lawsuit, and a seller in commercial real estate transaction. The personal injury plaintiff had a case that was unlikely to succeed due to evidentiary problems, though it was legally valid and factually plausible. The attorney wanted to charge a fixed, non-contingent fee up front for this case, and the client reluctantly agreed. The seller of the commercial real estate, on the other hand, was in a hurry to complete the deal and wanted to liquidate the asset for more than its fair market value, which was possible but also unlikely to succeed. The attorney offered to handle the transaction on a contingency fee basis. If she could negotiate with prospective buyers and convince one to buy the property immediately for a price above the appraised value, she would receive thirty percent of the sale price as a fee, but if it sold at or below the market value, or took more than two months to sell, the attorney would receive only reimbursement for the transaction's costs and expenses. The paralegal told the attorney that these fee agreements were impermissible because personal injury plaintiff's normally paid contingent fees, and real estate transactions had to be on a fixed or hourly fee basis. The attorney disagreed, but she did not check the ethical rules herself to confirm this. Which of the following is correct?

It is proper to charge a plaintiff in a personal injury case a flat fee regardless of the case outcome, and it is permissible to charge the seller a contingent fee in a real estate transaction.

An attorney was an associate in Big Firm for eighteen months from early 2003 to late 2004. Another lawyer at Big Firm had been representing MindGames Inc., a creditor in the bankruptcy proceeding of Education Support International since 1999. The associate left Big Firm in 2004 to work for Regional Cancer Center as general counsel, where the medical director was Dr. House. There is a long, sad story here, but the bottom line is that Education Support International, which was still in bankruptcy, also owed money to Dr. House as a major shareholder of the failed company. In the summer of 2005, the bankruptcy court entered judgment in favor of MindGames Inc. and the shareholders, and MindGames immediately filed for sanctions against (compensation from) the shareholders, including Dr. House. At that point, Dr. House's lawyer withdrew from representation because the case had taken a complicated turn, and Dr. House asked the general counsel at his medical center - the associate we met at the beginning of this story - to represent him going forward. MindGames filed a motion to disqualify the attorney from representing Dr. House in the proceeding and the appeal, because he had formerly worked at Big Firm, in an office a few doors down from their own lawyer there. The bankruptcy judge agreed, applying an irrebuttable presumption that the attorney learned confidential information about MindGames while working at Big Firm, but the attorney insists he never worked on any MindGames matters and has did not learn any confidential information. The attorney has appealed the disqualification to the Fifth Circuit. How should the circuit court rule?

It should reverse the disqualification order because the imputed conflict of interest disappeared when the attorney left Big Firm to work for Regional Cancer Center, given that the attorney knew no confidential information about MindGames.

An attorney had her own firm, and she employed a paralegal who had previously worked for another firm. The attorney agreed to represent two new clients: a plaintiff in a personal injury lawsuit, and a seller in commercial real estate transaction. The personal injury plaintiff had a case that was unlikely to succeed due to evidentiary problems, though it was legally valid and factually plausible. The attorney wanted to charge a fixed, non-contingent fee up front for this case, and the client reluctantly agreed over the phone, though the fee agreement was not in writing.The seller of the commercial real estate, on the other hand, was in a hurry to complete the deal and wanted to liquidate the asset for more than its fair market value, which was possible but not likely to happen. The attorney offered to handle the transaction on a contingency fee basis - if she could negotiate with prospective buyers and convince one to buy the property immediately for a price above the appraised value, she would receive thirty percent of the sale price as a fee, but if it sold at or below the market value, or took more than two months to sell, the attorney would receive only reimbursement for the transaction's costs and expenses. The client agreed to this arrangement over the phone, after the attorney had carefully explained it, though the fee agreement was not in writing. The attorney was successful in both matters, and both clients were satisfied with the results of the attorney's representation. The paralegal told the attorney that these fee agreements were impermissible because personal injury plaintiff's normally paid contingent fees, and real estate transactions had to be on a fixed or hourly fee basis. The attorney disagreed, but she did not check the ethical rules herself to confirm this. Which of the following is correct?

It was impermissible for the attorney to represent the seller in a commercial real estate transaction on a contingent fee basis without a written fee agreement, signed by the client, stating the method of determining the fee.

Attorney Ames and Attorney Adams work in the corporate legal office of Risk Company. A federal regulatory agency is investigating of the activities of Risk Company and is deciding whether to initiate criminal charges against Risk Company, some of its employees, or both. The regulatory agency has a long-established practice of not charging corporations for violations that corporate employees commit, where the corporation can show convincingly that it actively sought to discourage the offense in question. Showing this practice would, however, almost guarantee that an employee would face charges individually for the violation. Stevenson is a Risk Company employee upon whose activities the agency has begun to focus. Before Attorney Adams' employment by Risk Company, she had been in private practice and had advised Stevenson with respect to the very conduct that is the subject of the agency investigation. Can Attorney Ames, who works with Attorney Adams, represent Risk Company in the matter before the regulatory agency, without obtaining informed consent from Stevenson?

Neither Attorney Ames nor any other member of Company's corporate legal office may represent Company without obtaining Stevenson' informed consent.

On the same day, an attorney agrees to represent an undocumented immigrant in a visa-revocation matter, as well the victim in a tort action (product defect) against an automobile manufacturer. The two matters are unrelated. In the second case, the attorney anticipates that the defendant automobile manufacturer will argue that extensive federal safety regulations of automobiles, which require certain safety features and specifications, should preempt state tort law and therefore prevent a state court from adjudicating the case. On the other hand, many undocumented immigrants have relied upon federal preemption of state law in challenging onerous state penal statutes targeted at illegal immigrants. If the attorney prevails in his preemption argument in the vehicle manufacturing defect case, and on appeal creates precedent against federal preemption of state law, the precedent would probably be unfavorable to the attorney's other client, the undocumented immigrant. The state legislature has several bills pending that would impose criminal sanctions on landlords who lease apartments to undocumented aliens, drivers who transport undocumented aliens to work sites, and contractors who hire subcontractors who employ undocumented aliens. Does the fact that advocating a legal position on behalf of one client might create precedent adverse to the interests of the other client create a conflict of interest, for which the lawyer must obtain consent?

No the mere fact that advocating a legal position on behalf of one client might create precedent adverse to the interests of a client represented by the lawyer in an unrelated matter does not create a conflict of interest

A defendant faced charges in a white-collar crime case - corporate espionage, securities fraud, and so forth. The police brought the prosecutor voluminous files, with the file boxes filling an entire storage room at the district attorney's office. These dozens of file boxes were only part of the evidence the police had amassed before making the arrest, so they inform the prosecutor that they have another room full of corporate records documenting the crimes in the basement of their precinct. Unbeknownst the to the prosecutor, some of the files in both locations have a few documents that case doubt on the defendant's role in some of the crimes, and others that might tend to mitigate the some of the other charges. If the prosecutor has not yet reviewed voluminous files or obtained all police files, however, does Rule 3.8 require the prosecutor to review or request such files before the plea bargaining, so that the defense can make better-informed decisions during the plea negotiations?

No, Rule 3.8 does not require the prosecutor to review or request such files unless the prosecutor knows or infers from the circumstances, or it is obvious, that the files contain favorable evidence or information.

An attorney represented a defendant who was facing criminal charges for violating a newly enacted statute. The statute that made certain activities that had previously been minor misdemeanors into felonies. The district attorney handling the case spoke to the defendant's attorney, explaining that this was an important test case of a new statute, so the D.A.'s office was seeking the maximum penalty. The state did, however, offer a reduced sentence if the defendant would plead guilty, but this would still carry three years of jail time. Outraged, the attorney shouted that this had always been a misdemeanor charge in the past, which carried no jail time at all, and ended the conversation abruptly at that point. Without mentioning the conversation to the defendant, the attorney drafted an impassioned motion to dismiss the charges and filed it with the court. The attorney had a reasonable belief that his motion could be successful, though it was far from certain. The judge agreed with the attorney and granted the motion, dismissing all the charges against the attorney's client. Were the attorney's actions permissible under the Model Rules?

No, a defense lawyer who receives a proffered plea bargain in a criminal case must promptly inform the client of its substance, unless the client has previously told the lawyer to accept or to reject the offer.

A federal judge hired a clerk for the first two years after the clerk graduated from law school. During his second year as a clerk, he began applying for associate positions at local law firms, to secure a job that would begin immediately after his clerkship ended. A few of the firms to which he applied had pending matters before the same judge, and these were among the firms that interviewed the clerk for an associate attorney position. During the interviewing process, the clerk refrained from mentioning he knew about their pending matters on his judge's docket, though the interviewers always mentioned the fact that their firms regularly appeared before the judge in whose chambers the applicant was then clerking. Each firm that interviewed the clerk received a letter from the judge recommending the applicant to prospective legal employers. Even though some of these firms had pending matters on the judge's docket, the judge knew from the clerk which firms were interviewing the clerk. Was it improper for the clerk to apply for positions at firms that have pending matters before the judge for whom she was clerking?

No, a law clerk to a judge may negotiate for employment with a party or lawyer, even if the prospective employer is involved in a matter in which the clerk is participating personally, after the lawyer has notified the judge.

An experienced attorney had his own solo law practice. The attorney agreed to provide representation to a certain client, which would entail researching and writing several legal opinions for the client pertaining to the client's anticipated litigation, and the attorney's usual hourly rate. The proposed research and writing would require a substantial amount of time, so their agreement stipulated that the attorney would bill the client every two months. The client paid the first bill and then stopped paying. After several months, the anticipated litigation began, and the client requested copies of all the remaining legal opinions that the attorney had agreed to write. The attorney had followed state laws to secure a lien on his work product for the client after the client stopped paying. Could the attorney be subject to discipline if he were to retain the documents that the client has not yet paid for?

No, a lawyer may acquire and act upon a lien authorized by law to secure the lawyer's fee or expenses.

Boutique Firm charges its clients five dollars per page for photocopies done in-house on the firm's copiers. All new clients receive a schedule of fees before the representation begins that clearly specifies such charges, and client bills clearly itemize photocopying charges. The charge applies even if the client never sees the photocopies, as when associates conducting research must copy sections of cases, statutes, and regulations, or circulate draft memoranda to other lawyers working on the case. The charge also applies when the firm must produce documents for the other party in response to a discovery request. Boutique Firm set the amount at five-dollar per page, even though photocopies cost the firm only fifteen cents or so per page, because the surcharge generates a side revenue stream for the firm that enables it to charge lower legal fees, and to discourage clients from wasting paper. Boutique Firm is environmentally conscious. Is it permissible for Boutique Firm to assess clients a surcharge per page for photocopies done in-house?

No, a lawyer may charge the client no more than the actual cost of making a copy on the photocopy machine plus a reasonable allocation of overhead expenses directly associated with the provision of the service, such as the salary of a full-time photocopy machine operator.

A municipal election for a seat on the city council was remarkably close one year, resulting in a run-off election that was ever closer. Both candidates claimed victory, and each accused the opposing candidate of voter fraud and violations of various election rules. There is potential for litigation if the two cannot agree as to a winner in the election, with one or the other conceding. A certain attorney is a prominent lawyer in the community and has previously represented each candidate in various legal matters. Both candidates would like to hire the attorney to represent them in negotiating a resolution to the election. Each candidate fully understands their adverse interests and the potential conflicts of interest for the attorney, but each is willing to provide written informed consent to have the attorney represent them both in facilitating the negotiations. May the attorney represent both candidates in this negotiation?

No, a lawyer may not represent multiple parties to a negotiation whose interests are fundamentally antagonistic to each other, even in a negotiation.

A certain client hired an attorney to represent her at trial. After voir dire, the attorney wanted to learn as much as possible about each of the jurors, such as their views on political and social issues that might be relevant to issues in the case, so the attorney found each juror's social media accounts and reviewed their postings and comments. One juror, the foreperson of the jury, had limited the access of some of her social media accounts so that only her friends or connections on that platform could view what she shared. The attorney asked his law student intern to send a connection request to the juror in hopes of gaining access to the juror's shared photos, commentary, and so on. There would be no reason for the juror to know the intern worked for the attorney in the case, as the intern was never present in the courtroom, and her own social media accounts did not mention her internship. The intern did not otherwise engage in conversations online with the juror, did not discuss the case with the juror, and did not respond to or "like" anything the juror shared or posted through social media. Was it permissible for the attorney to have his intern request access to the juror's social media posts, if the attorney has no other communication with the juror?

No, a lawyer may not send an access request to a juror to review of the juror's electronic social media, even vicariously through an intern.

A certain client hired an attorney to represent her at trial. After voir dire, the attorney wanted to learn as much as possible about each of the jurors, such as their views on political and social issues that might be relevant to issues in the case, so the attorney found each juror's social media accounts and reviewed their postings and comments. One juror, the foreperson of the jury, had limited the access of some of her social media accounts so that only her friends or connections on that platform could view what she shared. The attorney sent a connection request to the juror in hopes of gaining access to the juror's shared photos, commentary, and so on. The attorney did not otherwise engage in conversations online with the juror, did not discuss the case with the juror, and did not respond to or "like" anything the juror shared or posted through social media. Was it permissible for the attorney to request access to the juror's social media posts, if the attorney has no other communication with the juror?

No, a lawyer may not send an access request to a juror to review of the juror's electronic social media.

An attorney represented a large pharmaceutical company that was part of an industry consortium. The industry consortium was pressuring the Food and Drug Administration (FDA) to relax its requirements for approval of new drugs. Regulators within the FDA had divergent views on this from a policy standpoint. The issue did not pertain to any individual drug or company; it concerned procedures for new drug approval as a general matter. Representatives from different companies within the industry would meet as a group with a team of regulators tasked with reviewing the FDA's policies in this regard, and during these meetings the regulators would ask the industry representatives probing questions about their research and development costs, market share, and retail pricing of drugs after approval. When executives from the attorney's company were preparing to attend one of these "negotiated rulemaking" meetings, the attorney coached him to say, "I do not recall" whenever the regulators asked questions that would reveal information unfavorable to the company's position. Was it permissible for the attorney to counsel the corporate executives to give evasive or vaguely untruthful answers at an industry meeting like this?

No, a lawyer may not submit false statements or false evidence through another to a rule making agency, which should be able to rely on the integrity of the submissions made to it.

An attorney represented a client in transactional matters, and another lawyer in the same firm represented the client in pending litigation. The attorney did not appear on a list of counsel for the litigation matter and was not planning to sit at counsel table or otherwise physically appear in support of advocacy. On the other hand, the litigation involves a transaction that the attorney handled previously for the client. Would it be impermissible for the attorney serve as a witness in support of the client's position in the trial?

No, a lawyer serving in a capacity other than that of a courtroom advocate may serve as a witness for the lawyer's client.

An attorney began her career as a prosecutor at the District Attorney's Office. During her tenure as a prosecutor, she brought charges against an individual suspected of sending ricin, a deadly toxin, in an envelope to a prominent politician, apparently in an unsuccessful attempt to assassinate the public official. The jury found the evidence too attenuated, and it acquitted the defendant. Shortly thereafter, another person, who was a member of a terrorist organization, confessed to sending the ricin and provided extensive evidence of his plot to kill the politician to make a political statement. The attorney resigned from the District Attorney's Office, partly out of humiliation over this case, and went into private practice. Eighteen months later, the accused individual decides to sue the government over wrongful arrest, slander, libel, and wrongful prosecution over the case in which he obtained an acquittal. The attorney feels that her superiors at the D.A's Office had pressured her to press charges in order to satisfy the public uproar over the ricin letters, despite having scanty evidence that the accused individual was actually guilty. The attorney offers to represent the accused individual in his lawsuit against the government, partly to make amends or atone for her role in what she now views as an abuse of government power and a great injustice. Would it be proper for the attorney to handle this case, given her good intentions?

No, a lawyer who has prosecuted an accused person could not properly represent the accused in a subsequent civil action against the government concerning the same transaction

An attorney spent several years working for the state Office of the Attorney General in its environmental litigation division. While there, the attorney began a case against a scrap metal facility for burying toxic materials on its grounds. The attorney then left government service and went to work for Big Firm. There, the attorney began representing a group of neighboring landowners in a lawsuit against the same scrap metal facility over the same burying of toxic material, as it had polluted the groundwater and had migrated to adjacent properties underground. Is it proper for the attorney to represent these plaintiffs?

No, a lawyer who has pursued a claim on behalf of the government may not pursue the same claim on behalf of a later private client after the lawyer has left government service, except when authorized to do so by the government agency

An attorney served for a while as a municipal court judge, and during that time, she sentenced certain defendants facing criminal charges to terms of probation. Eventually the judge left the court and returned to private practice. Once settled in her new practice, three prospective clients sought to hire her file motions to end their terms of probation early, due to their good behavior and their need to relocate for their jobs. Would it be proper for the attorney to represent them in filing these motions?

No, a lawyer who served as a judge may not represent a client in a matter in which the lawyer had personal and substantial involvement.

Conglomerate Corporation spilled a large quantity of toxic sludge along the edge of its property, and spillage polluting two adjacent properties, one parcel owned by a private individual, and the adjoining parcel that was state-owned. The subdivision of the state that owned the polluted parcel agreed with the private landowner to be co-plaintiffs in a tort action against Conglomerate as the polluter, and to use the same attorney to represent both the state and the private landowner. The private landowner was mostly concerned about the loss to his property values, as this was an investment property. The state was concerned entirely with cleanup costs and the threat to public health. An authorized official at the state agency provided the attorney with written consent to the potential conflicts of interest inherent in the joint representation, as did the private landowner. Under such circumstances, would it be improper for the same attorney to represent both the government and a private party at the same time, in the same matter?

No, after obtaining the necessary written consent, the attorney may represent both the private party and a government agency.

An attorney, who often serves as a court-appointed mediator, was appointed to mediate the divorce case between a husband and wife. The case settled in mediation and the divorce was finalized soon after. A year later, the husband sought to retain the attorney to represent him in a modification suit against his wife. The attorney accepted the case and sent a letter to the wife advising her that the attorney had been retained by the former husband to represent him in a modification suit. Are the attorney's actions proper?

No, an attorney who previously served as a third-party neutral is required to obtain informed consent, confirmed in writing, from all parties to the proceeding prior to representing a party in a suit connected to the previous matter

A client retained an attorney to represent him in two cases: a criminal case and a divorce case. The attorney required that the client pay a retainer fee for the family law case, which billed at the attorney's hourly rate. The attorney then arranged for the client to pay him based on a contingency fee for the criminal case. The attorney and the client both signed the combined contract, which detailed each fee arrangement for each case, and the attorney's representation began. Are the attorney's actions proper?

No, as attorneys cannot charge a contingent fee for representing a defendant in a criminal case

Years ago, as a law student, Attorney worked on a case for Client during a law firm internship. Now, Attorney's firm is representing a Defendant in a lawsuit in which Client is the plaintiff. Client's new lawyer moves to disqualify Attorney's firm from the representation when it learns that Attorney worked for another firm on behalf of Client when Attorney was still a law student. Is Attorney's firm subject to disqualification in this case?

No, as long as the firm screens the attorney from any personal participation in the matter to avoid communication to others in the firm of confidential information that both the non-lawyers and the firm have a legal duty to protect

A certain attorney represents a client in a civil suit. The client and the attorney often discuss their hunting trips and have gone hunting together on several occasions. The client tells the attorney he is purchasing a piece of property for hunting with five other people and asks the attorney if he would like to go in on the purchase. The attorney tells the client he would like to join in the purchase and he provides the client with a check for his portion of the purchase price. Is the attorney subject to discipline?

No, attorneys can enter into fair and reasonable business transactions with clients assuming the client receives an advisory in writing of the benefit of seeking advice from independent counsel and gives informed consent, in writing and signed by the client, of the transaction details.

A certain attorney, a partner at a law firm, prepares a will for Sister. In the will, Sister directs the attorney to receive a substantial part of her estate. Then the attorney also recommends Sister appoint the attorney as the executor of the will because of his knowledge in this field. The attorney explains to Sister the role of the executor and the pay the executor of the estate will receive and discussed alternative executor choices with her. In addition, the attorney recommends Sister seek independent legal counsel regarding the issue of the executor. Sister does so, and then she asks the attorney to list him as executor in the will. Is the attorney subject to discipline?

No, attorneys may permissibly include gifts to themselves in a will prepared by an attorney for a person related to the attorney, even if the gift is substantial.

Big Firm hired associates from the top of their class at the most prestigious law schools. Big Firm's partners often boasted to their clients, truthfully, that all their associates did federal judicial clerkships before joining Big Firm as lawyers. Conglomerate Corporation retained Big Firm regularly as outside legal counsel, partly in reliance on these representations from Big Firm's partners about the credentials and experience of their associates. On one occasion, an associate at Big Firm did several hours of legal research on a certain topic for one client, Conglomerate Corporation. The research later turned out to be relevant to another client's legal matter. Would it be permissible for Big Firm to bill the second client, who agreed to pay fees based on the time spent on the case, the same amount for the recycled work product that it charged Conglomerate, the first client, if Conglomerate consented?

No, attorneys who reuse old work product have not re-earned the hours previously billed and compensated when they first generated that work product.

A transactional attorney agreed to represent a new client who already had representation by trial counsel on another matter. The client agreed to a complex fee arrangement, which included a fixed flat fee for the first phase of the transaction, a modest hourly rate for the remainder of the transaction, and a modest contingent fee in addition to these other fees, scaled to the outcome of the transaction - that is, a higher contingent fee for obtaining more favorable final terms in the transaction. The attorney did not advise the client to seek the opinion of independent legal counsel for this transaction, and did not obtain signed, written consent from the client about the attorney's role in the transaction. The client's other lawyer reviewed the terms of the fee agreement and advised the client to accept it. Based on these facts, could the attorney be subject to discipline for violating the provisions Model Rule 1.8 that govern business transactions with clients?

No, because Rule 1.8 does not apply to ordinary fee arrangements between client and lawyer.

Alpha Firm and Beta Firm represent the two parties in a high-stakes commercial transaction - the sale of a subsidiary corporation from one large, international conglomerate to the other. An attorney at Alpha Firm is married to a lawyer at Beta firm, but the spouse at Beta Firm is not involved in the representation. If a problem arose, would a tribunal that follows the ABA Model Rules impute the marriage-based conflict of interest that Alpha Firm's attorney to all the other lawyers in the firm, if another lawyer at Alpha Firm handled the representation in this case?

No, because a conflict arising from a lawyer's marriage to another lawyer at an opposing law firm does not necessarily impute to all other lawyers in the firm.

An attorney defended a client in a criminal proceeding that attracted low-level media attention on the local evening news and a few local-interest blogs. One of these bloggers called the attorney at his office and asked for a quote about the client's case. The attorney stated that a member of the local clergy, as well as the Principal of the local high school, would testify as to the client's good character and volunteer activities. Was it proper for the attorney to discuss such things with a blogger?

No, because a criminal defense lawyer may not make any extrajudicial statements except to state the claim, offense or defense involved and, except when prohibited by law, the identity of the persons involved

An attorney is a criminal defense lawyer. The court has appointed him to represent a defendant who has already given a full confession of the burglary to the police, after receiving proper Miranda warnings, and the prosecution has several witnesses who either saw the crime or heard the defendant discussing his plans to commit the crime beforehand. The police properly obtained all necessary warrants during their investigation and arrest, and the defendant's actions clearly meet the elements in the statute. The lawyer explains to the defendant that he has almost zero chance of an acquittal, given the evidence against him and the fact that the Supreme Court has repeatedly upheld the penal code provision that furnished the basis of the charges in the case. In fact, the attorney cannot imagine any viable defense to raise at trial. Does the attorney have an obligation to ask the court for permission to withdraw from the representation?

No, because a lawyer for the defendant in a criminal proceeding may nevertheless so defend the proceeding as to require proof of every element of the case

A certain client hired an attorney to represent a client in a litigation matter, but after he filed the notice of representation and the initial pleadings in the case, the opposing party hired Big Firm to represent it. The attorney has already completed three rounds of job interviews with Big Firm and is now simply waiting for their answer, which he hopes will be an offer of employment. Rather than notify the client that an unforeseen conflict of interest has possibly emerged, the attorney simply slows down his work on the case, because if the job offer comes through, he will have to transfer the client's case to another lawyer anyway, and if he does not receive an offer, the potential conflict disappears and he can proceed with the litigation. The attorney thus waits until the last day to respond to any filings or discovery requests, and frequently calls the opposing party asking for more time, which they always grant. Is it proper for the attorney to stall the progress of the case for a while, to allow time for the conflict either to disappear or for him to need to transfer the case to some other lawyer?

No, because a lawyer has a duty to make reasonable efforts to expedite litigation consistent with the interests of the client.

After a terrorist attack that claimed many lives, authorities identified and arrested someone they believed to be the perpetrator of the attack. After the arrest, the prosecutor held a press conference, stating that the suspect was single and lived with his mother in a specific apartment complex in the city, and that the suspect would face charges related to the attacks. Could the prosecutor be subject to disciplinary action by the state bar for disclosing such information about the case to reporters?

No, because a lawyer in a criminal case may not disclose the residence, occupation, or family status of the accused

An associate attorney works at a law firm. The supervising lawyer, who is a partner at the law firm, directs the associate attorney to prepare a petition for a civil case. The associate attorney contacts the client and discusses the facts of the case. During the conversation, the associate attorney realizes that the suit he was directed to file is frivolous and there are no facts to support the claim. The associate attorney discusses his concerns with his supervisor, and the supervising lawyer directs the associate attorney to file the suit, explaining that it will probably settle prior to trial, Following the supervisor's direction, the associate attorney files the suit. Are the associate attorney's actions proper, given that he was merely following orders?

No, because a lawyer is responsible for any violations, including the filing of frivolous suits, even if directed to file such suit by a partner of the firm at which the lawyer works

An attorney, an associate at Big Firm, applied for a patent for a client and successfully obtained the patent. Three years later, another party sued the client for allegedly infringing on one of their patents. The attorney was a necessary witness in the patent infringement matter, and planned to testify on behalf of the client that the client had successfully obtained a patent to the invention in dispute. Two partners at Big Firm, where the attorney worked, handled the representation of the client in the infringement case, pursuant to the client's written consent. Will the two partners at Big Firm be subject to disqualification from representing the client in the patent infringement case, if the attorney will be a witness about the original patent application?

No, because a lawyer may act as advocate in a trial in which another lawyer in the lawyer's firm is likely to testify as a witness

During trial, the plaintiffs complained that the attorney's client had not fully complied with certain production requests during discovery. The judge ordered the attorney to produce the specific records. Yet the attorney believed that his client had no legal obligation to produce the records in question, because they included important trade secrets and were not relevant or material to the current litigation in any way. The attorney openly refused to produce the records and explained his position to the judge. The judge disagreed and ordered the attorney to bring the records to the courtroom the next day. Of course, the attorney did not obey the judge's order. Apart from any potential contempt-of- court sanctions, could the attorney be subject to discipline for violating the Rules of Professional Conduct?

No, because a lawyer may disobey an order from a tribunal when the lawyer has made an open refusal based on an assertion that no valid obligation exists.

A client fired an attorney after the attorney had completed 80 percent of the work involved in the representation. The client refuses to pay any of the fees that were in the original agreement at the beginning of representation. The client also demands that the attorney turn over all papers and documents relating to the representation. Must the attorney immediately return the client's documents regardless of the fees owed?

No, because a lawyer may retain papers relating to the client to the extent permitted by law

During his closing argument at a bench trial, an attorney makes the following statement to the judge: "Your Honor, I know this client, because we grew up together and I have represented him in various legal matters for years. I know that he is an honest person who would never lie or try to take advantage of another person unfairly. In fact, I am doing this case on a pro bono basis because I feel so strongly about the justness of his cause." All of these statements were truthful - Attorney had known Client since childhood and had represented him many times, Attorney admired Client's integrity, and Attorney had offered to handle this case without charging any fee because he believed so strongly that Client was on the right side. Was it proper for Attorney to make these comments during closing arguments?

No, because at trial, a lawyer shall not assert personal knowledge or facts in issue except when testifying as a witness, or state a personal opinion as to the justness of a cause, or the credibility of a witness

An attorney represented a newspaper publisher in a defamation case brought by a popular actor. A radio talk show invited the attorney to participate in their afternoon program and respond to calls from the radio listeners. The first caller asked the attorney to explain the case involving the superhero that the popular actor had played in a recent film. The attorney mentioned that the actor (using the actor's legal name as it appeared in the pleadings, rather than his stage name or the character for which the actor was most famous), and the legal name of the publisher the attorney represented. He also explained that the lawsuit was over alleged defamation by the newspaper, and that the newspaper planned to raise an affirmative defense of truth, that is, it would attempt to show that the stories it printed about the actor were factually accurate, even if they were unflattering. The attorney also mentioned that the actor owns a home and a business in the state, which is a matter of public record, and this is the reason the case is in the courts in that state. Did the attorney violate the Rules of Professional conduct by making these statements on a radio talk show program?

No, because a lawyer may state the claim, the defense involved, the identity of the persons involved, and matters in the public record.

An attorney represented a client in civil litigation. Early in the trial, the attorney had to testify briefly about an uncontested point. The testimony was necessary to establish a minor antecedent point for more critical issues in the case. The attorney made statements that she believed to be true at the time. The next day, while the trial was still underway, the client fired the attorney. When the attorney tried unsuccessfully to dissuade the client from doing so, the client told the attorney that the attorney's testimony was incorrect, and the client also explained some previously unknown information. Under these circumstances, could the attorney keep this information confidential, instead of taking remedial measures to rectify the false statements?

No, because a lawyer must correct a false statement of material fact or law previously made to the tribunal by the lawyer

A certain client was an indigent defendant and received court-appointed counsel for his trial. The trial ended in a conviction. A certain attorney served as his appointed counsel in the case. The client wanted to appeal his conviction, but the attorney believes, for several reasons, that there is no merit to an appeal. The client insisted that the attorney file an appeal before he missed the deadline and agreed that the attorney could withdraw from the case without the client's objection if he would simply file the appeal and provide the client with the opportunity to pursue the appeal pro se or with another lawyer. The attorney presented a "no-merit" letter to the appellate court explaining that his client was appealing his conviction but that the attorney could see no merit in the appeal. Was the attorney's conduct proper, according to the United States Supreme Court?

No, because a lawyer must prepare a brief referring to anything in the record that might potentially support the appeal and leave it to the appellate court to decide whether the appeal is truly frivolous.

A client hired an attorney to represent him in litigation, and he explained to the attorney his version of the incident that gave rise to the dispute with the other party. In response, the attorney took notes on the account that the client provided, and drafted pleadings that alleged the facts as alleged by the client. The attorney did no investigation before filing the pleadings to provide independent verification of the client's version of the story, because he thought that discovery would bring to light the necessary facts to reveal the truth of the matter. Similarly, the attorney submitted as evidence the various documents the client provided to him, without doing his own assessment of the authenticity of the evidence so that he could vouch for the evidence himself. It turned out, as the other side submitted its evidence, that the client's account of what happened was full of fabrications, and some of the evidence was invalid. The attorney did not know the client was being untruthful, but he neglected to make any efforts to verify the client's story before presenting it in court. Could the attorney be subject to discipline for undermining the integrity of the adjudicative process?

No, because a lawyer need not have personal knowledge of matters asserted in pleadings, for litigation documents ordinarily present assertions by the client, and not assertions by the lawyer.

An attorney represented a client in a residential real estate transaction. At the same time, the attorney agreed to represent the defendant in a large class-action lawsuit, an alcoholic beverage maker that understated the alcohol content of its products on its labels, leading to numerous cases of inadvertent intoxication, liver damage from continuous consumption, and a few deaths from overconsumption that led to alcohol poisoning. The client was an unnamed member of the plaintiff class in the class-action lawsuit against the alcohol producer. The attorney did not inform the client that he was representing the defendant in the class-action lawsuit or seek consent from the client or from the alcohol producer. Plaintiffs' counsel in the class action lawsuit discovered this situation, and he asked the court to disqualify the attorney from representing the defendant. Should the attorney be subject to disqualification under such circumstances?

No, because a lawyer seeking to represent an opponent in a class action does not typically need the consent of an unnamed member of the class whom the lawyer represents in an unrelated matter.

A flamboyant billionaire who founded a tech company faced charges of violating securities laws and regulations, and he became the target of an enforcement action by the Securities and Exchange Commission. Reports of the alleged crime generated significant media coverage and commentary, and the arrest and prosecution led to even more media attention. His attorney spoke to the press that assembled on the court house steps, and he gave the following statement: "I'm sure the only one guilty of anything here is the media - everyone knows my client is innocent, that the police framed him." Would it be permissible for the attorney to make such statements to the media, under the Model Rules?

No, because a lawyer should not publicly express any opinion as to the guilt or innocence of a defendant or suspect in a criminal case or proceeding that could result in incarceration

During opening arguments in a criminal trial before a jury, an attorney, who was representing the defendant, closed his statements by declaring, "My client is innocent, I know it in my heart, by the end of the trial, I am confident that you will agree with me that this an innocent man." Are such comments proper for a defense attorney to make during trial?

No, because at trial, a lawyer shall not state a personal opinion as to the guilt or innocence of an accused

An attorney injured his back and leg badly in a car accident. In the aftermath, the attorney became chemically dependent on prescription pain medications. This addiction progressed until it began to affect the attorney's relationships and work habits. The partners in his firm eventually insisted that the attorney seek professional help, so he enrolled in an outpatient rehab program and a twelve-step support group for painkiller addicts. The supervising psychiatrist in the outpatient program expressed concern about the attorney's complete dependence on the painkillers and his diminished ability to function physically or mentally. He advised the attorney to take a leave of absence from work, because he did not believe the attorney could competently fulfill his obligations to his clients. This same concern had prompted the attorney's partners to insist that he seek professional help. Just before enrolling in the outpatient program, a new client had approached the attorney about representing her in a tax dispute with the Internal Revenue Service. The attorney had handled such cases before, but it was not his specialty. The client is so desperate that he tells the attorney privately that he is considering shredding documents to hide some of his tax fraud from the IRS, which the attorney says he should not do, but worries that the client might do it anyway. May the attorney undertake the representation?

No, because a physical or mental condition currently materially impairs the lawyer's ability to represent the client

A certain attorney works as a prosecutor and brings charges against a defendant. In this instance, the attorney clearly has probable cause for alleging that the defendant committed the crime, but he also doubts that a judge or jury will find that the evidence satisfies the standard of "beyond a reasonable doubt." Yet the attorney brings the case anyway, and the defendant wins an acquittal. Has the attorney acted improperly, under the Rules of Professional Conduct?

No, because a prosecutor may bring charges if the prosecutor knows the charges have probable cause.

A prosecutor sees the backlog of prosecutions coming from his office and feels concern about whether all the cases will come to trial in time to comply with the Speedy Trial Act. To expedite some of the simpler cases, the prosecutor asks arrestees to waive their right to a pre-trial hearing, which saves up to a week due to scheduling complications and allows the defendants' cases to come to trial sooner. Because most of the defendants in these cases are unrepresented by counsel, the prosecutor explains that they have a right to a preliminary hearing, but that defendants without a lawyer usually accomplish little or nothing at such hearings, and that the defendant will have a full trial at which to argue his innocence. He also explains that if the defendant believes he can win an acquittal, waiving a preliminary hearing might bring about the defendant's moment of freedom a bit sooner. Most defendants without representation agree to waive their preliminary hearings, which relieves some of the pressure on the local criminal docket and makes this more manageable for everyone. Is the prosecutor behaving properly in this regard?

No, because a prosecutor must not seek to obtain from an unrepresented accused a waiver of important pretrial rights, such as the right to a preliminary hearing

An attorney was a state hearing officer for the Workers Compensation Board. The attorney left that position and opened his own law firm, primarily representing parties before the state Workers Compensation Board. One of the cases is the final rehearing of a case in which the attorney had presided as hearing officer at an initial preliminary hearing and ruled on preliminary matters, but the attorney left the Board without issuing any final decision in the case and the Board transferred the matter to another hearing officer. the attorney represents the injured worker, the client. All the parties involved give informed consent, confirmed in writing, for the attorney to represent the client. Is the attorney subject to discipline for representing the client in this matter?

No, because all the parties involved provided informed written consent to the representation, despite the obvious conflicts of interests at stake

A client asked an attorney to represent him in a lawsuit. The attorney conducts some preliminary research and quickly discovers that the lawsuit is a very long shot. In fact, based on the attorney's survey of the existing judicial decisions in very similar cases, the attorney estimates that they have only a 15 percent chance of winning, and it will depend on an extraordinarily lopsided jury, a strongly partisan judge whose political leanings go in their favor, as well as a mediocre lawyer representing the other side. Otherwise, all things being equal, the attorney advises the client that he is about 85 percent certain that they will not prevail. The client is willing to take risks, however, and urges the attorney to take the matter. The attorney reluctantly agrees, on the condition that he can charge a somewhat higher fee than usual, and files the lawsuit. Could the attorney be subject to discipline for bringing a frivolous claim?

No, because an action is not frivolous even though the lawyer believes that the client's position will not prevail in the end.

A certain attorney represents a client in a drug trafficking case. The client asks the attorney to deliver a package to a friend of the client. The client tells the attorney that the package contains illegal drugs, but he assures the attorney that he will not reveal who made the delivery if police discover that the transfer occurred. the attorney advises that he will not participate in the transfer. The attorney does not advise the court of the client's request and remains the client's attorney on the drug trafficking case. Are the attorney's actions improper?

No, because an attorney does not have to decline or withdraw from cases unless the client demands that the attorney engage in illegal conduct.

A client met with an attorney to discuss certain financial decisions that the client was considering making the future. The attorney discussed the pros and cons of making the decisions but did not give a recommendation to the client. The client went on to make the financial decisions and eventually came under investigation by the IRS for tax fraud. Is the attorney subject to discipline?

No, because an attorney may analyze and give an opinion about the potential consequences of a client's conduct

A client is on trial for a theft case. A certain witness was with the client at the time police state that the client committed the crime at a location far from the crime scene. The client chooses to take the case to trial. For the witness's attendance at trial, the attorney pays the witness a lump sum amount. Are the attorney's actions proper?

No, because lawyers may not pay a witness to attend and testify at a hearing or trial.

A famous professional athlete faced charges for murdering his wife and her male companion one evening outside their Beverly Hills home. The defendant assembled a legal "dream team" of the five most famous criminal defense lawyers from around the country. One of the lawyers was in possession of a handwritten letter from one of the murder victims saying that a drug cartel had been making death threats against the victim for a few weeks. The evidence might have been exculpatory for the defendant, but the lawyer would have to take the witness stand briefly during the trial to authenticate the document or explain how he received it. The document was a hotly contested piece of evidence in the case, but it was not the onlyevidence pointing toward the defendant's innocence or guilt. The prosecutor wanted the court to disqualify the lawyer from representing the defendant if he testified about the letter. The defendant insisted that this would work a substantial hardship on him, because this lawyer was the only criminal defense lawyer in the county with an undefeated record - he had obtained acquittals in hundreds of criminal trials and had never lost a case. Should the court side with the defendant in this case and allow the lawyer to continue as part of his defense team?

No, because disqualification of the lawyer would not work substantial hardship against the client

Three years into the litigation in a complex antitrust lawsuit, it became necessary to have Attorney, who alone represented the defendant corporation, testify as a witness at the trial. Attorney had been present at a private meeting between his client and an industry rival, at which they allegedly discussed a price-fixing scheme, and the testimony of the two rivals (the only ones besides Attorney at the meeting) contradicted each other. The question of what occurred at the meeting was a hotly contested issue in the case, but was only one of many issues in the protracted, extremely complex litigation. The opposing party moved to disqualify Attorney from representing his client after Attorney takes the stand to testify. Should the court disqualify Attorney from representation, or from testifying as a witness?

No, because disqualification of the lawyer would work substantial hardship on the client

A litigation attorney represented a certain defendant in a lawsuit. The client was absent during the final pre-trial hearing about which experts the court would permit to testify at trial for each side. As the hearing was wrapping up, plaintiff's counsel asked the court to have the record sealed in the upcoming trial, and to have reporters banned from the courtroom. The trial involved sensitive information about the mental health of some of the children involved as parties and witnesses in the case. The court agreed but asked if the defendant had any objections. The defendant's attorney tried to reach his client by phone, but he could not get through. There was no obvious reason to oppose the motion, so the attorney agreed on his client's behalf. The judge ordered the record sealed for the upcoming trial. The client never returned the attorney's phone call, and the attorney forgot to mention what had transpired until they were sitting in court on the first day of trial, two weeks later. The client was upset, having planned to use media publicity about the case to draw attention to the other side's exploitation of children as witnesses in litigation. The attorney told the client that the judge would not reverse the decision now that the trial was underway. Were the attorney's actions permissible, under the Model Rules?

No, because even when an immediate decision is necessary during trial, and the exigency of the situation may require the lawyer to act without prior consultation, the lawyer must tell the client about it as soon as possible.

An Assistant U.S. Attorney (federal prosecutor) is working for the Department of Justice, and he must prosecute the defendants arrested in a high-profile sting operation against a terrorist cell. This attorney faces tremendous political and media pressure to win convictions at any cost. As a result, the attorney argues with his supervisor that he is not subject to local ethics rules, as he is litigating exclusively in federal court in cases involving federal law, and that he should therefore be immune from state bar disciplinary proceedings. Is the attorney correct?

No, because federal statute, as well as Department of Justice regulations, subject federal prosecutors to the ethics rules of the state where such the attorney engages in that the attorney's duties.

An attorney received a call from his cousin, who lives in another city, one evening after work. The cousin was serving on a jury in a misdemeanor criminal case, and deliberations were set to begin the following morning. The cousin explained that part of the jury instructions focused on whether the defendant committed the act "knowingly." She is confused about whether that means that the defendant knew that he was committing the act, or that the defendant knew he was doing something illegal at the time. She called the attorney hoping for some clarification. The attorney practiced real estate law and had never handled a criminal case, but he vaguely remembered something about this from his first- year law school course in criminal law. Given that there was no time for him to research the subject, or to create an agreement for representation, and the fact that he had very limited information, the attorney offered the best explanation he could. Was it proper for the attorney to answer her question under these circumstances?

No, because he communicated with a juror about a pending case.

An attorney receives a report from a psychologist that provides the psychologist's professional opinion that the client is unstable. The psychologist's report indicates that the client believes himself to be perfectly sane, and that the client has indicated he will cause harm if the psychologist submits a report stating the client is not stable and sane. The attorney chooses not to provide the report to his client when receives it. Is the attorney subject to discipline?

No, because immediately providing the report to the client may cause harm to the client

An attorney interviewed an expert witness whom he thought he might hire to testify at a client's trial. The attorney explained he was meeting with several expert witnesses and would hire the one who he thought would seem most persuasive to the jury. The expert witness offered to work on a contingent fee basis; if the attorney did not win the case at which the expert testified, no fee would be due. The attorney would have to pay the expert witness only if his testimony was compelling enough to produce a favorable outcome in the case. The attorney thought that this would give the expert an incentive to prepare more thoroughly for trial, and that it would be fairer to the client, who would be left bankrupt if they lost at trial and would have trouble paying the expert's fee anyway. Would it be proper for the attorney to hire the expert witness under such terms?

No, because it is improper to pay the expert witness a contingent fee

An attorney used to work at Big Firm, and three years ago moved laterally to Medium Firm. One of the attorney's former colleagues at Big Firm also left and started a solo practice. While working at Big Firm, the solo received an assignment of literary rights from one of his clients about the client's high-profile case, immediately after the end of the case and the conclusion of the representation. That client also assigned any remaining literary rights to Kingpin Publishers to establish exclusive literary rights in the story. The first attorney's current firm, Medium Firm, represents Kingpin Publishers, and the solo represents himself. Another publisher is interested in acquiring the solo's book or movie script, and is funding the litigation expenses for the solo, even though it is not a client and does not yet have any ownership rights over the story. The case that gave rise to the literary rights occurred while both the solo and the first attorney were working together at Big Firm. The attorney performed some preliminary work on the case--reviewing and indexing a single deposition transcript before the client's trial--but the lawyer who is now the solo represented the client at trial. A paralegal at Big Firm is currently engaged to an editor at Kingpin Publishers, and the two are already living together. The attorney at Medium Firm, who is representing Kingpin Publishers, is in a sexual relationship with another lawyer at his own firm, and both are working in some capacity on the case. Executives at Kingpin Publishers have informed the first attorney that if his firm wins this case, they will hire Medium Firm for all their future legal work, which could be very lucrative in the long term. The solo has just filed a motion to disqualify the first attorney and the rest of Medium Firm from representing Kingpin Publishers in the case, alleging simply that there is a conflict of interest. Does Medium Firm have a disqualifying conflict of interest in representing Kingpin Publishers in this particular litigation?

No, because neither the attorney nor Medium Firm have any disqualifying conflict of interest in this case.

An attorney represents a client in a commercial litigation matter against a small independent bookstore. It is known in the local business community that the opposing party (the bookstore) has been on the verge of bankruptcy for the last two or three years. The facts and law of the present litigation, however, make it a close case--the attorney believes, accurately, that his client has at best a 50 percent chance of winning at trial. At the client's urging, the attorney files frequent motions asking for more time in discovery, more time to respond to the opposing party's motions, and a postponement of the trial date to allow more time to prepare and locate necessary expert witnesses. The attorney thinks that the opposing party may have to close and file for bankruptcy soon, which would make the opposing party's claims moot. The judge has an overcrowded docket and is always glad to grant postponements or more time on various responses. Is it proper for the attorney to take this "time is on our side" approach to litigation?

No, because realizing financial or other benefit from otherwise improper delay in litigation is not a legitimate interest of the client

An attorney prepared a contract for a client in 2015. The matter has concluded, the representation has ended, and the person for whom the contract was prepared is not a client of the attorney or law firm in any other matter. In 2018, while using that agreement as a template to prepare an agreement for a different client, the attorney discovers a material error in the agreement. On those facts, do the Model Rules require the lawyer to inform the former client of the error?

No, because the Model Rules do not require disclosure of material errors to former clients after the representation has ended.

An attorney works for a firm where another lawyer is representing the defendant in a personal injury lawsuit. The other lawyer has represented the defendant for a long time on related, non-litigation matters, but the personal injury lawsuit is a new case. The victim, the plaintiff in the same personal injury lawsuit is a new case. The victim, the plaintiff in the same personal injury lawsuit, was a college classmate of the attorney and he asks the attorney to represent him in the litigation. The attorney has not learned any confidential information yet about the defendant from his fellow associate at the firm, nor has the attorney learned any confidential information from the victim during their preliminary consultation. The firm decides to undertake the representation of the victim as well. The firm will carefully screen the attorney and lawyer from one another, forbidding them to discuss the case with each other or anyone else in the office, and ensuring that they do not have access to each other's files for the case. In addition, neither lawyer will receive a bonus from the fees received for this litigation. Under the Rules of Professional Conduct, is it proper for the attorney to represent the victim, given these circumstances?

No, because the Rules of Professional Conduct impute the conflict of the other lawyer to the attorney, and screening procedures do not apply to conflicts between current clients

An attorney worked for several years for a federal government agency in regulatory enforcement. Big Firm then hired the attorney for a substantially higher salary, and the attorney accepted the position and left her government position. One of the attorney's first assigned cases at Big Firm was a new action by the client against the same government agency for which the attorney had previously worked, defending against an enforcement action that the attorney had initiated while at the agency. The defense will involve challenging the constitutionality of a new regulation that the agency had recently promulgated. While at the agency, the attorney had not been involved with the review and promulgation of any new regulations, including the one at issue in the client's challenge, but instead worked exclusively on enforcement litigation matters. The government agency gives informed consent, confirmed in writing, to the representation. Is the attorney nevertheless subject to disqualification in the client's matter against the attorney's former employer?

No, because the appropriate government agency gave its informed consent, confirmed in writing, to the representation

Husband and Wife wanted to hire a certain attorney to prepare their wills. Before the formalities of representation were final, husband spoke with the attorney privately by phone and disclosed that Husband had been having an affair, and that his lover might be pregnant. Husband forbids the attorney to tell Wife about this. Then the attorney realizes there could be potential conflicts of interest between husband and wife about the wills, distribution of assets, potential challenges to the will by offspring from outside the marriage, and potential claims for child support against Husband's estate. Would it be proper for the attorney to proceed with representing Husband and Wife in preparing their wills?

No, because the attorney cannot violate the duty of confidentiality to Husband, which would be necessary to obtain informed consent from Wife.

An attorney worked for several years for a federal government agency in regulatory enforcement. Big Firm then hired the attorney for a substantially higher salary, and the attorney accepted the position and left her government position. One of the attorney's first assigned cases at Big Firm was a new action by the client against the same government agency for which the attorney had previously worked, challenging the constitutionality of a new regulation that the agency had recently promulgated. While at the agency, the attorney had not been involved with the review and promulgation of any new regulations, including the one at issue in the client's challenge, but instead worked exclusively on enforcement litigation matters. Is the attorney subject to disqualification in the client's matter against the attorney's former employer?

No, because the attorney did not participate personally and substantially in the matter as a public officer or employee

An attorney worked for several years for a federal government agency in regulatory enforcement. Big Firm then hired the attorney for a much higher salary, and the attorney accepted the position and left her government position. One of the attorney's first assigned cases at Big Firm was a new action by the client against Conglomerate Corporation. The attorney had worked on an enforcement against Conglomerate Corporation and learned confidential government information about the entity during the litigation, but the attorney does not know, and has no reason to know, that the information is confidential government information. The attorney is under the reasonable impression that all the information she learned about Conglomerate Corporation is now public information. The government agency gave its informed consent, confirmed in writing, to the representation. Is the attorney nevertheless subject to disqualification in the client's matter against the attorney's former employer?

No, because the attorney does not have confidential government information about Conglomerate that she knows is confidential government information.

A trial judge is going through a divorce, and he hired an attorney to represent him. The attorney's law firm partner is representing another client who is appearing before the same judge in his personal injury lawsuit. The judge and the litigation client both give written informed consent to the representation despite the potential conflicts of interest. Even so, the judge is trying to keep the divorce quiet until after the upcoming elections, because this occurs in a state with elected judges. The judge therefore refuses to disclose to the parties in the personal injury case that counsel for one side is from the same firm as the lawyer representing the judge in his pending divorce. Neither the attorney nor his partner can reveal to opposing counsel in the personal injury case that their firm represents the judge, due to their duty of confidentiality. The judge believes he will be unbiased in the personal injury case, even though he is the client of a partner of one of the lawyers in the case, so the judge does not need to disqualify himself from the case. The Code of Judicial Ethics does require, however, that the judge disclose the representation to the litigants appearing before him, which the judge has refused to do at this time. Can the attorney continue representing the judge in his divorce?

No, because the attorney must withdraw from the representation of the judge under these circumstances.

An attorney was preparing a will for one of her wealthy elderly clients. The client had no surviving family members - her spouse had passed away years before, as had her siblings, and she had no children. The client asked the attorney for suggestions about potential beneficiaries of the estate, besides her favorite charities, and she offered to leave the attorney some items. The attorney replied, "Well, I've represented you on various matters over the years, and I have always looked out for your best interests, so I would not object if you included me in the will. I've always admired your collection of antique furniture and books." The client was delighted by the idea and instructed the attorney to include a provision in the will bequeathing all the antique furniture and books in her large home to the attorney. The attorney prepared the will as instructed and the client executed it. Was the attorney's conduct proper?

No, because the attorney should not have prepared the will if the document made a significant bequest to the attorney.

A certain state has specialized family courts that handle divorces, child custody, child removal cases brought by state social service agencies, and spousal or child support enforcement. An unmarried couple had split up but they had two children, and the family court judge awarded custody of the children to the single father, and ordered the mother of the children to pay $500 per month in child support to the father. A few months later, the judge left the family court and returned to private practice, specializing in family law, which allowed him to draw on his valuable experience as a former judge in the family court. One day, the father from the case described above came for a consultation, and he explained that the mother of the children had been delinquent for the last two months in paying child support to him. Would it be proper for the judge to represent the father in the action to enforce the child support order?

No, because the attorney would be representing a party in seeking enforcement of his own order from his time on the bench.

After much effort, an attorney located a witness who could fully corroborate his client's story and could impeach the testimony of the opposing party's star witness. The witness, however, was afraid of retaliation from others if she testified, and did not want to be involved. The attorney offered the witness $10,000 to appear at the trial for one afternoon and testify for an hour or two. The witness reluctantly agreed. Was it proper for the attorney to offer to pay a favorable witness to undergo the trouble of testifying at the trial?

No, because the common law rule in most jurisdictions is that it is improper to pay an occurrence witness any fee for testifying apart from expenses

A district attorney (D.A.) discovers a single item of evidence that partly undermines the state's case against a criminal defendant--the state's star witness in the case, the prosecutor learns, had a suspension from high school for an instance of egregious plagiarism. The D.A. believes this is not material in that it would not change the outcome of the case, because the incident occurred ten years ago, and the witness is now an undercover police officer-informant. In fact, the D.A. believes it is trivial, and he is correct that the item would not fall under the duty of disclosure set forth by the Supreme Court in Brady v. Maryland. At the same time, the defense lawyer in the case has a reputation for making much ado about nothing, prolonging trials unnecessarily with tedious minutia. The D.A. decides to keep the information about the high school suspension to himself and let defense counsel discover it on his own if he wants. Did the D.A. act within the requirements of the Model Rules?

No, because the evidence tends to negate the guilt of the accused

An attorney is representing the defendant in a highly publicized civil trial between two celebrities. On his way into the courthouse on the day of jury selection, reporters gather around the attorney hoping for comments. The attorney explains that his client has agreed to take a polygraph test proving that he is telling the truth about the disputed matter, but that the opposing party has refused to take a polygraph test, which suggests that the other person is hiding something. The attorney has his client's permission to talk to the media. Opposing counsel is standing nearby waiting for his turn to talk, and he expresses no objection to the first attorney giving interviews like this, or to the attorney's comments. Were the attorney's statements proper?

No, because there is a presumption of prejudicial effect on the proceedings when a lawyer comments publicly about the performance or results of any examination or test or the refusal or failure of a person to submit to an examination or test.

An attorney tells a client that certain features of the client's business proposal would constitute money laundering under current federal statutes. The discussion goes through the statute in detail, and the attorney explains why the course of action would meet the statutory definition of money laundering. In addition, the attorney discusses the various monitoring and reporting mechanisms that federal enforcement agencies have in place to detect money laundering, to convince the client that he would not escape arrest and prosecution if he proceeds. The client absorbs the information and uses it to structure a more elaborate money-laundering scheme. He exploits some ambiguity in the statute and the reporting requirements to make his enterprise much more difficult to detect, and this complicates enforcement and prosecution efforts against him. Overall, the attorney's advice turned out to be incredibly useful to the client in avoiding detection and expanding his criminal enterprise. Is the attorney a party to the client's course of action?

No, because the fact that a client uses advice in a course of action that is criminal or fraudulent of itself does not make a lawyer a party to the course of action.

The plaintiffs' lawyers in a large class action suit against an insurer contacted an attorney at another firm seeking some advice. The attorney they called was a former commissioner with the state Insurance Commission, so he had vast insider knowledge of the regulation of the insurance industry in that state. The attorney talked to the plaintiffs' lawyers for thirty minutes on the phone, during which the plaintiffs' lawyers shared some confidential information about the class action, including their theories of the case and litigation strategies. A year later, the attorney left his firm and went to work for Boutique Firm. Around the same time, the defense team representing the insurance company in the class action - three lawyers - also moved as a group to the same Boutique Firm, but to their office in another city.Boutique Firm because the counsel of record for the defendant insurer. The plaintiffs' lawyers learned of this convergence, and they *expressed concern that an attorney who had confidential information from their side of the case was now working with opposing counsel at the same firm. Boutique Firm immediately implemented strict screening procedures, and the managing partners made inquiries to confirm that the attorney had not already transmitted confidential information to the defense team for the matter, who were working in another office. The plaintiffs' lawyers were unsatisfied and filed a motion to disqualify all the lawyer in Boutique Firm from representing the insurer defendant in the class action. While the motion was pending, the attorney who had the confidential information left Boutique Firm to accept a government appointment. Should the court disqualify Boutique Firm, due to the imputed conflict of interest?

No, because the firm avoided imputation of the conflict by implementing effective screening measures, and the fact that the lawyer was geographically in another office, and has already departed to work elsewhere, also support denying the motion.

An attorney represented a client who suffered from a partial mental impairment, though the client was legally competent and was able to participate in the decisions about the representation. The representation pertained to a litigation matter. The client's mental health issues were a private matter, and the opposing party was unaware of the situation. Due to the client's forgetfulness, it took the lawyer more time than usual to obtain information and documents from the client to comply with discovery and production requests. The attorney repeatedly missed deadlines for production during the discovery phase, and eventually the opposing party moved for sanctions. When the judge demanded an explanation from the attorney, the attorney panicked and blamed it on the client's deteriorating mental health, though the attorney had not discussed this potential disclosure with the client. Surprised, the judge offered to give the attorney and his client additional time to comply with discovery requests. The opposing party revisited their own trial strategy after this development. Was it proper for the attorney to make this disclosure when facing sanctions for missing discovery deadlines?

No, because the granddaughter's financial interests influenced her judgment and created a conflict of interest.

A prospective client consulted with an attorney about the possibility of securing legal representation in a matter. During the conversation, the client shared openly with the attorney about the strengths and weaknesses of her legal claims, including some personal information that would be embarrassing if it became public. Some of the information indicated the prospective client may have already waived some of her legal claims, and she may have been partly at fault on other points. The attorney considered it for a few minutes and then declined the representation, because he felt he could not devote adequate time to the case, and he thought the case was too problematic. In addition, he was skeptical that the prospective client would be able to pay his fees. A few weeks later, some of the attorney's other matters settled sooner than expected, freeing up his schedule, and another prospective client came for a consultation, who turned out to be the opposing party in the legal matter that the attorney had recently declined. This new prospective client had already gathered some convincing evidence supporting his side, and was wealthy, so paying the attorney's fees was not an issue. Would it be proper for the attorney to proceed with representing this new prospective client?

No, because the information learned from the first consultation with the other party would be so helpful to the new client, and so harmful to the individual the attorney declined to represent.

An attorney formerly represented Pharma Giant in obtaining FDA approval to market prescription drug Opticoton for treating diseases of the eye. Drug Conglomerate has now asked the attorney to help it obtain FDA approval for sale of prescription drug Dermicon for treating diseases of the skin. Drug Conglomerate is also interested eventually seeking FDA approval to market a variant form of Dermicon to treat diseases of the eye. If the FDA approved this application, Dermicon would significantly cut into the market share of Opticoton and Pharma Giant's profits from the drug. The attorney gleaned confidential information while representing Pharma Giant that relates extensively to work that the attorney would undertake in helping Drug Conglomerate obtain approval for Dermicon to be marketed as an eye salve, but none of the information would relate to Dermicon's use as a skin medicine. Drug Conglomerate and the attorney agree that the attorney's work will relate only to FDA approval for use of Dermicon to treat diseases of the skin. With this limitation in place, would it be impermissible for the attorney to represent Drug Conglomerate in obtaining the initial approval for Dermicon as a skin medicine, without obtaining Pharma Giant's informed consent?

No, because the limitation on the representation removes any substantial relationship between the two matters or concerns about confidential information from a former client giving a strategic advantage to the new client.

During the discovery phase of business litigation, Conglomerate Corporation receives a discovery request asking for "all documents, memoranda, emails, or other internal correspondence related to the transaction that is the subject of this dispute." An attorney represents Conglomerate Corporation. Thousands of documents stored in electronic format on Conglomerate's computers and servers would potentially fall under this request for production. The attorney proposes to opposing counsel that they produce the requested documents in electronic form on a set of compact discs, and the opposing counsel readily agrees. After receiving the production request, the attorney began using software to scrub the metadata from documents - electronically embedded information about the name of the user whose computer created the document, the date and time of creation, redlined changes from each stage of editing, and comments that other readers added to the document before it took its final form. Proposed contracts, letters to business partners, and memoranda between managers all have their embedded metadata erased. Was it proper for the attorney to scrub the metadata from electronic documents before delivering them to the other party in response to a discovery request?

No, because the main reason for scrubbing metadata is to conceal information that might be useful to an opposing party or tribunal in the present litigation.

A certain attorney is representing the defendant in a highly publicized trial. On his way into the courthouse on the day of jury selection, reporters gather around the attorney hoping for comments. The attorney explains that his client has a perfectly clean criminal record, while the state's star witness is already serving time on a felony drug conviction. In his opinion, he says, the client is innocent and should receive an acquittal, but he does not explain the defense theory of the case. The attorney declares that he has his client's permission to talk to the media, which is true, and that the prosecution expressed no objection to him giving interviews like this on the courthouse steps in previous cases. Were the attorney's statements proper?

No, because the official Comment to the Model Rules says that expressing an opinion about a party's guilt or innocence, or about the criminal record of a party or witness, is more likely than not to have a material prejudicial effect on a proceeding.

An attorney represents a client in a civil litigation matter. As they prepare for trial, at which the client will testify as a witness on his own behalf, the attorney realizes that the client is unlikely to tell the truth, even though the client insists he will be completely truthful. Even so, the attorney believes there is some chance that the client is indeed telling the truth, but he is about 70% certain that the client is being untruthful, despite the client's protestations. Does the attorney have an ethical duty to try to prevent the client from presenting testimony that the attorney believes is unlikely to be true?

No, because the prohibition against offering false evidence only applies if the lawyer knows that the evidence is false, and a lawyer's belief that evidence is false does not preclude its presentation to the trier of fact.

A husband and wife decide to divorce and reach an agreement to share the same lawyer in hopes of saving money. They hire an attorney to represent each of them in Family Court for the dissolution of marriage. The attorney explains that there is an obvious conflict of interest here, but the husband and wife insist, and sign informed consent forms waiving the conflict and their rights to assert any future claims related to the conflict. The husband and wife have no children, and they have always kept separate bank accounts. Each purchased their own car from the money in their own bank account and each car's title is in only one name. They live in an apartment whose lease is expiring soon, so there is no real property to divide. Would it be proper for the attorney to represent both in the divorce?

No, because the representation involves the assertion of a claim by one client against another client represented by the lawyer in the same litigation or other proceeding before a tribunal.

An attorney practiced family law. One of her previous cases involved representing a man who wanted to remove his estranged wife as the beneficiary of his life insurance policy. The attorney was successful in this undertaking for the client. Two years later, the client died, never having divorced his estranged wife. The wife asked the same attorney to represent her and to convince the insurer to undo the change in beneficiary so that the wife could receive the proceeds from her husband's life insurance policy. This representation would require extensive negotiation with the insurance company to convince the company that the previous change was invalid. Prior to his death, the husband, whom the attorney had represented, had sent the wife a letter saying that he made a mistake in removing her as the insurance beneficiary, and that he did not understand at the time the consequences of what he was doing. Would it be proper for the attorney to represent the wife in this matter?

No, because the subsequent representation would require the attorney to attack the same work the attorney performed for the former client.

After obtaining a favorable verdict trial, a client asked the court to award attorneys' fees, which was permissible under relevant law. An attorney had represented the client throughout the litigation and now had to testify as a witness about the fees he had charged during the representing, authenticating, explaining, and justifying both the billable hours recorded on the timesheets and the lodestar rate for his legal services. Was it improper for the attorney to testify as a witness in the same proceeding in which he had represented a party as trial counsel?

No, because the testimony relates to the nature and value of legal services rendered in the case

A certain client needed to sell a parcel of real estate to pay off a large amount of credit card debt. He brought this situation to the attention of his attorney, who was representing him in his interactions with collection agencies and credit bureaus. The attorney offered to purchase the property immediately for the full amount of the client's outstanding credit card debt - just over a hundred thousand dollars - without delaying the matter by arranging a mortgage first, or having the property appraised. The client was disappointed, because he thought the property was worth more than that, but he agreed due to his dire financial circumstances. The attorney fully disclosed the terms of the purchase to the client, in understandable written form, and advised the client in writing that it would be prudent to consult with another lawyer about the transaction, which the client could not realistically afford to do. The client gave written, informed consent to the terms of the sale and the attorney's role in the transaction. Two months later, the attorney sold the property to a developer for three times the amount he had paid for it. Did the attorney act within the requirements of the Model Rules?

No, because the transaction was objectively unfair.

A certain attorney is representing the defendant in a highly publicized criminal trial. On his way into the courthouse on the day of jury selection, reporters gather around the attorney hoping for comments. The attorney explains that his client is still considering whether to enter a guilty plea to lesser charges, as the prosecutor's offer is still open, and that they are waiting to see how jury selection goes before deciding whether to plead guilty or proceed to trial. The attorney also explains that his client has never actually confessed to the crime charged, despite several lengthy interviews with the police and the client's admitting that he was near the scene of the crime when it occurred. The attorney has his client's permission to talk to the media, and the prosecution has expressed no objection to him giving interviews like this on the courthouse steps in previous cases. Were the attorney's statements proper?

No, because there is a presumption of prejudicial effect on the proceedings when a lawyer comments publicly about the possibility of a guilty plea, or a party's refusal to confess to a crime

A certain client is an indigent criminal defendant and a certain attorney is his court- appointed counsel. The trial is taking place in a rural county where only a handful of lawyers practice law. Before appointing the attorney to represent the client, the court had tried to appoint five other local criminal defense lawyers, one after the other, but each was unable to provide representation due either to a conflict of interest or because their current caseload would have precluded them from providing competent representation. In fact, the attorney was the last lawyer on the court appointments list. Unfortunately, the attorney also needed to serve as a witness during part of the trial, to authenticate a piece of evidence, and the authenticity of the evidence was a matter of dispute in the case. In addition, the attorney realized that his testimony would radically contradict the testimony of his own client, though the attorney still believed he could obtain an acquittal by impeaching the prosecution's star witness. May the attorney continue to represent the client and testify as a witness in this matter?

No, because there is likely to be substantial conflict between the testimony of the client and that of the lawyer, so the representation involves a conflict of interest that requires compliance with the rules about conflicts

A client hired an attorney to represent him in suing his employer for wrongful termination. The attorney proposed a fee arrangement that made the fees contingent on the outcome, and he included in the fee agreement that the attorney would advance the costs of litigation. The attorney lost the case at trial, and the client then refused to pay back the costs that the attorney had advanced beforehand. Can the attorney force the client to repay the litigation costs that the attorney advanced to him?

No, because under the fee agreement, the client had to repay the attorney only if they won the case.

A jury convicted a defendant of murder, and they sentenced him to death. His lawyer at trial was unimpressive, and there were potential points to raise in an ineffective assistance of counsel appeal. For his appeal, the defendant used a different attorney from the same firm as his trial lawyer - one of the lawyers at the firm handled trials, and the other appeals. Can the appellate attorney from the same small firm as the trial lawyer handle this appeal?

No, because under the legal standard for ineffective assistance of counsel, the appeal would potentially require the attorney to disparage the representation of his own colleague as being unreasonably poor.

An attorney has applied to make a lateral move from her firm to Big Firm, and she has already gone through the first two of three rounds of interviews for the position. Then the attorney agrees to represent a client in filing a breach of contract claim against Construction Company over a commercial development project. Big Firm is representing Construction Company, and the firm's lawyers drafted the contract that forms the basis of the client'scomplaint. The client claims that Construction Company breached a certain provision of the contract that is ambiguous; Construction Company is confident that its conduct falls within the contractual language in that provision. Is it proper for the attorney to undertake representation of the client in this case?

No, because when a lawyer has discussions concerning potential employment with an opponent of the lawyer's client, or with a law firm representing the opponent, such discussions could materially limit the lawyer's representation of the client.

An attorney represents a defendant in a prosecution for rape. The client turned down several other experienced criminal defense lawyers who offered to take the case and hired the attorney to represent him. The client saw the victim early in the evening on the date when the rape occurred, but he has a solid alibi, supported by multiple credible witnesses, that he was nowhere near the scene where the rape occurred at the time that it happened, and no DNA tests link the client to the rape. The only evidence against the client, in fact, is the victim's memory of seeing him early that evening and feeling uncomfortable around him, as if she could sense that he was a sexual predator. Her rapist wore a mask, so she could not identify his face, but he was the same height and build as the client, so she is convinced he is the perpetrator. Despite the weakness of the evidence against him and his airtight alibi, the client is furious about the false accusation and wants to teach the victim a lesson. He informs the attorney that he plans to take the stand and testify that the victim has a reputation among his friends for being promiscuous, that when he saw her that evening she was wearing provocative clothing, and that he believes she was "asking to be raped." The attorney finds this repugnant, but he believes the client is truly innocent of the rape in this case, and the client is likely to receive an acquittal with or without this testimony attacking the victim's character and reputation. The attorney believes the court will allow him to withdraw from the case and that the client could easily hire one of the other lawyers to take over the representation. Is it improper for the attorney to withdraw from the representation, if he agrees with the objectives the client is pursuing (acquittal) but disagrees with the actions the client plans to take?

No, because where the lawyer agrees with the overall objectives of the client, a lawyer may withdraw from a case if the client insists upon taking action that the lawyer considers repugnant

A certain attorney agrees to represent a group of three individuals in the same matter, a business transaction. Their interests are not directly adverse. This attorney has represented each of the clients in separate matters previously, and he is already working under a retainer to do legal work for each under the same hourly rates. Two of the clients are currently traveling overseas, but everyone agrees to the representation by conference call. The attorney explains potential conflicts of interest that could arise in common representation, and all clients consent orally to the common representation despite the potential conflicts. Then the attorney proceeds with working on their matter for three weeks until all the clients are back from traveling and can sign written consent forms. By that time, the attorney has completed 50 hours of work, and has acquired significant confidential information by and about each of the three clients. Would the attorney be subject to discipline for performing this legal work before obtaining written consent to the conflict by each conflict?

No, it was not feasible to obtain or transmit the writing at the time the client gives informed consent, so the lawyer could obtain or transmit it within a reasonable time thereafter.

A large municipality has a labor dispute with its police union. The chairperson of the city council is a lawyer - she works for the city council part time, and she also has a law partnership with one other lawyer. As chairperson of the city council, she has the final word on which items will be on the council's agenda at each meeting. A few city council members who support the police union want their modest proposal for police pension reform to be on the agenda at an upcoming meeting.The police pension fund has not received its full contribution from the city for several years, and even though all current retired officers are receiving their full pension benefits on time, a wave of expected retirements over the next few years will create a crisis if the pension remains underfunded. The proposal would require the city to make a significant increase in its annual contributions to the fund, which would force cuts elsewhere in the city budget. The chairperson's law firm partner represents the police union in a variety of legal matters. The chairperson has screened herself from the representation, will receive no share of any legal fees from her partner's representation of the union in the pension reform matter, and she will recuse herself from debating or voting on the proposal at the city council meeting. May the attorney who is the chairperson's partner continue to represent the police union?

No, due to imputation of the chairperson's conflict of interest to her law firm partner.

A prosecutor learned that his cousin was serving jury duty in a criminal trial in the prosecutor's own district, although another lawyer from the prosecutor's office was handling that trial. Nevertheless, before and during the trial, the prosecutor repeatedly communicated with his cousin about the trial, even though the prosecutor himself was not involved in the matter. Was it permissible for the prosecutor to have this contact with the juror?

No, even though a colleague of the prosecutor was handling the trial in which the cousin was a juror, the prosecutor's conversation violated the ethical prohibitions on ex parte communication with jurors.

An attorney represented a client in a case for violation of federal employment laws by the client's former employer. The employer filed a motion for summary judgment because the attorney's client had left the company prior to the effective date of the relevant statute. The attorney requested repeated extension for more time to respond to the summary judgment motion, which the court at first granted, but eventually denied. It turned out that the attorney knew the statute as enacted was not retroactive, but he was hoping some case law might develop during the delay that would help his case. There were no pending appellate cases considering the issue of retroactivity for this statute. Was it permissible for the attorney to request more time to file a response when the sole reason for doing so was the remote chance that some courts would modify the law that governed the case?

No, given that the issue was not pending before any appellate courts at the time, it was improper for the attorney to request these extensions.

An attorney was representing a criminal defendant, and he agreed to meet with one of the defendant's co-conspirators to learn more about what happened and to discuss what to expect as the case proceeded. At the beginning of the meeting, the co-conspirator gave the attorney a dollar bill, saying, "This is to establish attorney-client privilege." The discussions then proceeded as planned. Later, the co-conspirator turned state's witness against the attorney's client, and near the end of the proceedings, the prosecution moved to disqualify the attorney due to his conflict of interest. Did it establish attorney-client privilege and a conflict-of-interest problem when the co-conspirator gave the attorney a dollar?

No, giving money to the attorney did not create any attorney-client relationship

An attorney agreed to represent an indigent client pro bono in a landlord-tenant dispute. The client had no money for food or medicines or other living expenses, so the attorney gave the client some money for food, transportation, medicine, and other basic living expenses. The representation was successful, and the client was able to stay in the apartment, so the representation concluded. The attorney represented other indigent clients on a pro bono basis and advertised this practice on his website, including a short video in which the attorney claimed he was always willing to help strugglingclients in this way. Were the attorney's actions permissible under the Model Rules?

No, if a lawyer representing an indigent client gives the client modest contributions for food and similar basic necessities of life, the lawyer may not publicize or advertise a willingness to provide such gifts to prospective clients.

An attorney agreed to represent an indigent client pro bono in a litigation matter. The client had no money for food or medicines or other living expenses, so the attorney gave the client some money for food, transportation, medicine, and other basic living expenses. The representation was successful, and the client prevailed an recovered a modest amount of compensatory damages, so the representation concluded. During the representation, the attorney learned that the client had a wealthy brother with whom the client would not associate due to an offense that had occurred years before. A month after the representation ended, the client died, and the wealthy brother contacted the attorney to thank him for all he had done for the client and offered to reimburse the attorney for the financial assistance he had provided during the previous months, which over time added up to a few thousand dollars. The attorney did not ask for this, but he gladly accepted it. Did the attorney act properly, according to the Model Rules?

No, if a lawyer representing an indigent client gives the client modest contributions for food and similar basic necessities of life, the lawyer may not seek or accept reimbursement from a relative of the client or anyone affiliated with the client.

A certain employee at Big Bank faced criminal charges for alleged embezzlement of bank funds, so she retained an attorney to defend her against the charges for a flat fee of twenty thousand dollars, which the client could pay in monthly installments. The next day, a different Big Bank employee confessed to having taken the money, so the prosecutor dropped the charges against the first suspect, that is, the employee who had hired the attorney. The attorney had done nothing on the case except the original consultation with the bank employee as a prospective client, checking for conflicts of interest, and drafting an appearance for the court. The prosecutor was not aware that the original defendant had retained counsel; the withdrawal of the charges was due solely to another individual confessing to the crime. The attorney did not have to decline any other potential clients when he agreed to undertake the representation. After confirming with the client that the matter was over and further representation was unnecessary, the attorney sent the client a bill for the $15,000 flat fee. Was it proper for the attorney to do this?

No, it would be unreasonable for the attorney to charge twenty thousand dollars for doing so little.

An attorney worked for Big Firm in their intellectual property department, specializing in patent applications and patent enforcement, as well as some trademark disputes for clients. Unbeknownst to the attorney, the regular litigation department at Big Firm undertook representation of a trucking company in defending against a personal injury lawsuit over a roadway accident involving one of the trucks. The attorney worked in the Washington, D.C. office of Big Firm has its own local computer network for sharing documents and files between lawyers there. It is possible for lawyers at Big Firm to access the networks of other satellite offices, however, with a special login that most lawyers never use. The attorney has never accessed the files of the Dallas office except for one trademark case four years ago. The attorney did not make partner at Big Firm, so he left and went to work for a small plaintiff's firm in Kansas. One of the attorney's first case assignments was the same truck accident case in which Big Firm was defending Trucking Company; the attorney's new firm represents Plaintiff in the case. The attorney was not aware of the case or that Big Firm represented Trucking Company until the new firm assigned him to the case as second chair on the litigation. Is the attorney subject to disqualification in this matter?

No, if a lawyer while with one firm acquired no knowledge or information relating to a particular client of the firm, and that lawyer later joined another firm, neither the lawyer individually nor the second firm are disqualified from representing another client in the same or related matter even though the interest of the two clients conflict

An attorney worked as an associate for several years at Big Firm, and while she worked there, she started a sexual relationship with one of the clients of the firm, whom the firm had already been representing before she began working there. Nevertheless, the attorney did not make partner at the firm due to this incident, even though it had not resulted in a disciplinary action, so she eventually left and started her own practice. She then made radio commercials to attract new clients to her firm, in which she boasted that she had been an associate at Big Firm, but that she did not make partner there merely because she had sex with a client a few times. This advertisement brought many new male clients to her firm. One day, the attorney was flying cross-country to attend a deposition on behalf of one client. This counted as travel time she would ordinarily bill to that client, as permitted by the ethical rules. During the flight, she decided not to watch the movie or read a book, but to work instead on drafting a motion for another client. Would it be permissible for her to charge both clients, each of whom agreed to hourly billing, for the time during which she was traveling on behalf of one and drafting a document on behalf of the other?

No, if the attorney flies for six hours for one client, while working for five hours on behalf of another, she has not earned eleven billable hours.

A legal secretary in a law firm is married to the owner of an independent retail-clothing store. The firm undertakes representation of a clothing wholesaler, who is suing the same independent clothing store over nonpayment for shipments of merchandise. The legal secretary's husband hires another firm to represent his store in the lawsuit, and his lawyer asks the court to disqualify the legal secretary's firm because of her position there. Should the firm be subject to disqualification?

No, if the firm screens the legal secretary from any involvement in the case or from access to any confidential information about the case

A client fired an attorney after two weeks of representation, long before the matter was complete. Client had prepaid a large refundable retainer, against which the attorney was to draw his fees as the representation went on. The client therefore has fully paid her fees up to that point to the attorney. The attorney is very upset about the client discharging him without cause and believes it is unfair and wrongful. The attorney refuses to return the remainder of the fees, and refuses to turn over any documents from the representation to the client. Is it proper for the attorney to take this course of action, if indeed the client had no good reason to discharge him?

No, it is improper for an attorney to retain either the unused funds or the documents

An attorney consulted with a potential client, a plaintiff in a personal injury lawsuit, and the client agreed to pay the attorney a contingent fee based on a percentage of the award in the case, which appeared to be a complicated matter that would necessitate the testimony of experts at trial, and depositions of the experts and other witnesses beforehand. As the consultation concluded, the client and the attorney signed an engagement contract for the provision of legal representation, which stipulated that the attorney would a contingent fee based on a percentage (one-third) of the award in the case. The document that the client signed clearly explained the percentage that should accrue to the attorney the event of settlement, trial, or appeal; litigation and other expenses that the attorney would deduct from the recovery; and that such deductions would come out of the total before the calculation of the contingent fee. The letter also discussed all potential expenses for which the client could be liable, if the client prevailed in the case or not. To impress the client, the attorney called the defendant's counsel in the matter, at the end of the consultation, while the client was still sitting in his office. Over the phone, the attorney explained the plaintiff's injuries, the medical expenses the plaintiff had incurred, and the one-million-dollar recovery they would seek in the lawsuit they planned to file. The defendant's lawyer checked with the defendant, who was standing next to him at the time, and then immediately agreed to pay the full amount that the client was seeking to recover - a million dollars - without litigation. Would it be permissible for the attorney to charge the client one-third of the million dollars as a fee, given these facts?

No, it is not reasonable for the attorney to charge over three hundred thousand dollars in fees for making one phone call at the end of the first consultation with the client.

An attorney represented a client in commercial litigation. One component of the case necessitated expert testimony about the economic losses suffered, interest calculations, and potential mitigation costs. The attorney hired the most famous expert witness that he could find on such matters, one who would easily be able to counter the opposing party's expert at trial. With the client's consent, the attorney agreed to pay the expert a six-figure retainer fee to review the case documents plus $2000 per hour for any courtroom time. Would the attorney be subject to discipline for paying the expert witness a huge sum to help with the case?

No, it is proper to compensate an expert witness on terms permitted by law, so long as it is not a contingent fee.

An attorney represented a defendant in a criminal proceeding. While preparing for trial, the defendant told the attorney that the main witness for their side, the defendant's friend who planned to corroborate his alibi, intended to lie on the witness stand. The attorney tried to dissuade the client and the witness from this course of action. He explained that committing perjury could subject the client to additional criminal changes, and that a rigorous cross- examination from the prosecutor would certainly expose the lies. Even so, the witness insisted on testifying at trial and stated his intention to present a fabricated version of the alibi. Should the attorney allow the witness to testify, and examine the friend as a witness, under these circumstances?

No, the attorney must either disclose the contemplated perjury to the tribunal, or refuse to call the witness, or withdraw from the representation.

An elderly client retained a local attorney to handle his estate planning matters. The attorney routinely practiced in this area and had a good reputation in the legal community. Unfortunately, the client had early-stage dementia and suffered from increasing forgetfulness and occasional confusion. The client always recognized the attorney as his lawyer, but he would sometimes forget her name and whether she practiced with other lawyers in the same firm. The client lived alone and took care of his own daily needs without incident, but he depended on relatives to for rides, as he could not drive, and friends or relative would check in on him every day. It was frustrating for the attorney to discuss any complex legal questions in the estate plan with the client, because he would frequently ask a question a few minutes after the attorney had given a thorough explanation about the same point. It was clear, however, that the client intended to divide the estate equally among the surviving heirs. Over time, the attorney included the client in fewer and fewer of the decisions, such as whether to set up a trust, or whether to sell off some of the real property in the estate rather than bequeathing the title to one or more heirs. These decisions were standard practices among estate planning lawyers and were objectively competent. Overall, the attorney fulfilled the client's objectives and protected the client's interests, even though she made many important decisions on her own without consulting the client. Was it proper for the attorney to use her own judgment on the practical questions of estate planning, without including the client in these decisions?

No, lawyers have an ethical duty to maintain, as much as possible, a normal client-lawyer relationship with the client, even when the client suffers from diminished capacity.

A businessperson hired a certain attorney to represent her in a tax dispute with the government, in which the government accused her of hiding assets in overseas accounts and failing to report income from certain obscure investments. During this representation, the attorney learned extensive private financial information about client, but the representation ended at the resolution of the tax case. Several years later, after the termination had ended, the husband of the client filed for divorce. The attorney was the only lawyer the husband knew, so he retained the attorney to represent him in the divorce against the client. Her new lawyer moves to have the attorney disqualified from representing the husband, but the attorney claims that the matters did not relate to each other enough to merit disqualification. Is the attorney correct?

No, matters are "substantially related" if there is a substantial risk that confidential information from the prior representation would materially advance the client's position in the subsequent matter, such as personal financial information.

An attorney agreed to represent a client in a litigation matter and had checked for conflicts of interest regarding the opposing party. Soon after agreeing to take the case, however, the attorney learned that opposing counsel was one of his closest friends. They gave each other gifts for their birthdays and celebrated special occasions together, socialized together regularly, and their children were friends and his children had gone for sleepover parties at the home of opposing counsel. The two families had even vacationed together on two occasions. In fact, opposing counsel was one of the few friends that the attorney felt he could talk to about his personal problems. Even so, the matter seemed straightforward and the positions of the two parties were close enough that a quick settlement seemed likely. Does the attorney have an obligation to inform the client about the nature of his friendship with opposing counsel and obtain written, informed consent from the client?

No, mere friendships between the lawyers on opposing sides of a case do not create the same type of conflict as family relationships or intimate romantic relationships.

Conglomerate Corporation owns a little more than half the stock of Giant Company. Conglomerate's stock, in turn, is public, available on the public stock exchange, as is the remainder of the stock in Giant Company. The president of Conglomerate Corporation has asked Attorney Stevenson to represent Giant Company in a deal by which Giant would make a proposed transfer of certain real property to Conglomerate Corporation. The property in question is unusual because it contains an underground particle collider used for scientific research, but also valuable farmland on the surface, as well as some valuable mineral rights in another part of the parcel. These factors make the property value difficult to assess by reference to the general real-estate market, which means it is difficult for anyone to determine the fairness of the transfer price in the proposed deal. Would it be proper for Attorney Stevenson to facilitate this property transfer at the behest of the president of Conglomerate, if Attorney Stevenson would be representing Giant as the client in this specific matter?

No, not unless the attorney first obtains effective informed consent of the management of Giant Company, as well as that of Conglomerate, because the ownership of Conglomerate and Giant is not identical, and their interests materially differ in the proposed transaction.

Big Law Firm represented Conglomerate Corporation, but the primary lawyer who handled Conglomerate's matters left Big Law Firm, and Conglomerate Corporation followed the lawyer to his new firm for further representation on other matters. Some time later, Giant Company consulted with Big Law Firm about legal representation that would be materially adverse to Conglomerate Corporation. A partner at Big Law Firm accepts Giant's Company's new case. Would it be improper for the partner or other lawyers still working for Big Law Firm to provide representation to Giant Company in a lawsuit against Conglomerate Corporation, if the new matter has no substantial relationship to Conglomerate previous legal matters?

No, prior lawyers' knowledge is not imputed unless the matter is the same or substantially related and another lawyer in the firm has information that is material to the matter

At a press conference about the prosecution of a notoriously vice-prone celebrity, the prosecutor stated that the District Attorney's office had filed charges against the celebrity for shoplifting and drug possession. The prosecutor then said he had no further comments and took no further questions. Was it proper for the prosecutor to disclose such information about the case to reporters?

No, recuasse in a criminal case, there is a presumption of prejudice when a prosecutor states publicly that a defendant is the subject of criminal charges, unless he includes a statement explaining that the charge is merely an accusation and that the defendant still has a presumption of innocence

A certain attorney was a solo practitioner with many years of experience. For the last few years, the attorney represented a local cupcake shop, jointly owned by Susan and Diane. Susan was in a traffic accident while doing a personal errand, but she was driving the delivery van of the cupcake shop. Susan was co-owner of the shop and was therefore free to use the shop's vehicle for occasional person errands. The police who arrived on the scene determined that Susan was not at fault in the accident. The attorney did not do personal injury litigation, so Susan asked him to refer her to a personal injury lawyer who could represent her at trial. At the same time, Susan insisted that the attorney who handled the business transactional work for the cupcake shop should receive a referral fee, and the attorney is willing to accept joint responsibility for the matter but will not assist in the litigation. The attorney has a reasonable belief that the cupcake shop will not become a party to the matter. Could the attorney be subject to discipline for making the referral and accept a referral fee without first obtaining written, informed consent of the cupcake shop, Diane, and Susan for a potential conflict of interest?

No, representation of one client is not directly adverse to another client, and there is not a significant risk that the referral of Susan will be materially limited by attorney's responsibility to the cupcake shop.

An attorney worked for several years for a federal government agency in regulatory enforcement. Big Firm then hired the attorney for a substantially higher salary, and the attorney accepted the position and left his government position. One of the attorney's first assigned cases at Big Firm was a new action by the client against the same government agency for which the attorney had previously worked, defending against an enforcement action that the attorney had initiated while at the agency. The defense will involve challenging the constitutionality of a new regulation that the agency had recently promulgated. While at the agency, the attorney had not been involved with the review and promulgation of any new regulations, including the one at issue in the client's challenge, but instead worked exclusively on enforcement litigation matters. The government agency refuses to consent tot the attorney representing the client, who is the adverse party to the agency, in this matter, and seeks to disqualify Big Firm from representing the client. Is Big Firm subject to disqualification in the client's matter against the attorney's former employer?

No, so long as Big Firm screens the attorney in time from any participation in the matter and provides the agency with prompt written notice about the screening measures in effect.

An attorney is representing a defendant in a highly publicized trial. On his way into the courthouse on the day of jury selection, reporters gather around the attorney hoping for comments. The attorney explains that the prosecutor already held a press conference in which she shared that the defendant had hd refused to take a polygraph test, and that DNA tests had confirmed the defendant's guilt. The attorney explains that polygraph tests are inadmissible due to their unreliability, and that the DNA results are in dispute and will be the subject of expert testimony at trial. He adds that the sleazy prosecutor has a habit of holding such press conferences to prejudice the proceedings before every criminal trial, and that it merely reveals that the prosecutor's cases are too weak to win on the merits without such stunts. His client, he says, is now guilty until proven innocent, which is a shame considering the serious criminal charges in the case. He also mentions that the state's star witness is a dangerous convicted felon who is testifying in exchange for early release from prison. Were the defense attorney's statements proper?

No, such responsive statements should contain only such information as is necessary to mitigate undue prejudice created by the statements made by others.

An attorney represented a defendant facing criminal charges. The client was concerned that his estranged brother would testify against him at trial to impeach the defendant's own credibility if the defendant chose to testify. The rift between them had begun in high school, with a fight over a girlfriend, and had escalated over the years, so that the estranged brother was always ready to list several of the defendant's greatest failures or lapses in character whenever the two interacted. The attorney approached the brother privately and explained that his client was facing serious jail time that would have long-term consequences for the entire family. He then pleaded with the brother not to testify against the client or even talk to the prosecutors about it. The brother found this entreaty moving and agreed to keep quiet. Could the attorney be subject to discipline for this conversation with the brother?

No, the Model Rules do not forbid a lawyer from asking a family member to refrain from giving information to the other party.

An attorney represented the seller in a commercial real estate transaction. During the negotiations over the sale, the only parties present were the attorney, the client (seller), the buyer, and the buyer's lawyer. After the consummation of the purchase, the buyer sought to rescind the sale, alleging that the seller and the attorney had made fraudulent misrepresentations before and at the closing. Each side had completely different versions of what each party said during the negotiations and at the closing. Could the attorney represent the seller in the litigation over rescinding the sale for fraud?

No, the attorney is a material witness for the seller in the upcoming trial.

An attorney represented a client in a divorce case and charged the client an hourly fee for the representation. The client won primary custody of the child from the marriage, and the ex-spouse (the child's other parent) would take the child during school vacations. A year after the case ended, the client wanted to reopen the case to seek additional child support, because in the intervening months, the child had developed a disability that imposed high medical care costs on the client, and at the same time, the ex-spouse had won the Mega-Millions lottery, and was living a luxurious, profligate lifestyle. Would it be permissible for the attorney to represent the client in this matter on a contingent fee basis, given that the divorce was already final?

No, the attorney may not enter into an arrangement for, charge, or collect any fee in a domestic relations matter, the payment or amount of which is contingent upon the securing of a divorce or upon the amount of alimony or support, or property settlement in lieu thereof

An attorney worked as a purchaser for Conglomerate Corporation for many years before law school. After graduating and becoming a licensed practitioner, the attorney opened his own firm and represented many of Conglomerate Corporation's outside vendors in their contractual disputes with Conglomerate. In fact, the attorney advertised every month in local trade journals that he was a former purchaser for Conglomerate Corporation and could provide "affordable and experienced legal representation" to vendors who had legal disputes with corporations like Conglomerate. Regarding fees, the attorney would tell prospective clients that he sometimes billed hourly and sometimes charged a flat fee, depending on the complexity and time demands of each matter, and that this was difficult to predict beforehand. If this uncertainty was acceptable to the client, the attorney would agree to represent the individual. After the representation was complete, the attorney would decide how to bill the client. Is it proper for the attorney to handle fees in this manner?

No, the attorney must inform the client of the basis or rate of the fee and expenses before or within a reasonable time after commencing the representation.

An attorney worked as in-house general counsel for Big Bank. The Federal Trade Commission was holding a series of hearings about the consolidation of the industry and anticompetitive activities, and certain consumer protection groups were advocating in the hearings for regulatory reforms. In one instance, the consumer protection groups persuaded the Commission to subpoena certain corporate from the largest banks to show that they had engaged in undetected predatory pricing and price gouging following natural disasters. After hearing about the request for this subpoena, but before receiving service of it from the Commission, the attorney deleted several computer archives about the company's pricing patterns, and shredded printed records pertaining to the same subject. Big Bank was not currently the target of an enforcement action, and no litigation was pending or immediately contemplated regarding this information. Was it permissible for the attorney to clean up the company archives before receiving a subpoena from the Commission for its public hearings?

No, the attorney obstructed another party's access to evidence and destroyed documents or other material having potential evidentiary value.

An attorney represented an immigrant who was facing deportation. At one point in the proceedings, the immigration judge ordered the attorney to file various documents and forms necessary to the case. The attorney simply ignored the judge's order, knowing that the judge would not close the case and issue a deportation order without these important documents in therecord. The client's deportation was inevitable, given the facts of the case and the relevant law, so the only thing the attorney could do to help the client was delay the deportation for as long as possible. Months passed, and the immigration judge repeatedly re-issued the orders for production of the documents, and the attorney continued to ignore them. Was it permissible for the attorney to hold off on filing the documents that would have hastened the deportation of his client?

No, the attorney's failure to file the papers was not a legitimate litigation strategy to prevent or delay the deportation.

An attorney represented a home builder at the closings of the sales of a few homes that the builder had constructed in a new subdivision. It is not unusual for lawyers who conduct residential real estate closings to encounter issues related to marketability title. Recently, the home builder switched to using another law firm, so the attorney no longer represents the builder. A prospective new client has consulted with the attorney about providing representation. This prospective client is a landowner in an adjacent town, who wants to sue the same home builder, that the attorney formerly represented over a parcel of land on which the home builder plans to construct a few homes. The prospective client claims to have an ownership interest in the property. The contemplated lawsuit would involve whether the landowner or home builder have clear title to the property. Would it be improper for the attorney to represent the new client against the home builder over the marketability of title for this new parcel of land?

No, the attorney's knowledge of marketability of other tracts is not necessarily relevant to litigation involving the marketability of title to the new parcel, so the attorney may represent the new client without informed consent of the home builder.

A litigation attorney represented a client in a lawsuit. The case was still at the pre-trial phase, and the parties had filed cross-motions for summary judgement. While researching the case law to write a responsive brief, the attorney discovered, to her dismay, a new decision from highest court in a sister jurisdiction that is directly adverse to her position in the case. In the attorney's own jurisdiction, the issue presents a case of first impression. The briefs from opposing counsel never mentioned this new decision, presumably because the other lawyer had not yet seen it. Is it improper for the attorney to keep this information confidential, and not disclose the unfavorable authority to the court?

No, the case is not controlling authority in that jurisdiction.

A litigation attorney normally represented clients at trial or in binding arbitration, but in some instances, she will represent a client in a mediation. In one mediation, the attorney knowingly made untrue statements of fact to the other party and opposing counsel. Has the attorney violated her ethical duty of candor to the tribunal, as delineated in Model Rule 3.3?

No, the duty of candor in Model Rule 3.3 is inapplicable to mediation; nevertheless, other rules such as Rule 4.1 may apply to the lawyer's untruthfulness here.

An attorney works for a firm. She also describes herself as an outspoken advocate for the rights of unborn children, that is, she passionately favors legal restrictions on abortion. A local abortion clinic asks the firm to represent it in litigation over recent zoning measures that would significantly limit its hours of operation and therefore the number of clients the clinic could accept. The firm agrees to the representation. The attorney firmly refuses to have any part in the representation, and though no formal screening measures are in place, everyone else in the firm avoids discussing the case with her or around her because they are afraid of receiving another lecture about the wrongfulness of abortion. Early in the litigation, the judge considers disqualifying the firm because it employs the attorney, who has a reputation in the community for her advocacy against legalized abortion. Neither the clinic nor the opposing party (the municipal zoning authority) provided written consent to a conflict of interest. Should the firm be subject to disqualification in this case.

No, the firm should not be disqualified where one lawyer in a firm could not effecting represent a given client because of strong political beliefs, but that lawyer will do no work on the case and the personal beliefs of the lawyer will not materially limit the representation by others in the firm.

An attorney grew up in poverty but worked hard to overcome obstacles and achieve success. Now a successful practitioner, the attorney is idealistic and passionate about helping the less fortunate. Every Saturday morning, he uses a small conference room at the local YMCA to assist pro se litigants in divorce and custody matters - the attorney helps them complete their own court forms (court filings) for a nominal fee, gives some advice about their individual situation, and reviews forms they have completed before the individuals themselves file them. The attorney is concerned about these pro se litigants misunderstanding his role and believing he is their lawyer, so the attorney requires each one to sign a printed disclaimer declaring that no attorney-client relationship exists. It reads, in relevant part, "I understand that this attorney has no legal or ethical obligation to provide legal representation to me in this matter." Given that the pro se litigant signed a form acknowledging that no legal representation will follow, is the attorney correct in believing that no lawyer-client relationship exists in these circumstances?

No, the lawyer is reviewing court documents and providing legal advice about pending legal proceedings, which constitutes the practice of law by the lawyer, even if the representation has a limited scope.

Attorney Stevenson was willing to represent anyone, and rarely turned clients away. In fact, Attorney Stevenson would push the permissible limits under the conflicts of interest rules. At one point, Stevenson helped a construction company obtain the necessary permits from federal, state, and municipal agencies for constructing a new shopping center in an affluent suburban area. Obtaining the permits was not difficult - in fact, Attorney Stevenson found this kind of legal work boring. Before the construction was complete, another company acquired the property and the building project, and brought the construction to completion. Seventeen months after the building was open for tenants, one of the tenants missed to pay rent for his unit for three consecutive months, and the property manager started an eviction process. The tenant hired the same attorney to represent her in the eviction proceedings. The shopping center's owner filed a motion to have the attorney disqualified due to the substantial relationship between his previous work in securing construction permits for the building and the present eviction action against the tenant. Should Attorney Stevenson's previous work for the construction company disqualify him from representing tenant in the eviction proceedings?

No, the matters are not related enough, because they do not involve the same transaction or legal dispute, and any confidential information learned while obtaining the construction permits prior would be unimportant for the nonpayment of rent by a tenant sometime later.

A plaintiff who had prevailed at trial needed representation for the appeal, because the defendant in the case appealed the verdict. Plaintiff's counsel did only trial work, not appellate work, and referred the client to an appellate attorney nearby. The trial lawyer even accompanied the plaintiff to the initial consultation with the appellate attorney to help facilitate the transition and to safeguard his client's interests in retaining new counsel. Theappellate attorney asked the plaintiff to sign an agreement waiving potential malpractice claims against the appellate attorney, because the plaintiff had not yet terminated the representation with her trial lawyer, and the appellate attorney did not want to be responsible for the trial lawyer's mistakes. The appellate attorney did not inform the plaintiff fully about the risks or downsides of waiving future malpractice claims, nor did he advise the plaintiff of the desirability of seeking the advice of independent legal counsel in connection therewith. Could the attorney be subject to discipline, based on these facts?

No, the plaintiff already had independent legal counsel in connection to the malpractice waiver.

A group of several individuals seeking to form a joint venture asked an attorney to represent them in drafting the necessary documents and making the necessary filings with government agencies. Two of the individuals were to provide most of the initial funds for the startup; two others were experienced inventors who were to provide new product designs; two others had expertise in business management and were to serve as managers; and two had proven records in high-end sales and marketing. They have not yet resolved the allocation of ownership shares, bonuses for managers, whether to have anti-compete agreements for each participant, whether patents will belong solely to the joint venture or partly to the inventors themselves, and whether sales reps will work on salary or commissions. Everyone says that she wants whatever terms would be best for the joint venture overall, rather than what would be most beneficial for each one individually. The shared objectives and goals of the group lead the attorney to conclude that no conflicts of interest are present and that it would be counterproductive to try to convince each member of the group to sign an informed consent form acknowledging that conflicts of interest exist, and that the attorney may still represent everyone at once. May the attorney trust his professional judgment and proceed without obtaining separate consent forms from each person in the joint venture?

No, the situation is likely to limit materially the attorney's ability to recommend or advocate all potential positions that each might take because of his duty of loyalty to the others; representing the group's overall interests in effect forecloses alternatives that would otherwise be available to the client.

An attorney provided lobbying services as part of his practice. He represented Giant Gas Company for a few years lobbying on environmental issues related to fracking. During the representation on a certain matter, the attorney learned the basis for Giant's fracking decisions in terms of location, timing, and methodology. Conglomerate Corporation, a major purchaser of the natural gas produced from fracking, has now asked the same attorney to represent it in an antitrust lawsuit against Giant Gas, alleging a conspiracy to impose limits on production. Conglomerate's claims against Giant Gas are likely to include addressing the same production decisions that the attorney learned about in his representation of the latter. This confidential information would certainly advance Conglomerate's position in the anticipated antitrust matter. The attorney no longer represents Giant Gas, because the company was not timely in paying the attorney's fees for the lobbying work. Would it be proper for the attorney to represent Conglomerate in this matter against Giant Gas?

No, there is a substantial relationship between the matters, so the attorney may not represent Conglomerate in the matter without effective consent from both Conglomerate and Giant Gas.

An attorney worked at Big Firm, which a court disqualified from representing a client in a case because one of the other lawyers at the firm had a conflict of interest regarding a former client, and this conflict was imputable to the entire firm. The firm was not timely in implementing screening measures and became subject to disqualification. The attorney was at the firm during this time but was not involved in the matter and did not learn any confidential information about the client. Eventually, the attorney left that firm and went to work at another firm. It turned out that the attorney's new firm is representing the client instead - the client hired the new firm after the previous firm was subject to disqualification. The new firm has no measures in place to screen the attorney from participation in the matter, though the attorney is not in fact participating in the representation. Will the new firm be subject to disqualification now, because the attorney joined the firm from another firm that was subject to disqualification?

No, there is no doctrine of double- imputation that would impute a purely imputed conflict from the attorney onto the other lawyers in the new firm.

A grand jury indicted a defendant on a multiple-count assault and robbery of a woman, a violent mugging in which the perpetrator stole the woman's purse. The victim did not know her assailant, but afterward she identified the defendant in a photo array and then picked him out of a line-up. A bystander made the same identification from a photo array and a subsequent lineup. At the same time, the police informed the prosecutor that two other eyewitnesses viewed the same line-up, but those witnesses stated that they did not see the perpetrator. Moreover, a confidential informant attributed the assault to someone else. Concerned, the prosecutor interviewed the other two eyewitnesses, but he decided that they did not get a good enough look at the perpetrator to testify reliably. The prosecutor also interviewed the confidential informant, but he learned that the informant had previous convictions for fraud, and therefore was not credible. Given the early state of the proceedings, the prosecutor decides that if the case goes to trial, he will inform defense counsel about the other witnesses, because defense counsel may want to call them to testify. On the other hand, it seems unnecessary to mention the other witnesses during the plea-bargaining negotiations, because they are not part of the evidence the prosecutor would use in the case. Has the prosecutor acted within the parameters of the Model Rules?

No, to allow the defendant to make a well- advised plea at the time of arraignment, prior to a guilty plea, the prosecutor must disclose known evidence and information that would be relevant or useful to establishing a defense or negating the prosecution's proof.

An attorney was a partner at Big Firm, which represented Conglomerate Corporation and Giant Company in corporate merger negotiations. Big Firm had state-of-the-art network firewalls, virus protection, password protection, and other data security features in place. Nevertheless, one Friday evening some hackers managed to breach Big Firm's networks and access client information and partner emails, for purposes of engaging in insider trading. The firm detected the breach within a few hours and notified state and federal law enforcement. The stock exchange had closed for the weekend, and law enforcement managed to apprehend the hackers over the weekend, before they had a chance to review the stolen information and share useful data or engage in illegal stock trades. The clients suffered no losses or adverse effects, but they could have. Big Firm is worried about how news of the breach would affect their reputation, and that it might invite other hackers to target their firm, so they would prefer to keep the incident a secret. The partners at Big Firm claim they have no duty to disclose to its clients that the breach occurred, given that no harm resulted. Are they correct?

No, when a data breach occurs involving, or having a substantial likelihood of involving, material client confidential information a lawyer has a duty to notify the client of the breach.

An attorney has a successful blog about legal practice, and the blog generates substantial side income for the attorney. The attorney posts entertaining stories about his clients that attract the attention of his readers and make the blog successful and lucrative. He does not obtain client consent for these posts, but he is careful 1) not to post anything that would seriously injure the client's reputation or legal interests, and 2) not to post information about individuals that is truly confidential, that is not part of the public record. On the other hand, he does post about his personal observations and opinions of clients and their lifestyles, and often shares generalizations based on confidential information of former clients, such as: "On three occasions I've had clients who lived a double life, maintaining separate families in separate cities, and their families never knew." Another post recounted, "Last year I had a client who admitted after the case ended that he had been sleeping with one of the jurors." Apart from potential violations of Rule 1.6 (client confidentiality), which of the following is true?

The attorney has a common-law fiduciary duty not to profit from using client information even if the use complies with the lawyer's ethical obligations, without accounting to the client for any profits made.

A prospective client met with an attorney to discuss a physical assault by her estranged husband. The estranged husband had broken into the hime of the friend with whom the prospective client was staying. During this break-in, the estranged husband had assaulted her, leaving minor injuries. The attorney agreed to represent her in seeking a restraining order against her estranged husband. The next day, upon hearing about this meeting, the estranged husband appearing in the attorney's office, also seeking representation in the same matter. The husband expressed a desire for reconciliation and asked the attorney to represent him at the restraining order hearing. The attorney agreed to represent him, just as he had done with the estranged wife. Both the husband and wife were aware that the same attorney represented them, but neither signed paper work actually consenting to the joint representation. The wife was unhappy that her estranged husband had hired her attorney, but she felt she could not do anything about it. What sanction could the attorney face because of this dual representation?

The attorney is subject both to discipline and to disqualification for violating the model rules, as the parties are directly adverse in the same proceeding.

A certain defendant was facing charges for assault with a deadly weapon. A local criminal defense attorney offered to represent this defendant on a contingent fee basis. In other words, the attorney would charge no fee (the client would pay nothing) if the case resulted in a conviction, but he would pay only if the lawyer won an acquittal. Having no funds on hand to hire a lawyer by any other means, the client was eager to do this and consented to the arrangement, in writing. Which of the following best describes the lawyer's situation?

The attorney is subject to discipline for charging a contingent fee in a criminal matter.

An attorney undertook the representation of a client in a breach of contract claim and began working on the matter. A few weeks later, the opposing party in the litigation consulted with another lawyer in the attorney's firm about the same matter, but during the consultation, disclosed no confidential information except the identity of the other party and the nature of the claim. The other lawyer did a routine conflict check, quickly discovered the conflict with this new potential client, and immediately declined to represent the party. The lawyer and the attorney already representing the first client discussed the situation. Would it be proper for the attorney to disclose to his client that the opposing party had come in for a consultation with another lawyer in his firm?

The attorney may not disclose to the client that the opposing party consulted with another lawyer in the firm but may continue to represent the client if the attorney does not use any information gleaned from the other party's consultation against the other party.

An attorney worked at Big Firm for five years, and she specialized in real estate. During the last three years, the attorney has worked mostly on Conglomerate Corporation's properties and disputes that arose related to them. Shortly before leaving big firm, the attorney handled a matter for Conglomerate Corporation related to the construction of a new apartment complex. The attorney now works at Boutique Firm, and a prospective client explains during an initial consultation that she needs to sue Conglomerate Corporation over a disputed right of way through the very property that holds the new apartment complex--one of the last matters that she handled for conglomerate. Conglomerate is still using Big Firm for its legal representation. Would it be proper for the attorney to represent the prospective client in this matter against Conglomerate Corporation?

The attorney may represent the new prospective client if Conglomerate Corporation consents in writing to her doing so.

An attorney serves as the lawyer for a corporation and is a member of its board of directors. Which of the following is true regarding this situation?

The attorney must advise the other board members that in some circumstances, matters they discuss at board meetings while the attorney is there as a fellow director would not be protected by the attorney-client privilege in later litigation; and that conflict of interest considerations might require the attorney's recusal as a director, or might require the attorney to decline representation of the corporation in a matter.

A college sophomore is facing criminal charges related to a drunk driving accident the previous night. To his great relief, the student's parents hire an attorney as defense counsel for their song, agreeing to cover all fees and expenses. The criminal charges in this case allow for jail time or hefty fines as potential punishments. The prosecution offers a plea bargain--a $2,000 fine, a few hours of community service, and six months' probation, but no jail time. The student, who is the defendant, is thrilled, but his his parents insist that the attorney decline the plea bargain and go to trial, which the attorney believes will result in a conviction and a short jail sentence. The parents believe a short jail term will be good for their son and teach him a lesson, and paying the $2,000 fine on top of the attorney's fees will force them to forego their vacation plans that year. A loud argument ensues in the conference room between the client and his parents over the best course of action. How should the attorney proceed?

The attorney must follow the decision of the defendant, who is his real client, and accept the plea; he should have explained to the parents in advance that they could not control the case even if they paid his fees

An attorney had to abandon his home and his vehicle to take refuge in a FEMA rescue shelter following a natural disaster in his area. Some of the attorney's clients required immediate legal services that the attorney was unable to provide. What would be the attorney's ethical duty in this situation?

The attorney must withdraw from representing the clients mentioned.

An attorney, a venture capitalist, and a land developer agreed to form a corporation to develop a new shopping mall. Their agreement allocates ownership shares based on the appraised value of the venture capitalist's land, which he is contributing for this enterprise, the market value of the developer's design and construction work, and the attorney's regular fees for the hours contributed to the formation and ongoing representation as corporate counsel. The attorney was already representing both the venture capitalist and the developer as his clients in unrelated matters. Which of the following is NOT a duty of the attorney in this situation, if the attorney performs the others?

The attorney must withdraw from representing the venture capitalist and the developer on the other matters, at least until the process of forming the corporation is complete, to avoid conflicts of interest.

An attorney filed a lawsuit on behalf of a client against Conglomerate Corporation as the defendant. The attorney's contingent fee contract stipulated that the attorney would receive thirty percent of recovery, if the case settled before trial, and a higher percentage if a trial was necessary. The client and the attorney signed an engagement contract for the provision of legal representation, which stipulated these terms. The document that the client signed clearly explained the percentage that should accrue to the attorney the event of settlement, trial, or appeal; litigation and other expenses that the attorney would deduct from the recovery; and that such deductions would come out of the total before the calculation of the contingent fee. The letter also discussed all potential expenses for which the client could be liable, if the client prevailed in the case or not. While the case was still in the discovery phase, Conglomerate Corporation offered the client a structured settlement. Under the settlement terms, Conglomerate would pay the client one million dollars up front, which would cover the plaintiff's medical costs, and the defendant would also purchase an annuity for the client. The annuity would cost Conglomerate $153,000, and it would guarantee the client monthly disbursements of $1000 until the client's death. The client is thirty years old. In terms of fees, how much should the attorney receive?

The attorney should receive $300,000 when Conglomerate's million-dollar lump sum payment arrives, and $300 of each subsequent disbursement from the annuity, when the disbursements occur, until the client's death.

A client owns a partnership share of a closely held business, and the other partners vote to impose an involuntary buy-out of the client to remove him from the firm. The client is clearly upset about this, but the partnership agreement clearly permits involuntary buyouts by a majority vote of the other shareholders. Then the client hires an attorney to represent him in the buyout transaction, to review the necessary documents and provide legal counsel about it. No litigation is under consideration yet. The attorney's sister is also a lawyer in that city, at another firm, and the sister represents the other shareholders in the partnership. Nevertheless, the attorney did not disclose that her sister represented the other partners, as she and her sister are not close and rarely speak, and the matter is unlikely to turn into litigation. Is the attorney, or the other lawyers in her firm, subject to disqualification in this matter?

The attorney would be subject to disqualification, but ordinarily the other lawyers in her firm would not be subject to disqualification.

An experienced attorney practiced at a small firm in a rural area. The attorney regularly represented the county school district in employment discrimination matters. One day, a group of citizens asked the attorney to represent them before the county planning commission to oppose the widening of a county road. The school district had separate budgetary funding, and it had an elected governing Board with its own authority to hire legal counsel. In contrast, the members of the county planning commission were appointees by the County Executive, and lawyers at the County Solicitor's office handled the legal work for the commission, though the commission and the County Solicitor's office received their funding from separate line items in the county budget. Would it be proper, under these facts, for the attorney to agree to represent the citizens against the Commission, without informing them of her existing relationship with the School District, and without also securing the Board's consent?

The attorney would have no obligation under the ethical rules to inform the citizens group about her representation of the school district, or the school district about her representation of the citizens group against the county planning commission in the road- widening dispute.

A doctor was facing criminal charges for an illegal kickback scheme - accepting bribes to refer patients to a certain hospital. The attorney representing the doctor in the criminal matter previously represented the hospital, and he had drafted one of the contractual agreements between the doctor and the hospital that federal prosecutor now allege to have been a sham agreement (payment for services never rendered). The attorney also provided some legal advice several years ago to another doctor, in one passing conversation, and that doctor now turns out to be part of the same kickback scheme.This other doctor, in fact, has turned state's witness in the case against the attorney's current client. The federal prosecutors have filed a motion to disqualify the attorney from the case because he is a potential witness about the agreement between the doctor. On the other hand, it has not yet listed him as a witness who will testify at trial, and it does not appear his testimony would be necessary to prove any of the elements in the case, given the number of other witnesses and documentary evidence available. How should the court rule on the motion to disqualify?

The court should deny it because the government has not met its burden of showing that the attorney would be a necessary witness in the case, or that he possessed confidential information about the other doctor who will serve as a hostile witness in the case.

An Assistant District Attorney, who has recently joined a county prosecutor's office, represented a defendant at a preliminary hearing in a pending criminal case while in private practice. Now that this attorney has joined the prosecutor's office, how can the office proceed with the prosecution of the same defendant?

The office must either hire a special prosecutor for the case, borrow a prosecutor from a neighboring jurisdiction, or implement effective screening measures to exclude the new attorney from the prosecution.

An attorney served for several years as an appellate court judge. At one point, the judge was on a panel that affirmed two trial orders in an ancillary probate proceeding. Soon thereafter, the attorney left the appellate court and returned to private practice at Boutique Firm. The larger probate matter was still dragging on, and relators brought a mandamus appeal arising out of the same ancillary proceeding and hired Boutique Firm to represent them on the appeal. This necessitated filing a motion to substitute counsel from a previous firm that had provided representation up to that point. Opposing counsel did not oppose the motion, as they did not know Boutique Firm had hired a former appellate judge who had signed earlier orders in the case. Boutique Firm did not screen the former judge from the matter. When opposing counsel eventually realized this fact, the lawyer immediately filed a motion to disqualify Boutique Firm from the appeal. Boutique Firm responded that opposing counsel had already consented to the potential conflict when it did not oppose the motion to substitute counsel; moreover, there was no demonstrated prejudice to the opposing party. How should the court rule on the disqualification motion?

The court should grant the motion as the matters related to each other, and the moving party did not have adequate notice about the conflict to give informed consent.

A potential client sought representation from an attorney in a legal dispute over the inheritance rights in an estate matter. The attorney was indecisive, because the estate was extremely complicated, so he met with the client several times over the next few months, trying to understand the intricacies of the will, the trusts involved, and the rival heirs. The potential client provided extensive confidential information about the estate to the attorney in meetings, phone calls, and emails. Eventually, however, the attorney declined the representation. During this time, the attorney had been in negotiations with another lawyer about forming a new law firm together. The other lawyer, unfortunately, was representing the rival heir, that is, the opposing party in the same estate matter. When the attorney and the other lawyer formed their new firm, the heir who had been the potential client then sought to have the attorney's new firm disqualified from the estate matter, arguing for imputation of the attorney's knowledge of confidential information to the other lawyer, who was the heir's opposing counsel in the case. The other lawyer, who was now partners with the first attorney, argued that no client-lawyer relationship had existed between the heir and the attorney, because the attorney had declined the representation at the end. The heir who had been the prospective client insisted that the attorney had received confidential information from her, and that he had disclosed it to the other lawyer, who represented the rival heir in the matter. As a factual matter, the judge ruled that the prospective client-heir had introduced substantial evidence that she had provided extensive confidential information to the attorney in the process of seeking representation from him; conversely, the judge was surprised that the attorney had almost no evidence to show that he had not disclosed any confidential information to his new partner. How should the court rule on the motion to disqualify both lawyers?

The court should grant the motion, because lawyers have some ongoing duties of confidentiality toward prospective clients, even after declining the representation, and the other lawyer has a conflict of interest by imputation.

An attorney worked as a prosecutor in a local district attorney's office. A month before leaving there to go into private practice, she briefly worked on a case in which applied for the search warrants for the police to try to locate a fugitive suspect. When the police apprehended the fugitive a few weeks later, another prosecutor filed the charges and proceeded with the case. Eventually, the attorney who had left to start her own practice received a referral client who turned out to be the same defendant. When she filed an appearance to represent the defendant, however, the prosecutor filed a motion to have her disqualified, because she had worked on the same case by applying for the warrants. The attorney responded that the defendant was not even in custody yet when she applied for the warrants, that the warrant application was a purely administrative chore, and that the filing of the charges did not occur until after she left her position there. How is the court likely to rule?

The court will disqualify the attorney from serving as defense counsel because she had participated in the matter personally and in a substantial way as a prosecutor.

A certain client hired an attorney to represent her at trial. After voir dire, the attorney wanted to learn as much as possible about each of the jurors, such as their views on political and social issues that might be relevant to issues in the case, so the attorney found each juror's social media accounts and reviewed their postings and comments. Some of the social media platforms notify the account holder whenever someone views their profile, so jurors with these social media accounts received notifications that the attorney had visited their profile page and reviewed items there. Which of the following is true, given this scenario?

The fact that a juror or a potential juror may become aware that the lawyer is reviewing his Internet presence when a social media network setting notifies the juror of such review does not constitute a communication from the lawyer in violation of Rule 3.5(b).

A prospective client met with an attorney at Boutique Firm for an initial consultation about a personal injury lawsuit over injuries the prospective client had sustained. The attorney declined the representation because he thought the client's case was unwinnable and would therefore generate no fees. During the consultation, the attorney asked some probing questions about the incident, and the client admitted facts indicating an unreasonable assumption of foreseeable risks beforehand, as well as the client's own intoxication at the time, which in the case would constitute contributory negligence. Furthermore, the client had failed to take obvious measures afterward to mitigate the damages. The attorney was certain that all these unfavorable facts would come out during discovery, and the client's claim would become laughable at trial. Two months later, another client came in for a consultation with another lawyer at Boutique Firm. This prospective client had was service of process in a new personal injury lawsuit, and he was the named defendant. The plaintiff in the lawsuit was the same individual who had met with the first attorney for a consultation a few weeks before. Boutique firm agreed to take the case and represent the defendant in the litigation. Which of the following is true, according to the MRPC?

The other lawyer at Boutique Firm can represent the defendant in the matter if the first attorney has not disclosed any confidential information to others in the firm, and the firm carefully screens the attorney completely from the matter and provides written notice to the other party.

A prosecutor obtained an indictment from a grand jury against a defendant on a multiple- count assault and robbery of a woman, that is, a violent mugging in which the perpetrator stole the woman's purse. The victim did not know her assailant, but afterward she identified the defendant in a photo array and then picked him out of a line-up. A bystander made the same identification from a photo array and a subsequent lineup. At the same time, the police informed the prosecutor that two other eyewitnesses viewed the same line-up, but those witnesses stated that they did not see the perpetrator. Moreover, a confidential informant attributed the assault to someone else. Concerned, the prosecutor interviewed the other two eyewitnesses, but he decided that they did not get a good enough look at the perpetrator to testify reliably. The prosecutor also interviewed the confidential informant, but he learned that the informant had previous convictions for fraud, and therefore was not credible. Does Rule 3.8(d) require the prosecutor to disclose to defense counsel that two bystanders failed to identify the defendant and that an informant implicated someone other than the defendant?

The prosecutor would have to disclose that two eyewitnesses failed to identify the defendant as the assailant and that an informant attributed the assault to someone else, because the prosecutor knew that information from communications with the police.

A judge lost his temper with an attorney and spoke very abusively to him in open court, in front of a jury, using profanity and calling the attorney "an embarrassment to the profession and a menace to his own clients." Defensively, the attorney shot back that the judge was completely out of line, that the judge should have retired years ago; the attorney also made a mildly obscene gesture at the judge. Eventually, both calmed down and apologized to each other profusely. Opposing counsel reported the attorney to the state bar disciplinary authority, but did not report the judge, before whom opposing counsel appears regularly. Could the attorney be subject to discipline?

Yes, because a lawyer may stand firm against abuse by a judge but should avoid reciprocation; the judge's default is no justification for similar dereliction by an advocate.

Attorney Stevenson did not know anything about the construction industry, but he thought he knew how to draft contracts. Giant Equipment Corporation manufactures cranes, bulldozers, and large backhoes used for building construction. These machines are pricey. Twenty-seven months ago, the manufacturer hired Attorney Stevenson to help with drafting Purchase and Sale Contracts for the manufacturer to use for all these items of heavy equipment. Attorney Stevenson advised the company on what provisions to include and some of the exact wording they should use in the Purchase and Sale Agreements. Then the representation ended, and the company has not contacted an attorney since. Last week, Ashby Building Construction retained Attorney Stevenson to handle a dispute with a manufacturer of one of its construction cranes. It soon becomes apparent learns that the piece of equipment came from Giant Equipment Corporation, and that the procurement officer for Ashby consummated the purchase by signing one of the contacts on which Attorney Stevenson had advised Giant. Now Ashby wants to rescind the contract and return the machine for a full or partial refund, because it used the crane for a week before it broke down. Would Attorney Stevenson be subject to disqualification in a such a latter, if litigation ensued?

Yes, Attorney Stevenson cannot seek to rescind on behalf of a new client a contract drafted on behalf of the former client.

A client hired an attorney to represent her in a litigation matter. At the end of the first day of trial, the client is unhappy with her lawyer's performance in the courtroom and informs the attorney that she is firing him and will find another lawyer. The attorney wants to continue representing this client until the end of the trial. May the client discharge the attorney after a trial has begun?

Yes, a client has a right to discharge a lawyer at any time, with or without cause, subject to liability for payment for the lawyer's services

Big Bank hired Big Firm to represent it in a matter against Developer. Big Firm's partners explained to Big Bank before commencing the representation that they had hired an associate who previously worked for the firm that was representing Developer, and that he had worked on various matters for Developer while there. None of the associate's work was on the same case that was now pending, but it was unclear whether some of the matters had overlapping factual or legal issues with the present matter. Big Firm gives consent to the representation despite the conflict of interest, but it conditioned its consent on Big Firm implementing strict measures to screen the associate from any participation in the matter - including relocating the associate to another office at the firm, where he would not have day to day contact with lawyers representing Big Bank. The partners agreed, but it took a few months for them to free up office space to move the associate out of his current office, which was the office between the two lawyers serving as lead counsel in Big Bank's matter. Otherwise, the firm followed the standard screening procedures delineated in Model Rule 1.10. If a disciplinary action or legal malpractice action arose later regarding the representation of Big Bank, would the associate's conflict of interest be imputable to the other lawyers at Big Firm?

Yes, a client's informed consent to a conflict can be qualified or conditional, as here, and Big Firm violated the client's condition, so it did not have valid consent to the conflict.

Two separate clients hired the same attorney, signing their retainer agreements one week apart, on unrelated matters, though both involve property owners' rights under the state's common law doctrine of public trust for beaches, which guarantees public access to beaches up to the vegetation line on the shore. In one case, erosion has moved the boundary back on the property owner's lot to the point where his house is now clearly on the public access portion, and he seeks a declaratory judgment that erosion cannot jeopardize the private ownership of a building and its curtilage. Current public trust doctrine in the state would suggest that the property owner has lost all the value in his property, so he needs to seek a change or exception to the current law. The other case involves a property owner whose lots had always been separated from the beach by a small public park, but erosion has eliminated the park and given him water access from his property, which has doubled the value of his land under current public trust doctrine. The state government, however, is seeking a declaratory judgment in his case, arguing for an exception or change to the current law that would rob the owner of the windfall he received due to the erosion. Does this situation present a conflict of interest that would require the attorney to obtain informed consent, in writing, from both clients, before proceeding with the representation?

Yes, a conflict of interest exists if there is a significant risk that a lawyer's action on behalf of one client will materially limit the lawyer's effectiveness in representing another client in a different case, as when a decision favoring one client will create a precedent likely to seriously weaken the position taken on behalf of the other client.

Client hired an attorney to represent her federal court litigation, defending against antitrust enforcement actions by the Federal Trade Commission and the Department of Justice. The attorney adopts a "quagmire" strategy, burying the government lawyers in several dozen motions to limit or compel discovery, to compel admissions or stipulations, to limit the admissibility of certain evidence or witness testimony. The strategy also includes maximizing the number of depositions and repeated requests for extensions of time and postponements of proceedings. On a few occasions, the attorney even re-filed a motion after the court ruled on the motion in the government's favor, merely to make the government lawyer spend the time filing objections or replies based on the court's previous ruling on the same issue. The government lawyers filed a complaint against the attorney with the state bar authorities, but the state disciplinary authority decided not to pursue the matter, in part because it was in federal court and involved exclusively federal issues. Could the attorney also face sanctions or penalties under federal law, if the state bar rejected the complaint?

Yes, a federal statute authorizes federal courts to require a lawyer to pay all the excess costs, expenses, and legal fees incurred because of the lawyer "unreasonably and vexatiously" multiplying the proceedings.

An attorney was representing a client in a criminal matter. At the bail hearing, the prosecutor told the court that the defendant was a flight risk, and he asked the court either to confine the defendant until trial or to set bail at $15,000. When it was the attorney's turn to speak, he assured the judge that the client had a medical condition that would prevent him from leaving the area, and that the client did not intend to flee the jurisdiction, but was confident that he could and trial and clear his name of all charges. The attorney knew, however, that the client already had plane tickets to Venezuela, a non-extradition country, and that the client had already fully recovered from his serious medical condition. Is the attorney subject to discipline for making these statements to the court?

Yes, because a lawyer may not knowingly make a false statement of fact or law to a tribunal

An attorney represented Conglomerate Corporation in the company's regulatory compliance work. The Environmental Protection Agency (EPA) proposed new pollution emission regulations through notice-and-comment rulemaking procedures as proscribed under the Administrative Procedure Act. The proposed new regulations would impose burdensome financial costs on Conglomerate Corporation. During the public comment period, the attorney submitted comments arguing that the proposed rules made only marginal improvements to public health but imposed devastating costs on the regulated industry, which would violate the "feasibility" requirement in the relevant governing statute. The attorney relied entirely on published scientific studies to argue that the health benefits were minimal, but knowingly exaggerated how much it would cost his client to comply with the proposed standards. Any member of the public could submit comments during the comment period; there were no public hearings and none of the submissions to the agency were under oath. Could the attorney be subject to discipline for his conduct regarding the submission of comments to the agency?

Yes, a lawyer cannot submit false statements or comments to a regulatory agency functioning in its rulemaking capacity.

A certain attorney obtained a successful outcome in a client's matter, and the client was grateful. The client sent the attorney a gift basket that year as a holiday gift, containing high-quality fresh fruit, sample-size jars of gourmet fruit preserves, and a few other delicacies. The gift basket cost the client $50. Is it proper for the attorney to accept this gift, or must the attorney refuse it?

Yes, a lawyer may accept a simple gift such as a present given at a holiday or as a token of appreciation.

An attorney represented a client in a criminal matter. The client had a history of mental illness, and the court ordered a psychological examination to determine if the client would be competent to stand trial. The case did not involve an insanity defense or a defense of diminished capacity. The psychologist who evaluated the client spoke privately to the attorney and explained that the client was indeed competent to stand trial, but that in his opinion, the client also suffered from delusional narcissism, paranoia, and oppositional-defiant syndrome. The psychologist pleaded with the attorney not to tell the client about this diagnosis, because the disclosure could harm the client, triggering an episode of paranoia in which the client would suspect that everyone around him was conspiring to institutionalize him, and he would become uncooperative at trial and mistrustful of his own lawyer. Then the attorney told the client that the psychologist had deemed him competent to stand trial and did not disclose the rest of the psychologist's assessment. Was it proper for the attorney to conceal the psychologist's diagnosis from the client?

Yes, a lawyer may delay transmission of information when the client would be likely to react imprudently to an immediate communication, including a psychiatric diagnosis of a client when the examining psychiatrist indicates that disclosure would harm the client.

An attorney had his own firm specializing in small business transactions. The clients were small business owners who did not have in- house counsel or other legal representation. His representation agreements with clients included all necessary disclosures, fee schedules and rates, and a clause stipulating that all potential legal malpractice claims would go through binding arbitration. The attorney would explain this term fully to each client, but he would decline representation for any potential client who would not agree to binding arbitration. The attorney did this in hopes of limiting his future malpractice liability to clients. Was it permissible for the attorney to do this?

Yes, a lawyer may make an agreement with the client to arbitrate legal malpractice claims, provided such agreements are enforceable and the lawyer fully informs the client of the scope and effect of the agreement.

An attorney agreed to represent a plaintiff in a personal injury lawsuit, and the next day agreed to represent a defendant in litigation where the defendant faces vicarious liability. Only after the attorney has conducted some investigation of the case, and has obtained confidential information from each client, does the attorney discover that the plaintiff client is actually suing another of the attorney's clients, under a theory of vicarious liability. The two clients are actually adverse parties in the same litigation. Must the attorney withdraw from representing both clients?

Yes, a lawyer shall withdraw from the representation of a client if the representation will result in violation of the rules of professional conduct.

A certain attorney represents Conglomerate Corporation in a regulatory compliance matter, drafting documents for Conglomerate to file with the Securities and Exchange Commission and the Federal Trade Commission regarding executive salaries (for the SEC) and product market share (for the FTC's antitrust inquiry). Conglomerate Corporation owns or co-owns numerous subsidiaries and affiliates in unrelated industries. This attorney's retainer agreement limits his representation exclusively to the SEC and FTC regulatory matters. Victim hires the attorney to represent him in a personal injury suit against Subsidiary Corporation, partly owned by Conglomerate Corporation, over a slip and fall accident in Subsidiary's parking lot. Is it proper for the attorney to represent Victim in a tort action against an affiliate or subsidiary of his other client, Conglomerate Corporation?

Yes, a lawyer who represents a corporation or other organization does not, by virtue of that representation, necessarily represent any constituent or affiliated organization, such as a parent or subsidiary, and the lawyer for an organization may provide representation adverse to an affiliate in an unrelated matter.

An attorney represented a client in pending litigation that had just begun. A magistrate judge held a preliminary hearing in the matter to settle whether the matter should remain under seal for the time being. A regular trial judge would later conduct the jury trial on the merits. The attorney's testimony was necessary to establish a matter at the preliminary hearing, so another lawyer from his firm represented the client at the hearing and conducted the direct examination of the attorney as a witness. After the attorney has served as a witness at a preliminary matter, could he then represent the client at the trial, the following year?

Yes, a lawyer who testifies before a judicial officer concerning only a preliminary motion may still serve serving as advocate at a subsequent trial before a jury.

An attorney served for several years as a professional mediator. She decided to change careers and become a litigator, and one of the parties from her final mediation sought to retain her as their attorney in a matter closely related to the subject of the litigation. The other party, which already had legal representation, provided written, informed consent to this arrangement. Under such circumstances, would it be permissible for the former mediator to represent a party in the same matter in which the attorney served as mediator?

Yes, as it appears all parties to the proceeding gave informed consent, confirmed in writing.

An attorney represented Small Business Associates while working at Big Firm, her first law firm after law school. When the attorney did not make partner at the firm, she ended her employment there and started her own firm. The attorney took some of her clients with her, including Small Business Associates, whom she continues to represent. Big Firm no longer has Small Business Associates as a client. Big Firm then agrees to represent Conglomerate Corporation in a trademark infringement case against Small Business Associates, the first such case that the latter has ever faced. Can Big Firm represent Conglomerate in a case against its former client, Small Business Associates?

Yes, as long as the matter is not the same or substantially related to that in which the attorney formerly represented the client; and no lawyer remaining in the firm has confidential information about Small Business Associates from when the attorney represented them at that firm.

The Workers' Union at a manufacturing plant is having annual collective bargaining negotiations with the Management. Wages and benefits are not in dispute this year, as the parties reached an agreement in the previous year's collective bargaining about a five-year schedule for wages and benefits that was acceptable to both the Union and Management. The sole issue in dispute this year is about hiring. The Workers' Union wants the plant to hire five or six new assembly line workers so that there will be more efficiency and more flexibility for workers requesting days off or changes in their shifts. The Management wants to hire fewer new workers, potentially two at most, to keep payroll costs down and their stock share prices high. The Union and Management agree to hire a certain attorney, an experienced labor lawyer at an outside firm, to facilitate the collective bargaining negotiations. Neither side is currently expecting a breakdown in bargaining that would lead to litigation. Would it be proper for the attorney to have both the Union and the Management as clients while facilitating the negotiations?

Yes, assuming both clients provide written informed consent, common representation is permissible where the clients' interests mostly align, even though there is some difference in interest among them, so a lawyer may seek an agreement between them on an amicable and mutually advantageous basis

An experienced attorney handles claims against banks for many clients for issues regarding the failure of banks to investigate in a timely manner claims of fraud or unauthorized use of bankcards. Most of the attorney's work consists of sending demand letters, and most cases never actually result in the filing of a suit. Bank, a small local bank, retains the attorney to handle a certain claim against a customer for non-payment of a loan. The attorney has not represented any clients against Bank. Even so, the attorney includes in his contract for services a clause in which Bank waives any conflicts that may arise in the future - conflicts that involve the attorney representing clients against Bank for issues regarding failure to investigate claims of fraud or unauthorized use of bankcards. Is the attorney's conduct proper?

Yes, attorneys can include waiver clauses for specific future conflicts in their contracts, if the clients are aware of the waiver, and if the contract delineates the types of future representations that may arise.

An attorney prepared a contract for a client in 2013. The matter concluded, and the representation regarding that matter has ended, though the attorney continues to represent the same client on some unrelated matters. In 2017, while using that agreement as a template to prepare an agreement for a different client, the attorney discovers a material error in the agreement. On those facts, do the Model Rules require the lawyer to inform the client of the error?

Yes, because Rule 1.4 requires that lawyers disclose material errors made during the representation to current clients, and the attorney's ongoing representation on other matters means a client-lawyer relationship still exists.

An attorney prepared a contract for a client in 2016. The matter is nearing conclusion, so the representation regarding that matter has not ended. In 2018, while using that agreement as a template to prepare an agreement for a different client, the attorney discovers a material error in the agreement. The error does not, however, furnish a colorable claim for malpractice, because the client has not yet suffered any injury, and it is not clear that the attorney's error falls far enough below the industry standards to meet the legal standards for negligence. On those facts, do the Model Rules require the lawyer to inform the client of the error?

Yes, because Rule 1.4 requires that lawyers disclose material errors made during the representation to current clients, even though the error does not furnish the basis for a valid malpractice claim.

An attorney prepared a contract for a client in 2016. The matter is nearing competition, so the representation regarding that matter has not ended. In 2018, while using that agreement as a template to prepare an agreement for a different client, the attorney discovers a material error in the agreement. The error does not, however, furnish a colorable claim for malpractice, because the client did not suffer any injury, and the client in the meantime canceled the agreement with the other party due to other factors. Even so, any reasonable client who learned of this mistake would lose confidence intheir lawyer's competence or diligence. On those facts, do the Model Rules require the lawyer to inform the client of the error?

Yes, because Rule 1.4 requires that lawyers disclose material errors made during the representation to current clients, whenever the error would predictably cause a client to consider terminating the representation even in the absence of harm or prejudice.

Client is an inexperienced drug dealer and consults with his attorney about the legal ramifications of his business. Without explicitly endorsing or encouraging the client in his criminal enterprise, the attorney conducts research at the client's request about various drug laws and sentencing guidelines. The attorney writes a detailed memorandum of law explaining that certain threshold quantities of drugs, according to the relevant statutes, create a presumption of "intent to distribute" or trigger a significant sentencing enhancement. Similarly, the attorney explains that statutes and sentencing guidelines impose higher-grade charges and severe sentencing enhancements if a drug dealer brings a firearm to a transaction. The client mulls over the information and decides to change his business model from bulk sales of narcotics to selling smaller quantities in more individual transactions, such that each sale constitutes only the lowest-level misdemeanor. The client also instructs all his subordinates to avoid carrying firearms and instead to refill pepper spray devices with hydrochloric acid, which they spray in the face of their opponents in any altercation, causing severe disfigurement. Is it proper for the attorney to provide such legal advice to the client?

Yes, because a lawyer may discuss the legal consequences of any proposed course of conduct with a client and may counsel or assist a client to make a good faith effort to determine the validity, scope, meaning or application of the law

Client, age 18, is facing criminal charges of sex with a minor, based on his sexual relationship with his thirteen-year-old girlfriend, who lives in the same tenement building. The relevant statute has strict liability for perpetrators - that is, no mens rea or scienter element - and places the victim's age cutoff for the most serious grade of felony at age 14. It is indisputable in the case that the defendant had a sexual relationship with the victim when she was thirteen, but the victim claims she wanted the relationship and willingly consented to the sexual contact with her boyfriend. A state psychologist examined the victim and included in his report that she was emotionally mature for her age and was making relationship decisions in the same way as an adult. Even though Attorney is certain that the trial court will convict Client, he believes there is a slight chance that he could convince an appellate court to take a loose view of the age-of-consent provision in the statute, either on substantive due process grounds or simply as a matter of progressive statutory construction. Attorney believes that many thirteen-year-olds, and even younger, are sexually active nowadays and that the criminal laws should reflect the changing values of society. Attorney agrees, therefore, to take Client's case and to use it as a test case to try to change the law of sexual consent in the appellate courts. Is it proper for Attorney to make a defense in a criminal case that goes against the clear statutory verbiage and established case precedent?

Yes, because a claim or argument is not frivolous if the lawyer is making a good-faith argument for modification or reversal of existing law

An attorney spent several years working on the state intermediate appellate court as one of its nine justices in a state in which such judges run for election in the general elections every four years. When the attorney ran for re-election, she lost, and needed to return to private practice. The client wants the attorney to represent her in her appeal of a state trial verdict. The case previously came up on appeal before the state intermediate appellate court, but the attorney was not on the panel that decided the case. The state Supreme Court subsequently reversed the decisions of both the appellate court and the trial court and remanded the case for a new trial. The new trial resulted in an unfavorable verdict for the client, so she wants to appeal the case again. Would it be proper for the attorney to represent her in this matter?

Yes, because a judge who was a member of a multimember court, and thereafter left judicial office to practice law, is not prohibited from representing a client in a matter pending in the court, but in which the former judge did not participate

An attorney worked for several years for a federal government agency in regulatory enforcement. The attorney was involved in several enforcement matters against Conglomerate Corporation. Big Firm has always represented Conglomerate Corporation in all its litigation and regulatory compliance matters. The attorney made a good impression on the Big Firm partners when serving as opposing counsel in the same litigation. At the end of a deposition of Conglomerate Corporation's executives during the discovery phase of an enforcement proceeding, Big Firm partners approached the attorney privately and asked if the attorney would be interested in leaving the agency for a position at Big Firm. The attorney explained that they would have to match his current salary at the government agency for him to condor the proposal. Big Firm then scheduled an employment interview with the attorney, at the end of which they offered to double his salary if he left the agency and accepted a position at Big Firm. The attorney decided to postpone deciding until the pending agency enforcement matters against Big Firm's client were complete, in order to avoid the appearance of conflict of interest. The matters dragged on for another year, however, and Big Firm eventually withdrew its offer. Is the attorney subject to discipline?

Yes, because a lawyer currently serving as a public officer or employee shall not negotiate for private employment with any person who is involved as a party or as lawyer for a party in a matter in which the lawyer is participating personally and substantially

At a press conference about the prosecution of an accused serial killer, the prosecutor stated that the police arrested the defendant at the scene of one of the crimes soon after the crime occurred, at 11 pm on Saturday. Was it proper for the prosecutor to disclose such information about the case to reporters?

Yes, because a lawyer in a criminal case may state the fact, time, and place of arrest.

After an attorney has been representing a client in a transactional matter for six months, the client asks the attorney to draft and deliver some documents that the attorney knows are fraudulent. The attorney tries to dissuade the client, but the client insists. The attorney believes the recipient of the documents will probably realize they are fraudulent before irreparable harm happens to the recipient. The client is willing to sign a private document for the attorney in which the client takes full responsibility for the fraud and states that the attorney was merely following orders and is not blameworthy in the matter. Would it be improper for the attorney to acquiesce, and draft and deliver the documents according to the client's instructions?

Yes, because a lawyer is required to avoid assisting the client, for example, by drafting or delivering documents that the lawyer knows are fraudulent

An attorney is a litigator and finds it helpful to talk to jurors after a trial concludes to see what they thought about the performance of the lawyers in the case. Assuming the judge has not forbidden talking to jurors and the jurors are willing to talk to him, is it proper for the attorney to have conversations with jurors in their homes, a week after the trial?

Yes, because a lawyer may communicate with a juror after the discharge of the jury and must respect the desire of the juror not to talk with the lawyer

An attorney provides itemized billing to her clients: hours worked by partners and associates, expert fees, international phone call charges, court costs, stenographers used in depositions, and so forth. She also includes some itemized prorated charges for overhead costs. Her mobile phone, which she uses exclusively for work, has a plan with a fixed monthly charge and unlimited minutes and data, so she divides her monthly phone bill into hourly increments for each day of the month, and for each hour of time she works on a client's matter, she bills the client for an hourly increment of her phone bill, even if she did not use the phone during that hour. She reasons that she was paying to have a phone available during that time in case clients needed to reach her, so the clients can share the costs. She takes a similar approach with other fixed overhead costs, like the salaries of her support staff - each client bill has a ten-dollar charge for "general staffing costs." A nominal charge on each bill is for the administrative costs of billing clients. Could the attorney be subject to discipline for charging clients a share of her overhead costs and operating expenses?

Yes, because a lawyer may not charge a client for overhead expenses normally associated with properly maintaining, staffing, and equipping an office.

During the discovery phase of business litigation, Conglomerate Corporation receives a discovery request asking for "all documents, memoranda, emails, or other internal correspondence related to the transaction that is the subject of this dispute." A certain attorney represents Conglomerate Corporation. Thousands of documents stored in electronic format on Conglomerate's computers and servers would potentially fall under this request for production. Then the attorney proposes to opposing counsel that they produce the requested documents in electronic form on a set of compact discs, and the opposing counsel readily agrees. Long before the litigation began, the attorney began using software to scrub the metadata from documents - electronically embedded information about the name of the user whose computer created the document, the date and time of creation, redlined changes from each stage of editing, and comments that other readers added to the document before it took its final form. Proposed contracts, letters to business partners, and correspondence with opposing counsel are all free from embedded metadata. Was it proper for the attorney to scrub the metadata from electronic documents that could potentially be subject to a discovery or production request in future litigation?

Yes, because a lawyer may take measures to eliminate metadata from documents that could later fall into the hands of an opposing party.

A client hired a certain attorney to represent her in a personal injury lawsuit in which the client is the plaintiff. After an initial consultation and two meetings to review the main evidence in case and to discuss the nature of the claims, the attorney drafted the initial pleadings, served the opposing party, and filed the pleadings in the appropriate court. Nevertheless, the attorney did not allow the client to review the pleadings before filing them, and afterward, the client expresses disappointment that she did not have the opportunity to review the pleadings beforehand and make suggested edits, given that it is her case and that the attorney is working for her. Was it proper for the attorney to draft the pleadings based on conversations with the plaintiff and file the documents without first having the plaintiff review them?

Yes, because a lawyer may take whatever actions the client has impliedly authorized as part of the representation.

A prosecutor in a felony drug case addressed a group of reporters outside the District Attorney's office. In response to questions about the specific case underway, the prosecutor explained that the judge had consolidated the trials of three co-defendants into a single proceeding and had postponed the proceeding until the next summer, four months away. Was it proper for the prosecutor to disclose such details about the case to reporters?

Yes, because a lawyer may tell reporters the scheduling or result of any step in litigation

A client, who happened to be a judge, hired an attorney to represent her in her divorce proceeding against her husband, who is guilty of marital infidelity. Their fee agreement stipulates that the attorney would bill the client every month for the work performed in the previous thirty days. After two months of representation, the attorney has sent the client two bills, and has received no payments, Is it proper for the attorney to seek to withdraw from the case on the basis of unpaid fees?

Yes, because a lawyer may withdraw if the client fails substantially to fulfill an obligation to the lawyer regarding the lawyer's services and has been given reasonable warning that the lawyer will withdraw unless the obligation is fulfilled

A client explains to his attorney that he is operating an illegal website where users can anonymously upload and download pirated music and videos, in violation of copyright laws and other anti-piracy statutes. The website is very lucrative for its operator, and the client has become a multimillionaire by founding and operating the site. The client is concerned about potential criminal charges or civil lawsuits over the website. His attorney explains to the client how he could use a series of dummy limited liability corporations, mail forwarding addresses, and offshore bank accounts to avoid detection. Each of the steps of the process the attorney describes is technically legal - creating the corporate entities, purchasing mail-forwarding services, and opening bank accounts in Belize. The attorney decides not to charge the client for this advice session but bills the client for other transactional work performed. Is the attorney subject to discipline?

Yes, because a lawyer must avoid assisting a client in fraudulent or criminal activity, which includes suggesting how to conceal the wrongdoing.

While conducting research on a litigation matter, an attorney finds a very new case from the highest court in his jurisdiction that is directly adverse to his client's legal position in the case. The opposing party did not mention the case in its briefs, and the attorney realizes that the opposing party's lawyer has been recycling his firm's briefs for this type of case for several years without updating his research. Does the attorney have an ethical duty to disclose the unfavorable binding precedent to the court?

Yes, because a lawyer must disclose to the tribunal legal authority in the controlling jurisdiction known to the lawyer to be directly adverse to the position of the client and not disclosed by opposing counsel

An attorney agreed to represent a new client in a potential litigation matter, but the client had insufficient funds to pay the attorney's fees. Instead, the client asked the attorney to propose an amount that would be a reasonable fixed fee for the matter. The client then offered to sign over title to a small parcel of real estate worth about the same amount as the proposed fixed fee, and the attorney agreed. The value of the property, and the proposed fixed fee, were fair and reasonable, and the client agreed to these terms in writing. The attorney did not advise the client to seek the opinion of independent legal counsel for this transaction, and did not obtain signed, written consent from the client about the attorney's role in the transaction. Based on thesefacts, could the attorney be subject to discipline for violating the provisions Model Rule 1.8 that govern business transactions with clients?

Yes, because a lawyer must meet the written notice requirements of Model Rule 1.8 when the lawyer accepts an interest in the client's business or other nonmonetary property as payment of all or part of a fee.

An attorney represented a client in a prosecution for murder, and the prosecutor was seeking the death penalty. The trial was not going well, and the judge had not sequestered the jury, so the attorney sent his secretary to visit some of the jurors in their homes one evening, bringing them cookies and talking to them about the seriousness of sentencing a fellow human being to death. The secretary did not say she worked for the attorney, but instead introduced herself as a member of an advocacy group that seeks to abolish the death penalty, and she left pamphlets about abolishing the death penalty in each juror's home. Could the attorney be subject to discipline for this activity?

Yes, because he was communicating ex parte with the jurors through the secretary during the proceeding

An attorney is a litigator and represents a client in a civil lawsuit in which the client is the defendant. The attorney explains the general strategy and prospects of success and consults the client on tactics that are likely to result in significant expense, such as the hiring of experts or jury consultants. At the same time, the attorney believes their best shot at winning the case will be to elicit an admission from the plaintiff during cross-examination when the plaintiff testifies at trial. More specifically, the attorney plans to elicit a mild, innocuous admission during the first round of cross-examination, expecting opposing counsel to rehabilitate the witness on re-direct examination. His plan then calls for a short, direct, re-cross consisting of three yes-or-no questions that should elicit a devastating admission from the plaintiff, which opposing counsel is probably not anticipating. There has been no discussion with the client, however, about this plan for cross and re-cross. Even if the re-cross does not go as well as the attorney hopes, they might prevail in the case by several other ways. Is it proper for the attorney to leave the client out of the planning for the cross-examination and re-cross of the plaintiff?

Yes, because a lawyer ordinarily will not be expected to describe trial or negotiation strategy in detail.

An attorney testified before a state legislative committee about the need for the state to privatize its dysfunctional prison system. The attorney said he was there to testify as a concerned citizen of the state and a taxpayer, and the attorney did in fact believe that prison privatization was smart public policy. Yet the attorney did not disclose that he was representing Alcatraz Incorporated, the largest private prison company in the country, which hoped to secure the lucrative contracts to operate the state's prisons after the legislature votes to privatize them. Was it improper for the attorney to neglect to disclose his representation of the private prison company?

Yes, because a lawyer representing a client before a legislative body or administrative agency in a nonadjudicative proceeding shall disclose that the appearance is in a representative capacity.

A certain attorney represents a defendant in a murder case. At trial, the jury convicted the client and sentenced him to death, and the appellate courts upheld the conviction as well as the sentence. The attorney has now offered to file a habeas corpus petition in federal court to appeal the case to the United States Supreme Court, if necessary. The defendant, however, has developed terminal cancer, and does not expect to live another six months. The defendant tells the attorney to drop the appeals because even if they won, the defendant would not live long enough to enjoy his freedom. Even so, the defendant does not terminate the representation, because he wants the attorney to handle his estate planning matters while he is on death row, and he has some administrative complaints in progress against the prison where he is living. The attorney strongly opposes the death penalty and believes his client is innocent, so he files the habeas petition anyway. While the habeas petition is making its way through the federal appellate process, the defendant succumbs to his illness and dies in prison. Is the attorney subject to discipline for filing the habeas petition, despite the client's reservations?

Yes, because a lawyer shall abide by a client's decisions concerning the objectives of representation and shall consult with the client as to how to pursue these ends.

An attorney represents criminal defendants. One day, a client appeared in the attorney's office and explained that he had been blackmailing his former employer for the last year. The client had hired a prostitute to seduce the former employer in a room with hidden cameras, then showed the embarrassing photographs to his former employer and demanded monthly payments of $500, which the employer paid, not wanting to destroy his marriage. The prostitute subsequently died of a drug overdose. The client's former employer eventually tired of making the monthly blackmail payments and went to the police about the matter. The client is now worried that he will face charges for blackmail, which would violate his parole and result in a lengthy incarceration. The client retained the only copies of the photographs, as he merely showed them to the former employer a year ago to extort the payments. After the client explained all this to his attorney, he gave the attorney the documents and instructed the attorney to destroy them or hide them so that the police could not find them. Attorney put the photos in a folder marked ATTORNEY WORK PRODUCT - PRIVILEGED AND CONFIDENTIAL, and sent the folder to a secret overseas document storage service in the Caymans. The police obtained an arrest warrant for the client based on the former employer's affidavit, and at trial, the prosecutor obtained a conviction based on the employer'stestimony and the bank records showing the monthly transfers. Is the attorney subject to discipline?

Yes, because a lawyer shall not assist a client in conduct that the lawyer knows is criminal or fraudulent, such as destroying evidence when there is a pending criminal investigation.

A client is aware that he is under investigation for student loan fraud. A friend who works at the courthouse tips off the client that a magistrate issued a warrant to search the client's home for evidence the next day in the early morning. In a panic, the client calls his attorney, whom he has retained to represent him during the investigation and any prosecution that follows, and asks what he should do. The attorney informs him that the agents executing the warrant will surely seize any computers and hard drives that they find, and that the client should probably wipe and reformat all his drives or dispose of his computers, that he should probably smash his cell phone, and that he might want to go on a long vacation immediately. Is the attorney subject to discipline for this advice?

Yes, because a lawyer shall not counsel or assist another person to destroy or conceal a document or other material having potential evidentiary value

An attorney represented a client in an action for replevin. After the filing of the case, but before the court had sent any notices about the docket number, the attorney spoke to a clerk at the courthouse, and inquired whether the case had received an assignment yet to a judge. The clerk said it was still unassigned. The attorney then asked the clerk to mention to the Director of Judicial Administration, who was also the Chief Presiding Judge, that they should not assign the case to a particular judge, who was notorious for having a bias against parties like the attorney's client, and who had an extraordinary high reversal rate from the appellate courts in replevin cases. The clerk said he would mention the conversation to the Director, which he did. The Director said she could not accommodate special requests from lawyers regarding case assignments, but when it came time to assign the case, she assigned the case to another judge merely to avoid another embarrassing reversal from the appellate courts. Was it improper for the attorney to ask the clerk to pass his concerns along to the Director?

Yes, because during a proceeding a lawyer may not communicate ex parte with persons serving in an official capacity in the proceeding, such as judges, masters, or jurors, unless authorized to do by law or court order

Police arrested several protestors who were advocating a cause that an attorney strongly supported. One of the protestors had a violent altercation with police, and she was facing criminal charges. This attorney practices corporate transactional law and not litigation. The news media reported that jury selection would begin the following Monday in the protestor's prosecution. The attorney waited outside the courthouse where prospective jurors were reporting for jury service, and a long line formed at the metal detectors for entering the courthouse. For a long time, the attorney waited in line and started conversations with the prospective jurors in front of him and behind him in the line, during which he explained that he was a lawyer and that the case against the protestor was ridiculous from a legal standpoint. He told them that he hoped the jury would follow the laws of the state and acquit the protestor. Once the attorney made it through the security line, he walked out of the courthouse and got back in the security line again and had similar conversations with more prospective jurors. During voir dire, the prosecutor asked the prospective jurors if anyone had spoken to them directly about the case, and three people mentioned their conversations with a lawyer in the security line waiting to get into the building. None of the individuals with whom the attorney spoke ended up on the jury in the case. The prosecutor eventually determined the attorney's identity and filed a grievance with the state disciplinary authority. Could the attorney be subject to discipline?

Yes, because a lawyer shall not seek to influence a judge, juror, or even a prospective juror

During a personal injury trial, the court took a lunchtime recess for an hour. The plaintiff's attorney from the case walked across the street from the courthouse to a familiar diner to buy lunch. The diner was very crowded, so it was difficult for patrons to find a table to sit and eat. After ordering his sandwich at the counter, the attorney noticed two jurors from his own trial standing with their food, waiting for a free table. One of the jurors asked the attorney if they could share a table with him when one became available. The attorney agreed, but he reminded them that they could not talk about the case. The three sat together and ate their sandwiches. The two jurors talked most of the time, getting to know each other--discussing their children, their jobs, and their pets. The attorney did not participate in the conversation except to answer their questions about how many children he had, and whether he owned any pets. Another juror from the trial was at the diner, and noticed the attorney sitting with the other two jurors, which he reported to the judge when court reconvened. Could the attorney be subject to discipline for sharing a table with the jurors during the lunch break?

Yes, because a lawyer should not have any ex parte social contact with jurors during a proceeding, even if they do not discuss the case

An attorney defended a client in a criminal proceeding that attracted low-level media attention on the local evening news and a few local-interest blogs. A semi-retired reporter for the local evening news called the attorney at his office and asked for a quote about the client's case. Then the attorney stated that the client had no prior criminal record and that they planned to put on a rigorous defense, and he hoped the prosecutor would drop all the charges before trial. Was it improper for the attorney to make these statements?

Yes, because a lawyer should not make extrajudicial comments about the criminal record of a party during a criminal matter.

An attorney represented a client in litigation over a breach of contract. After jury selection but before the opening arguments of trial the following Monday, the opposing party contacted the attorney with a settlement offer. The attorney, an experienced litigator, familiar with opposing counsel from previous cases, and knew that opposing counsel always follows up an initial settlement offer with a better offer a day or two later. Therefore, the attorney decline the offer immediately, knowing from experience that a better offer was forthcoming. When the attorney met his client at the courthouse the following Monday for the first day of trial, he mentioned that he was encouraged by the opposing party's initial offer the previous week, which he had declined, because it meant that a more generous offer was on the way any time. The client was surprised that his attorney had not consulted with him about the offer, but he accepted the attorney's explanation for declining it and agreed they would wait for the next offer. As both parties and their lawyers took their places in the courtroom. the opposing counsel passed a note to the attorney with a new settlement offer, and just as the attorney expected, it was much more generous. The attorney and his client agreed to settle the case right then and avoided the inconvenience of going through the whole trial. Is the attorney subject to discipline?

Yes, because a lawyer who receives from opposing counsel an offer of settlement in a civil controversy must promptly inform the client of its substance prior to taking any action

An experienced attorney represented a new client in civil litigation. The client lied extensively on the witness stand during the trial, but the attorney was not aware of the untruthfulness of the statements at the time. The verdict was favorable to the client and there was no appeal. A year later, the client boasted to the attorney about lying convincingly to the court and winning the lawsuit as a result. Is it permissible for the attorney to keep this information confidential, and not disclose to the tribunal that the perjury occurred?

Yes, because a lawyer's duty to take remedial measures after perjury occurs continue only to the conclusion of the proceeding.

An attorney has represented his client in the past on various transactional matters. They have always operated under an oral agreement about the fees, and they have never had a dispute over fees in the past--the attorney would send the client a bill, and the client would pay it. Recently, the client contacted the attorney by phone about representing him as a plaintiff in a personal injury lawsuit. The attorney agreed, and then explained that he would charge a contingent fee in the case, so that the client did not have to worry about how much time his attorney had to put into the case, as the client would still receive the same she of whatever amount they won. Given their long history of working together, the attorney offers to set the contingent fee below the rate charged by other attorneys in the area, and they agree over the phone on a 25 percent contingent fee for the attorney, after costs and expenses. They never formalize this agreement in writing, though at the end of the case, after they prevail and win a large verdict, the attorney sends the client a written statement about keeping 25 percent of the award for his fee. The client is very happy with the outcome of the case and they have no dispute over this fee. Would the attorney be subject to discipline in a situation like this?

Yes, because contingent fees must always be in writing at the beginning of representation.

Client is the leader of a radical religious group that protests at the funerals of soldiers who died tragic combat deaths overseas. The protests are not against the war, however, but against society's increasing tolerance of homosexuality and gay marriage. The client and his followers stand outside the funerals as grieving family members arrive, and they hold large picket signs emblazoned with hateful sayings against homosexuals, some of which use shocking language. They also hold signs indicating they are happy that American soldiers die frequently, because they believe these deaths validate their point that the country is on the wrong course morally and has become evil by being more tolerant. The group heckles those attending the funerals, but then disperses once the funeral ceremony starts. The group receives regular national media coverage because of the intentionally sensational and shocking nature of their protests. The client now faces a tort lawsuit by the father of a deceased soldier whose funeral the group picketed; the plaintiff claims intentional and negligent infliction of emotional distress. The client is certain that his First Amendment rights trump such subjective-harm tort claims and has a recent Supreme Court case supporting his position. The client asks an attorney to represent him in the matter. The attorney reluctantly agrees to take the case and the trial court gives an unfavorable verdict against the client. After the case, reporters interview the attorney asking how he could represent such a client and the attorney states during the interviews that he did not necessarily endorse the client's religious, social, moral, or political views, but was merely providing representation. Are the attorney's actions proper in this case?

Yes, because a lawyer's representation of a client does not constitute an endorsement of the client's political, economic, social, or moral views or activities.

In preparation for trial, an attorney and the client sat down together to go over the client's upcoming testimony. The client mentioned, as he recounted his version of the facts, something that the attorney knew would constitute an admission of fault on a critical point in the case. The attorney interrupted the client and said, "if you admit that, you will have forfeited your entire case." The client nodded to show his comprehension of what the attorney said. The client testified at trial and changed his story significantly, carefully omitting the statement that the attorney had identified as a legal admission of guilt. Did the attorney violate the Rules of Professional Conduct in preparing the client for his testimony in this way?

Yes, because a layer must not counsel another person to conceal a matter with evidentiary value

A client hires an attorney to represent her in business litigation. Another lawyer in the firm, unknown to the attorney, approaches the client with a proposal for an unrelated business transaction, the sale of a parcel of real estate adjacent to the lawyer's own land. The client agrees to sell the other lawyer in the firm the parcel of real estate for a reasonable price. The lawyer is not involved at all in the representation of the client and works exclusively in the estate- planning department of the firm, rather than in litigation. Must the lawyer nevertheless advise the client in writing of the desirability of seeking the advice of independent legal counsel, and obtain written informed consent from the client before proceeding with the purchase?

Yes, because a prohibition on conduct by an individual lawyer under the conflicts of interest rules would automatically apply to all lawyers associated in a firm with the personally prohibited lawyer, even if the first lawyer is not personally involved in the representation of the client.

An attorney agreed over the phone to represent a client and began working on the case immediately. The client came into the office two weeks later to sign the representation agreement. At the same time, the attorney gave the client a written statement of the hours worked so far and requested immediate payment for that portion of the fee, plus a $10,000 retainer up front against which the lawyer would draw fees as the representation proceeded. The fee arrangement was complicated, in addition to the hourly fee for the time he had already worked, the agreement called for an hourly rate of $150 per hour for any work done before trial. If the case were to go to trial, the hourly fee would be $250 per hour for the entire trial phase and any appeals. The agreement also stipulated that it incorporated by reference any oral agreements regarding additional fees and expenses. The client signed the agreement. Then the lawyer explained orally in addition to the hourly fees and the non-refundable retainer, he would take a 25 percent contingent fee of any money that the other side had to pay the client as a result of the representation, whether in damages, as there were claims and cross-claims in the case, or in court-ordered attorneys' fees. The client agreed, and they shook hand to confirm their oral agreement. Finally, the agreement authorized the lawyer to have full discretion to accept or reject any settlement offers without prior approval from the client, although no such offers occurred. The case proceeded through the discovery phase and went to trial. On the last day of the trial, before closing arguments, it appeared that the client might win a large verdict. The client became resentful about the prospect of sharing this with the lawyer and fired the lawyer during a recess before closing arguments. The client returned to the courtroom alone, waived his right to closing arguments, and still won a significant verdict. The client now refuses to pay the lawyer the contingent fee or even the hourly fees for the last day of trial, because the client claims the attorney performed incompetently that day. The attorney has threatened to sue the client to obtain the fees. Could the attorney be subject to discipline?

Yes, because all contingent fee arrangements must be in writing, not merely oral agreements incorporated by reference.

Three sisters sustained injuries in a car accident last year. Because they did not have enough money to get separate attorneys, they decided to consent to using the same attorney. Before trial, the opposing party made a settlement offer. One of the sisters had sustained severe injuries, but the other two had only superficial scrapes and bruises. Their attorney believed that the settlement offer was reasonable, and that it would be in the clients' best interest to accept it to avoid the costs of trial. Even so, the attorney expected the sister with the more serious injuries to be resistant to a settlement offer, because she might obtain a larger judgment if she presented her case in front of a jury. If the attorney obtains consent from the other two sisters to accept the defendant's settlement offer will he be subject to discipline if he accepts the offer without discussing it the third sister, who had the worst injuries?

Yes, because an attorney must inform and obtain written consent from each client about all the material terms of the settlement before accepting any settlement offer on behalf of multiple clients

A court orders that a certain client should receive child support from her ex-husband. The client's ex-husband stopped making child support payments twelve months ago. The client hires an attorney to handle the enforcement of child support against the client's ex-husband. The attorney agrees to take the case on a contingency basis because the client cannot afford to hire an attorney since she has not been receiving child support from her ex-husband. The client also asks the attorney to pay her court costs, as she cannot afford those either. The attorney prepares a contract that states the attorney will only be paid for his representation if the client prevails on the enforcement motion, but that court costs will be reimbursed by the client within thirty days of the finalization of the case regardless of whether the client prevails. Is the attorney's conduct proper?

Yes, because attorneys may accept cases on a contingency basis in domestic relations issues if the case is merely to enforce a prior order, and attorneys may pay court costs for clients

A certain attorney represents a client in a litigation matter. The client was not present during the last pre-trial hearing at which the lawyers argued about whether certain experts on each side could testify at trial. The trial was to start the following week. At the end of the hearing, the opposing counsel asked the court to have the record sealed in the upcoming trial, and to have reporters banned from the courtroom.He explained that the testimony at trial would necessarily reveal some of his client's trade secrets, and it was important to the client to keep the trial records sealed. The judge was amenable to this suggestion and asked the attorney if he had any objections. The attorney tried to call the client, but the client did not answer his phone right then. Unfortunately, the attorney could not think of a compelling reason for the client to oppose the motion, so he agreed, and the judge set the matter for a sealed-record trial. Three hours later, the client returned the attorney's call, and the attorney explained what had transpired. The client felt dismayed because he had planned to use this litigation as a test case for subsequent litigation over the same type of issue, but the attorney explained that it would now be difficult to get the judge to reverse course on this point. Was it proper for the attorney to agree to the request without obtaining the client's prior consent?

Yes, because during a trial, when an immediate decision becomes necessary, the exigency of the situation may require the lawyer to act without prior consultation, assuming the lawyer promptly informs the client of actions the lawyer has taken on the client's behalf.

A certain client hired an attorney to represent him in litigation because of the attorney's reputation for being the meanest, most aggressive litigator in town. The client is the defendant and the attorney bills by the hour. The judge in the case orders the parties to participate in a "caucused mediation" to encourage a settlement before trial. Then the attorney begins the mediation by declaring that his client is unwilling to compromise at all, even though the client had told him that they might settle the case for a reasonable amount. The attorney overstates the strength of the client's case and grosslyunderstates the strength of the opposing party's position in what everyone knows is a close case. The attorney is merely posturing or bluffing in hopes of obtaining a more favorable settlement for his client. Due to the attorney's hardline approach, the mediation drags on for several sessions spanning several days, and proves to be futile, so the parties schedule a trial. Is the attorney potentially subject to discipline for this approach in court-ordered mediation?

Yes, because even if the statements were not material facts, lawyers must make reasonable efforts to expedite litigation consistent with the interests of the client.

A local abortion clinic hires the McCorvey Law Firm to represent it in an enforcement action brought by a state health agency. The action pertains to alleged health code violations at the clinic. The firm's principle partner, Norma McCorvey, has strong, outspoken political beliefs against abortion, and cannot set aside her personal convictions to provide representation to the clinic in the matter. An associate at the firm, however, supports the clinic's mission, and offers to represent the clinic instead of Attorney McCorvey. If McCorvey agrees to let the associate represent the clinic, would it be proper for the associate to do so, despite the partner's strong convictions that the clinic should close?

Yes, because even though Attorney McCorvey could not effectively represent the client due to her political beliefs, this would not materially limit the representation by the associate at the firm.

An attorney has represented a client on various small matters in the past. The client now needs representation for a more substantial matter involving a business transaction. During a phone call, the attorney agrees to represent the client at a slightly higher hourly rate, given the complexity of the matter, and when they meet to discuss the transaction in more detail, the attorney double-checks with the client about the fee arrangement verbally, explaining it carefully and answering any questions the client may have. The attorney and the client never formalize the fee arrangement in writing, but the attorney does send printed bills to the client periodically. Eventually, the client starts to feel that the representation is costing too much, and objects to one of the bills. Was it permissible for the attorney to have an oral agreement over hourly fees, without putting the fee arrangement into writing?

Yes, because even though it is always preferable to have fee arrangements in writing, it is not required in this type of case.

An attorney meets with a potential client regarding representation in defense of a nuisance suit. Upon review of the original petition, the attorney realizes that she knows plaintiff's lawyer. The attorney and plaintiff's lawyer worked together as associates at the same law firm directly out of law school twenty years ago but have not practiced law together since the attorney started her own firm eleven years ago. The two lawyers do not regularly socialize together or otherwise encounter one another aside from occasionally crossing paths at the courthouse. From time to time, when they coincidentally come across one another at the courthouse, they have lunch together at a nearby café. The attorney discloses the nature of her relationship with the plaintiff's lawyer. The potential client states that she still wants to retain the attorney, but the client does not provide written informed consent. Will the attorney be subject to discipline if she does not decline representation in this case?

Yes, because even though the attorney disclosed the relationship to the client, she failed to obtain a written informed consent from the client.

A client hired an attorney to research the legality of a musical "mash-up," a sound recording that includes brief sound clips and samples from many other artists' commercial recordings. The client's unique approach puts it in the gray area around "fair use" and "composite works of art" under prevailing copyright law, and no court has yet ruled on the precise issue, though the question has been the subject of seventeen lengthy law review articles in the last two years, reaching a range of different conclusions. No litigation is pending, and the client has not yet undertaken any activity that could constitute a copyright infringement; he is seeking reassurance before proceeding that he would not face liability for copyright infringement. Because the client primarily wants a memorandum of law answering his hypothetical legal question, he asks the attorney to limit his research and writing to two hours of billable time. The attorney agrees, spends an hour reading and an hour writing, and gives the client a short memorandum. Given that the client's objective was merely to secure general information about the law the client needs, was it improper for the attorney to agree to this limitation on the scope of representation up front?

Yes, because given the complexity of the subject and the uncertainty about this certain point of law, two hours was not a reasonable amount of time to yield advice upon which the client could rely

A witness testified on a client's behalf at trial. That evening, when the attorney was reviewing exhibits and documents to prepare for the next day of trial, he noticed a document that completely negated the witness' testimony from earlier that day. The testimony was material evidence in the case. The witnesses left the jurisdiction after his testimony concluded, and he is no longer available to correct the false statements. The opposing party's lawyer waived his opportunity to cross-examine the witness, because the testimony was unfavorable to his side and he was eager to move on to a more favorable witness. Does the attorney have a duty to take remedial measures to correct the false testimony, such as disclosing the falsehood to the court?

Yes, because if a witness called by the lawyer has offered material evidence, and the lawyer comes to know of its falsity, the lawyer shall take remedial measures, including, if necessary, disclosure to the tribunal

Big Firm represents hundreds of corporate clients out of a dozen offices in different states. The firm has no formal procedures in place to check for clients at the outset of representation for new clients, but the managing partner of the firm has an incredible memory and has never failed to spot a potential conflict of interest in the past. Attorney agrees to represent a new corporate client that owns many subsidiaries, and checks with the managing partner, who assured Attorney there are no potential conflicts. After the new corporate client had disclosed a substantial amount of confidential information, it emerged that some of its subsidiaries were directly adverse to other clients of Big Firm. Attorney was completely unaware of the potential conflicts at the time he agreed to the representation, despite asking the corporate client a few questions about the opposing parties in pending litigation it might have. Will Attorney be subject to discipline for not declining representation in this case?

Yes, because ignorance caused by a failure to institute reasonable procedures, appropriate for the size and type of firm and practice, will not excuse a lawyer's violation of the Rules regarding conflicts of interest

A client is a defendant in a criminal prosecution, and a certain attorney is his court- appointed defense lawyer. The client wants to testify at his own trial, despite the attorney's recommendations that he not do so. As they are preparing for trial, the attorney asks the client what he plans to say on the stand. The client's story seems suspicious to the attorney - he has serious doubts about its veracity - but the client insists that he is telling the truth, and the attorney is not sure. Does the attorney have an ethical duty to allow the client to give this improbable testimony at trial?

Yes, because in a criminal case, a lawyer cannot refuse to offer the testimony of a client where the lawyer believes, but does not know, that the testimony will be false; unless the lawyer knows that the testimony will be false, the lawyer must honor the client's decision to testify.

An attorney represents a defendant in a criminal matter. The defendant face felony charges. The attorney is very experienced in handling this type of case and knows from experience that defendants receive acquittals far more often in jury trials than in bench trials, at least with this type of case. The client, however, does not want to incur the legal fees involved in jury selection (voir dire, etc.) and cannot really afford it, so the client tells his attorney that he does not want a jury trial, but rather a bench trial. The attorney is convinced that his client is innocent of the crimes charged, and that a bench trial is likely to result in a wrongful conviction in this case, given some of the evidentiary issues. The attorney postpones notifying the court that the defendant will waive his right to a jury trial, in hopes of changing the client's mind. The court schedules jury selection, and the attorney appears and participates in the voir dire without telling his client, because he still hopes and believes that he will change his client's mind about the issue. On the first day of trial, the client arrives in court and is shocked to see a jury seated. The defendant stands and objects loudly to the jury and explains that he wants to waive his right to a jury trial and have a bench trial instead. The judge refuses to dismiss the jury at this point, informing the defendant that his opportunity to request a bench trial has passed. The trial proceeds and the jury acquitted the client of all charges, as the attorney had expected, and to the apparent dismay of the judge, who would have ruled to convict if it were up to him. Is the attorney subject to discipline in this situation?

Yes, because in a criminal case, the lawyer shall abide by the client's decision, after consultation with the lawyer, as to whether to waive the right to a jury trial.

Am attorney worked as a litigator, but she is also a single mother of two young children. She had to pick her children up from daycare every weekday by 4 P.M. As a result, whenever she was scheduling hearings, conferences, settlement negotiations, or trial dates, she simply refused to schedule anything in the late afternoon, as that could easily run into the time when she had to pick up her children. The result is that her cases would stretch out over a long period, as she was available for hearings, trials, and other litigation-related meetings only in the mornings and early afternoons, and otherwise must seek postponements. Could the attorney be subject to sanctions for managing her schedule in this way?

Yes, because it is not proper for a lawyer to fail routinely to expedite litigation solely for the convenience of the advocates

An attorney located a witness who could corroborate his client's story. The witness, however, was afraid of retaliation from others if she testified, and did not want to be involved. The witness also lives 1,000 miles away and works as a waitress, so she cannot afford the travel expenses and lodging, and cannot afford to miss work, because she receives no wages if she does not work. The attorney offers to pay all the witness's expenses. The attorney then pays for airfare and pays to put the witness in one of the nicest hotels in the city, and pays for all of the witness's dining bills at expensive downtown restaurants. Was it proper for the attorney to offer to pay the expenses for a favorable witness to undergo the trouble of testifying at trial?

Yes, because it is proper to pay a witness's expenses, so long as the attorney does not offer to pay the witness an inducement to provide favorable testimony

An attorney represents a client in a family law matter. A hearing is set for Monday. On the Wednesday prior to the scheduled hearing, the client calls the attorney and advises that the client no longer wants the attorney to represent her; the attorney's representation is over as of the date and time of the call. The client advises that she intends to retain another attorney prior to the hearing. After receiving the call from the client, the attorney schedules another matter for Monday, does not appear at the hearing, and does nothing further on the case. Is the attorney subject to discipline?

Yes, if representation has begun, the attorney must to withdraw from the case and take reasonable steps to mitigate consequences to client if discharged by client.

An attorney works in a partnership with one other lawyer. A client wants the attorney to represent her in litigation over a contract dispute, because the attorney helped negotiate the contract. In fact, the attorney was the only other party in the room when the client and the other party reached a final agreement on the terms and signed the contract. The attorney explains that he may have to testify as a witness at the client's trial, as the dispute involves the parties' intention regarding a certain ambiguous provision of the contract. The attorney said he would truthfully corroborate the client's version of the events. As a result, the attorney explains, he cannot represent the client at the trial, but his partner at the firm (a two-lawyer partnership) could represent the client instead. The client retained the attorney's partner to represent her in the litigation. Is this arrangement proper?

Yes, because lawyer may act as advocate in a trial in which another lawyer in the lawyer's firm is likely to be a witness

An attorney represented an alternative energy firm that is lobbying the state legislature to provide subsidies for companies that develop wind, solar, or geothermal energy sources. When appearing before a legislative committee, the attorney disclosed that he represents the company and submitted reports from his client about the efficiency of his client's products and the saving that could accrue to the public if more people used their products. The reports also purported that the company was having trouble staying in business and could not survive without a large government grant or subsidy. The attorney knew, however, that many of these figures were inaccurate, and that in fact the company was making a handsome profit on products that were less efficient the fossil fuel sources of energy. Was it improper for the attorney to submit such documents to a legislative committee?

Yes, because lawyers appearing before a legislative body in a non adjudicative proceeding shall conform to the same standards of candor and honesty that apply in the courtroom.

A thirty-lawyer firm in Chicago affiliated with Boutique Firm, three lawyers in a small city in New England. Each firm includes, on its masthead under the list of its own lawyers, the affiliation of the other firm (with its lawyers each named). Each firm also mentions the affiliation with the other in its Martindale-Hubbell listing. Boutique Firm has represented Conglomerate Corporation in intellectual property matters for a few years, and has on file extensive information about Conglomerate's patents, patent applications, and prior patent litigation.Recently, Copycat Company has hired the thirty- lawyer firm in Chicago to seek a declaratory judgment that it is not infringing on certain patents owned by Conglomerate Corp., or in the alternative, that these specific patents are invalid. Conglomerate Corporation hired a new litigation firm to represent it in the matter, due to its concern about its regular firm having a conflict of interest. During the pleading and discovery phase, Conglomerate filed a motion to disqualify the Chicago firm from representing Copycat Company, due to its affiliation with Boutique Firm, even though Boutique Firm is not handling Conglomerate's litigation in this matter. Should the court grant the motion to disqualify the Chicago firm?

Yes, because separate firms that publicly identify themselves as "affiliated," even if they are located several states away from each other, count as the same firm for purposes of imputed conflicts of interest under Rule 1.10.

A new federal Treasury Regulation provides that attorneys who prevail in tax cases on behalf of their clients against the Revenue Service may receive attorneys' fees at the fixed rate of $100 per hour, not to exceed $100,000. A certain attorney lives in a state that allows "reasonable" fees, and he makes a written fee agreement with the client for an additional $100 fee per hour, on top of whatever fees the Treasury Regulations allow in their case. If the client provides written informed consent, could the attorney be subject to discipline for this fee agreement?

Yes, because state rules about legal fees are subject to limitations by applicable law, such as government regulations regarding fees in certain tax matters.

A client hired an attorney to handle a transactional matter. The client, a billionaire, wants to devote several million dollars to philanthropy. There are several alternative ways to achieve the client's goals--incorporating a 501(c)(3) charitable corporation, establishing a private foundation, creating a charitable trust, operating a nonprofit unincorporated association, or simply donating the money to an existing charity of some kind. Each alternative has different pros and cons regarding immediate tax benefits for the donor versus tax deductions for subsequent contributors, permissible activities for the charitable entity, donor control versus independence, eligibility for government grants, and administrative costs related to accounting and record keeping. The attorney does not discuss all of these details with the client, though, because the client said at the outset that he trusted his attorney's judgment, and the attorney believed the client would find the details tiresome and confusing. The attorney set up a private foundation for the client because this could provide his client with the greatest immediate tax benefits and the highest degree of control in the long term. The downside was that the private foundation option involved burdensome paperwork and reporting to the IRS every year, imposed annual spend-down requirements, and limited the tax benefits for any other philanthropists who wanted to donate to the foundation later. The attorney believed the pros outweighed the cons in this case, but the client was unhappy because he wanted to start something that would grow and attract other wealthy philanthropists who might get involved, and the administrative costs drained some of the funds that the client had hoped would go directly to charitable causes. Could the attorney be subject to discipline for how he handled the matter?

Yes, because the Rules of Professional Conduct require a lawyer to consult with the client about the means to be used to accomplish the client's objectives.

An attorney normally represents a client in commercial litigation matters, but in one specific case, the attorney had to testify as a witness during the trial, so he arranged for another firm to represent the client during the trial at which the attorney testified. The client prevailed at trial, and the opposing party filed an appeal. In this instance, the attorney's testimony from the trial is not an issue in the appeal; instead, the appeal focuses on the apportionment of fault and certain guarantees in a commercial contract. The firm that handled the trial did not do appellate work and ended their termination of the client after the trial ended in a favorable verdict. May the attorney represent the client in the appeal, even though the attorney testified at the trial?

Yes, because the advocate-as-witness rule applies only to representation during the trial, unless the lawyer's testimony is an issue on appeal.

An attorney is representing himself in his divorce proceeding. Would it be proper, under the advocate-witness rule, for the attorney to testify as a witness on his own behalf in the proceeding in which he represents himself?

Yes, because the advocate-witness prohibition does not apply to pro se litigants who are attorneys.

An attorney made an agreement to borrow money from a client who had received a large inheritance. The attorney agreed to pay the client the same interest rate that banks in that area were charging for unsecured business loans, and she gave the client a detailed written disclosure of the terms and conditions of the loan, with phrasing that a nonlawyer could understand. The client gave written, signed consent to the essential terms of the loan, including the fact that the attorney was not representing the client in the transaction. During one of their conversations about the loan, the attorney also advised the client in writing that it would be prudent to obtain the advice of another lawyer about the transaction, and she offered to give the client time to find another lawyer, but the client did not want to do this. Upon consummation of the agreement, the client transferred the loan amount to the attorney, who made regular payments according to the terms of the agreement, eventually repaying the full amount with interest. Based on these facts, were the attorney's actions proper in this transaction?

Yes, because the attorney complied with the requirements of the Model Rules for this type of transaction with a client.

An attorney responded to a distressed call from a client asking that he meet him immediately on the street behind the attorney's office. Immediately, the attorney rushes downstairs to meet the client outside his building. The client is very distraught and has blood splattered on his clothes, hands, and face, and is holding a pistol. The client stammers, "You will not believe what just happened." Quietly, the attorney takes the pistol and throws it down the closest storm gutter on the street, and they can hear the gun clanging against concrete as it tumbles deep down into the storm sewer. Then the attorney says, "It is late, and you are too upset to talk. Go home and clean yourself up and do your laundry - you are a mess. We can discuss this tomorrow morning when you are in a better frame of mind." The client goes home to shower and launder his clothes, and the attorney returns to his office and resumes his work on the brief he was writing. Did the attorney's conduct constitute a violation of his ethical duties?

Yes, because the attorney concealed or obstructed the police's access to potential evidence by discarding the gun, and he counseled the client to destroy the evidence on his clothes.

Two successful business partners hired an attorney to help with the dissolution of the partnership, as the two partners no longer want to work together. The attorney was very concerned about the obvious conflict of interest, but the partners insisted that they did not want to complicate matters unnecessarily by hiring separate counsel, and that they were already in complete agreement about how to divide the business. They even signed informed consent statements acknowledging and waiving the conflict. Each partner believed it would be in both of their best interests to use only one lawyer to dissolve the business. The matter was purely transactional at this point, and it did not involve any anticipated litigation before a tribunal. One partner had already mentioned to the attorney that he might need his legal services in setting up a new business, as well as handling some estate planning issues for him. The attorney still believed that dual representation was not a good idea, given the complexity of the business and the debatable future value of some of the patents, goodwill, and other intellectual property involved, and because one partner contributed a much larger share of the start-up funds years before. The partner who mentioned hiring the attorney to do estate-planning work wanted the dissolution to include assigning his proceeds from the business to his heirs. The attorney proceeded with the dual representation and the dissolution appeared to proceed smoothly. Is the attorney subject to discipline for representing both partners?

Yes, because the attorney did not have a reasonable belief that he would be able to provide competent and diligent representation to each affected client

A certain attorney is a criminal defense lawyer, and he represents a client, who is facing charges for burglary of a private residence. The client has asserted an alibi - he claims that on the evening of the burglary, he was 100 miles away on a romantic getaway with his girlfriend. Naturally, the attorney interviews the client's girlfriend, who recounts a similar story about being on a romantic getaway, but a few details do not match the client's account, such as what they ordered for dinner when they stopped at a restaurant, and whether they had to stop for gas along the way. The attorney suspects the girlfriend is lying to protect the client, and that they rehearsed an alibi story without working through the fine details together. The attorney lectures both the client and his girlfriend about the wrongfulness of perjury and the fact that they do not have to testify at all, as well as the hazard of having their stories crumble under rigorous cross-examination. Is it permissible, under the Rules of Professional Conduct, for the attorney to call the client and his girlfriend as witnesses during trial?

Yes, because the attorney does not know with certainty that they are lying, he must allow the client to testify, and it is permissible to call the girlfriend as a witness as well.

An attorney agrees to represent a client in a divorce proceeding against her husband. The client is particularly concerned about obtaining her fair share of the marital property or assets--as much as possible in fact--as well as a suitable level of child support for their child support for their children. The client agrees to pay the attorney his usual flat fee for divorce cases, $5,000, but also offers to pay him 10 percent of whatever he wins in terms of payments and distribution of assets, on top of his usual fee. After a protracted, acrimonious divorce proceeding, the attorney obtains a settlement worth approximately $2 million for the client. Is the attorney subject to discipline in this scenario?

Yes, because the attorney entered into an arrangement for a fee in a domestic relations matter, the amount of which was contingent upon the amount of alimony, support, or property settlement.

In response to an attorney's advertising, which describes the attorney's education, experience, areas of practice, and contact information, and individual sent an email to the attorney describing their legal problem at length, including many personal details. Some of the information was unfavorable to the individual's legal interests. The attorney, who had never met or had any contact with the individual, read the long email in its entirety, and immediately sent a terse reply declining the representation. There was no consultation with the individual, and the attorney did not promise to provide representation. A few days later, the attorney received an inquiry from the opposing party in the case, and he agreed to represent the opposing party, and used information gleaned from the other individual's email to prevail in the matter. Was the attorney's conduct proper?

Yes, because the individual who sent the original email was not a prospective client for purposes of the Model Rules, and the attorney had no duty to keep the information confidential.

An attorney agreed to represent a client as plaintiff in a patent infringement lawsuit. The attorney was part of a partnership that specialized in intellectual property law. The attorney prepared, and the client signed, a written fee agreement that specified the attorney would receive a tiered contingent fee in the case; 25 percent if the case settled before trial, 30 percent if they went to trial and won, and 35 percent if the case went up on appeal and they prevailed in the appellate stage. In addition, the agreement specified that the contingent fee would come from total award before court costs and other expenses, and that the client would be responsible for court costs and expenses out of his own pocket, either along the way as expenses arose during the proceedings, or from the client's share of the award after the attorney received his contingent fee. The attorney never revealed that his partnership agreement required him to share his part of the fees with three other partners in budget from which the partnership paid the salaries of non-lawyer staff, such as paralegals and secretaries. The attorney obtained a favorable settlement before trial. He telephoned the client with the good news and explained that he would deduct his 25 percent contingent fee, as they had agreed, and would send the client the remainder of the settlement funds. At that time, there were no outstanding unpaid expenses or court costs. The client was glad to hear the news, and the attorney promptly sent the client a check for 75 percent of the total amount received from the other party. The attorney and the client had no other contact except to exchange holiday greeting cards. Were the attorney's actions improper?

Yes, because the attorney failed to provide the client with a written statement stating the outcome of the matter and showing the remittance to the client and the method of its determination

An attorney worked in the legal department of Conglomerate Corporation for a few years, then left there to start his own firm. His experience at Conglomerate proved useful, as he regularly represented some of Conglomerate's newer industry rivals in their transactional and pre-litigation work - small startup businesses that did not have in-house counsel. Whenever a new client needed legal representation in a matter that could potentially be adverse to the legal interests of one of his other clients, the attorney would obtain informed consent, confirmed in writing to the potential conflict of interest. In such cases, the attorney would also ask new clients to sign a waiver of liability for all potential legal malpractice by the attorney. Attached to the waiver was a cover sheet explaining what the waiver entailed, the downsides for the client in signing a waiver, and recommending the client seek the advice of independent legal counsel in connection therewith. As with the consent to conflicts of interest, the clients normally gave informed consent, confirmed in writing, to the waiver of malpractice claims against the attorney. Could the attorney be subject to discipline, based on these facts?

Yes, because the attorney is making an agreement prospectively limiting the lawyer's liability to a client for malpractice, and the client does not have independent representation in making the agreement.

An attorney represented a client in a criminal prosecution. The client agreed to a plea bargain, and the case moved on to a sentencing hearing. The prosecution's pre-sentencing report to the judge erroneously indicates that the client has no prior convictions, and the trial judge asked the client directly whether that is true. The client affirmed that he had no prior criminal record, and the judge sentenced him leniently, giving him six months' probation. Yet the attorney had represented the client previously in another jurisdiction in a criminal matter, and he knew that the pre-sentencing report was erroneous. Before adjourning, the judge asked the attorney if he had anything else to say. Could the attorney be subject to discipline if he does not correct the judge's misperception about the client's criminal record?

Yes, because the attorney must not allow his client to offer evidence that he knows to be false to a tribunal

An attorney agreed to represent a plaintiff in a personal injury lawsuit, and the client agreed to pay the attorney a contingent fee based on a percentage of the award in the case. The attorney put all the terms of the fee agreement in written form in a letter to the client. The letter explained the percentage that should accrue to the attorney the event of settlement, trial, or appeal; litigation and other expenses that the attorney would deduct from the recovery; and that such deductions would come out of the total before the calculation of the contingent fee. The letter also explained all potential expenses for which the client could be liable, if the client prevailed in the case or not. The client received the letter, read it carefully, and called the attorney to give verbal assent and confirmation to all the terms. The client's spouse later discarded the letter, and the attorney proceeded with the representation. Could the attorney be subject to discipline, based on these facts?

Yes, because the client did not sign the fee agreement.

An attorney represented a client in a litigation matter, and while the matter was still pending, the attorney and the client also agreed to purchase an investment property together.The client had another lawyer who regularly represented the client in transactional matters, but not litigation. The litigation attorney and the client contributed equal amounts toward the purchase of the investment property, and each received an equal share. The attorney did not advise the client in writing of the desirability of obtaining the opinion of independent legal counsel in the transaction, but the client nevertheless asked his other lawyer, who handled the client's transactional matters, to review the terms and render an opinion. The other lawyer provided the client with a written disclosure of the terms and conditions of the agreement and recommended that the client proceed. Did the litigation attorney act properly in thistransaction, purchasing an investment property with the client?

Yes, because the client had representation by another lawyer in the transaction.

During a trial, the judge overruled an objection by one of the attorneys. The attorney felt that the judge had made a fundamental error and had ignored a clear provision of the official Rules of Evidence. Court adjourned for the day a few minutes later, and the judge retreated to his chambers. The attorney approached the judge's clerk, who was still in the courtroom, and gave him a handwritten note, folded into a square, to pass along to the judge. The clerk gave the note to the judge. The note thanked the judge for recently inviting the attorney to the judge's home, along with sixty other people from the legal community, for a holiday party. It also said that the judge had made a mistaken ruling on the attorney's objection that day, and it referred the judge to the relevant provision of the Rules of Evidence. Could the attorney be subject to discipline for his actions?

Yes, because the lawyer communicated ex parte with a judge during the proceeding, without being authorized to do by law or court order

An attorney represented a client in litigation over a breach of contract. After a long period of discovery, as the trial date approaches, the two parties make a new attempt at settlement negotiations, with each party's lawyer acting as representative. The client is the plaintiff in the case and has told the attorney on several occasions that she will not consider any settlement offer less than $100,000. The client is a sophisticated business owner who has weathered litigation many times in the past, including litigation over a breach of a near-identical contract term. Based on her experience, the client has made an informed estimate that her chances of winning a $250,000 verdict at trial are almost exactly 50 percent, and that trial expenses are likely to be around $50,000 whether she wins or loses, and from there she derived her reserve amount of $100,000. The attorney met with client the evening before the attorney would meet with opposing counsel for negotiations, and the client reiterated her reserve amount to the attorney, adding, "Do not even call me if the opposing party offers less than $100,000--I will not accept it, and I want you to simply decline lowball offers." The next day, the client leaves on a business trip, and the attorney heads to the settlement negotiation meeting, where opposing counsel offers $90,000 to settle plus a written apology from the defendant to the client for breaching their contract. May the attorney reject this offer without first consulting with the client?

Yes, because the client has previously indicated that the proposal will be unacceptable and has authorized the lawyer to reject the offer

A certain defendant was indigent and received court-appointed defense counsel in his felony larceny case. The defendant insisted that he was completely innocent and that he would not accept any plea bargains, because he wanted an opportunity to prove his innocence at trial. When the defendant told the attorney his expectations, the attorney explained that there is a special type of plea called an "Alford Plea," in which a defendant may agree to accept a conviction while still contesting his guilt or maintaining his innocence. The defendant refused, and told the attorney, "Do not even contact me with offers from the prosecutor for a guilty plea. I will not plead guilty. I will prove my innocence in a court of law!" The prosecutorindeed made several plea offers, and each time the attorney presented the offer to the defendant, who rejected it and reminded the attorney that he did not want to hear about any offers to "make a deal." The defendant's hard line proved effective as a negotiating strategy, and eventually the prosecutor called the attorney to say they would reduce the charges to a misdemeanor and the sentence to "time served" if the defendant would plead guilty. The attorney thought this was a ridiculously generous offer but simply rejected it without consulting his client. The client proceeded to trial and the jury convicted him, and he received the maximum sentence for the crimes charged. Was it proper for the attorney to reject the final plea bargain offer without informing the client?

Yes, because the client has previously indicated that the proposal will be unacceptable and has authorized the lawyer to reject the offer.

An attorney agreed to represent a plaintiff in a claim against the client's employer for intentional infliction of emotional distress, because of insulting remarks the supervisor makes about his subordinates' intelligence and maturity. The attorney researched past court decisions and concluded that intentional infliction of emotional distress claims usually lose in employment settings like this. Moreover, in his various discussions with the client, the story has changed a little each time. The attorney now suspects that the client either is lying or is so confused that he will not be a credible witness at trial. The attorney would like to withdraw before filing an answer to the lawsuit asserting a defense of mistake of fact, because he knows they are unlikely to win, and he is not even sure if his client is telling the truth. Nevertheless, the client insists that the attorney should file the complaint before withdrawing from the case, so that the client does not miss the statute of limitations and forfeit the potential claim, but the client does not mind if he must find another lawyer to handle the discovery and trial phase. Would it be permissible, under the Model Rules, for the attorney to file the complaint, alleging intentional infliction of emotional distress?

Yes, because the client's defense has some basis in fact and law, even if it seems improbable in both regards.

A certain client calls an attorney to ask if it is possible to apply for an extension on filing his annual tax returns, if the deadline for filing returns is still two weeks in the future. This attorney offers to research the matter for a few hours and write a formal legal memorandum for the client about filing extensions. Even so, offhand, the attorney can assure the client over the phone that it is indeed possible to apply for an extension and that the IRS routinely grants them if an application for extension arrives before the regular deadline. The attorney practices tax law and is familiar with the rules. The client thanks the attorney and says that he is satisfied with the "short answer," and that he does not want the attorney to do any more research or writing about it, but to send a bill for the phone call. Then the attorney agrees and bills the client for the telephone conversation and conducts no further research on the matter. Is it proper for the attorney to limit his representation to a single telephone call like this?

Yes, because the client's objective is no more than securing general information about the law the client needs to handle a common and typically uncomplicated legal problem, so the lawyer and the client may agree that the lawyer's services will be no more than a brief telephone consultation.

A client is struggling through a deposition, during which opposing counsel is subjecting him to intense questioning. The attorney, who represents the client, tries objecting a few times in order to break the opposing counsel's momentum, but it was to no avail. The attorney then stood up, shouted, and with a heave overturned the conference table around which the lawyers, court reporter, and deponent were sitting. Notes, cell phones, and open briefcases flew across the room, and the stenographer's equipment tumbled to the floor. The attorney and the client gathered their things and stormed out of the room. A few days later, the attorney called opposing counsel and halfheartedly apologized, and agreed to reschedule the deposition if opposing counsel would agree to behave himself this time. Opposing counsel reported the attorney to the state bar disciplinary authority. Could the attorney be subject to discipline for the ay in which he disrupted the deposition?

Yes, because the duty to refrain from disruptive conduct applies to any proceeding of a tribunal, including a deposition.

Three individuals plan to form a joint venture and ask an attorney to represent them in drafting the necessary documents and making the necessary filings with government agencies. They have already agreed that everyone will contribute exactly one-third of the startup funds for the venture, each will own a one-third share, each will have equal control over the Board, and each agrees to indemnify the others for a one- third share of any personal liability related to the joint venture. They have also agreed that they will have no non-compete agreements. The joint venture will hire managers, marketers, and other employees to operate the business. The three individuals are co-owners of a patent that could potentially be very lucrative when they bring it to market, and they have known each other and worked together for a long time. The attorney cannot find any current areas of conflict between them, though he knows that it is technically possible that some unforeseen conflict could arise in the future. The shared objectives and goals of the group lead the attorney to conclude that no conflicts of interest are present and that it would be counterproductive to try to convince each member of the group to sign an informed consent form acknowledging that conflicts of interest exist and that the attorney may still represent everyone at once. May the attorney trust his professional judgment and proceed without obtaining separate consent forms from each person in the joint venture?

Yes, because the mere possibility of subsequent harm does not itself require disclosure and consent.

An associate in a law firm consulted with a prospective client about providing legal representation. The prospective client wanted to file a lawsuit against a nightclub. A fistfight had erupted at the nightclub between two other patrons, and the potential client had intervened to try to break it up. One of the fighting patrons shoved him out of the way, and he sustained some bruising when he fell. Worse, the nightclub's security guard then arrived andmisinterpreted the situation, and he thought the prospective client had started the fight. The security guard dragged him outside behind the nightclub, where the two had an angry exchange of words. The security guard became enraged and beat the prospective client badly, leaving him with a concussion, black eyes, some missing teeth, and broken ribs. The security guard had been since quit working there and was judgment- proof, but the nightclub had a long prior history of problems with this guard resorting to unnecessary violence against unruly patrons and the club. The associate attorney immediately agreed to represent the prospective client, and only later discovered that the managing partner at his firm owned a 50% share of the same nightclub as a side investment. Even so, the partner gave the associate permission to represent the victim, because he said the bar's liability insurer would cover the claim and settle quickly, and it would generate fees for the firm. Furthermore, the potential client, who ran a real estate business, might hire the firm for other lucrative legal matters. The nightclub's liability insurer, however, refused to settle the matter before the plaintiff had filed a claim in court, and as soon as the associate filed the claim, the insurer's lawyer filed a motion to disqualify the associate's entire firm from the case. Should the court disqualify the firm because one of the partners has invested money in the nightclub, even if that partner is not directly involved in the representation?

Yes, because the partner's conflict of interest would impute to all the other lawyers in the firm, especially if the managing partner has the conflict and associates are handling the representation with his permission.

An attorney is engaged in civil litigation. On his way into the courthouse on the day of jury selection, reporters gather around the attorney hoping for comments. The attorney explains that the (unrelated) criminal trial happening at the courthouse that day is far more important, and he expresses regret that he is not involved in that case at all. He states that he believes the criminal case should result in an acquittal because the police (who are testifying as witnesses in the case) violated the defendant's civil liberties, and because the relevant penal statute itself, which furnished the basis for charges in the case, violates the Bill of Rights. His own civil case, he says, is a brief matter scheduled for a one-day trial, so he hopes to observe the closing arguments tomorrow in the important criminal case in the other courtroom. Are the attorney's statements proper?

Yes, because the rule limiting trial publicity applies only to lawyers who are, or who have been involved in the investigation or litigation of a case.

An attorney was a criminal defense lawyer and she represented a client, who was a defendant in a criminal prosecution. The prosecution called the attorney to the witness stand to authenticate a piece of evidence, which the attorney was willing to do because the authenticity of the evidence was not really in dispute; the attorney planned to use alibi evidence to defeat the charges against the client, which would make this piece of evidence relatively unimportant to the case. May the attorney testify in this manner in a case in which she represents the defendant?

Yes, because the testimony relates to an uncontested issue

An attorney represents Conglomerate Corporation in a lawsuit against the company brought by an individual plaintiff. The lawsuit could bring bad publicity to Conglomerate Corporation and could adversely affect its stock share price. Conglomerate offers to settle the matter quietly, but the plaintiff rejects the settlement offer. The attorney then files a counterclaim against plaintiff, alleging libel and slander of Conglomerate Corporation, vexatious litigation, and tortious interference with contract, for which he demands millions of dollars in damages. The attorney and plaintiff's counsel both know these counterclaims lack any basis in fact, and that they will be costly for plaintiff to defend. The attorney uses the counterclaims as leverage in reopening the settlement negotiations offering to withdraw the counterclaims if plaintiff will accept a new, slightly higher settlement offer. The plaintiff calculates the cost of settlement offer and the expected damages if plaintiff wins at trial, and reluctantly agrees to accept the terms of the offer. Could the attorney be subject to discipline for filing the counterclaim?

Yes, because there is no factual basis for the claims, and the lawyer did not bring them in good faith

Media Company holds the exclusive right to license and distribute certain pay-per-view sporting events, which commercial establishments must license to broadcast at their facilities. It sued a sports bar, for broadcasting one of its major sporting events without a license. The Three Brothers Law Firm were involved before the Media Company filed suit, and Three Brothers Firm had managed to broker a tentative settlement agreement between the parties. Afterward, however, the parties reneged on the agreement and litigation ensued. Three Brothers Firm now represents the defendant sports bar in the matter, and it is counsel of record. Media Company has filed a motion to disqualify Three Brothers from the case, but the attorneys there claim that Media Company was never their client. There was no representation agreement between Media Company and Three Brothers, and Media Company never paid Three Brothers any legal fees. On the other hand, Media Company was otherwise unrepresented during the pre-trial attempt at negotiating a settlement, and its managers asked attorneys from Three Brothers for advice about whether to agree to the settlement instead of going to trial, and initially followed their legal advice on several points. Should the court now disqualify Three Brothers Firm from the case entirely?

Yes, because they obtained confidential information during the negotiations in the same matter, or a matter with significant overlap.

Early in the pre-trial phase of a civil lawsuit involving multiple crossclaims, the court enjoined the parties from transferring any assets out of the jurisdiction. The next day, an attorney heard that his client had transferred millions of dollars to a confidential Swiss bank account. The attorney did not make any affirmative representations to the court about following the court's order. It was clear to the attorney, however, that the court and the opposing party were under the impression that the client was complying with the court's order, and they were relying upon that fact in the ongoing proceedings. The client did not use the attorney's services in any way to make the transfers, and the attorney did not recommend it or know about it until after it occurred. Would it be improper for the attorney to do nothing and say nothing about the matter at this time, to protect the client's confidential information?

Yes, because this is a circumstance where failure to make a disclosure is the equivalent of an affirmative misrepresentation.

A family law attorney represented a client in a divorce proceeding. Early in the representation, before the client's spouse had retained counsel, the attorney advised her client to meet with other lawyers in the area for the sole purpose of creating a conflict of interest, that is, so that the client's spouse would be unable to retain the other lawyers for and scheduled consultations with several other divorce attorneys in a "taint shopping" campaign, but he never intended to retain any of their services. Could the attorney be subject to discipline for instructing the client to do this?

Yes, because this is dishonest, interferes with the administration of justice, and has no purpose other than to interfere with the opposing party's ability to form a client-lawyer relationship.

An attorney has already represented a certain client on several matters. Most recently, the attorney has represented the client in a litigation matter against the city's largest manufacturer. The manufacturer, whom the attorney is suing on behalf of the client, is both the city's largest employer and the largest purchaser of goods and services from small businesses in the area. As the discovery phase winds to a close and the court sets a trial date, the attorney learns that the client misused the attorney's services in the past to perpetrate fraud by having the attorney submit falsified documents to government entities and to insurance companies. The attorney is furious and yells at the client, using profanity. the attorney then petitions the court to let him withdraw from the representation, stating the reasons in general terms that do not betray specific client confidences. The client strongly objects to the attorney withdrawing from the representation, because the trial is only two months away, and all the other litigation firms in the city have conflicts of interest that prevent them from taking a case against the large manufacturer. It is indisputable that the withdrawal is materially prejudicial to the client, who may have to proceed into the trial pro se or must find a new lawyer from out of town. The court is willing to postpone the trial by three weeks to give the client time to find a new lawyer or prepare to represent himself. Is it proper for the attorney to withdraw from representation in this case, if the court has no objection?

Yes, because withdrawal is permissible if the client misused the attorney's services in the past, even if the withdrawal would materially prejudice the client.

An attorney sued Giant Company on behalf of a client in a personal injury matter. During the protracted litigation that ensued, Conglomerate bought Giant Company. The attorney was already representing Conglomerate in a regulatory compliance matter before a federal administrative agency. Assuming this development was unforeseeable at the outset of representing the client against Giant Company, will the attorney have the option to withdraw from one of the representations to avoid the conflict?

Yes, but the attorney must see court approval where necessary and take steps to minimize harm to the clients, and he must continue to protect the confidences of the client from whose representation the lawyer has withdrawn

Three co-owners of a successful startup business hire a certain attorney to help with working out the financial reorganization of their enterprise. The attorney seeks to resolve potentially adverse interests by developing the parties' mutual interests. In assenting to represent all the parties as clients simultaneously, the attorney agrees to adjust the relationship between clients on an amicable and mutually advantageous basis. The clients each provide written consent to the potential conflicts of interest. Is it proper for the attorney to represent three clients with potentially adverse interests in a negotiated transaction?

Yes, common representation is permissible where the clients' interests mostly align, even though there is some difference in interest among them, so the attorney may pursue an agreement on an amicable and mutually advantageous basis.

Two sisters are co-tenants of a house that they inherited from their father. They want to sell the house and hire an attorney to handle the real estate transaction. This attorney explains the potential for conflicts of interest in detail, and each sister readily agrees to provide written informed consent in the form of a waiver of future conflicts of interest. After a prolonged period, they finally find a buyer who is interested in the house, but the buyer wants to impose several onerous conditions on the purchase and engages in unreasonably protracted negotiations over the purchase price. The sisters themselves cannot agree on whether to accept any of the buyer's proposals, further dooming the negotiations. Eventually, one sister becomes frustrated with the attorney over the prolonged, hitherto unsuccessful negotiations, and fires the attorney. The other sister wants the attorney to continue the representation. Regarding the sister who seeks to discharge the attorney, may she do so?

Yes, each client in the common representation has the right to discharge the lawyer as stated in Rules of Professional Conduct and the accompanying Comments.

An attorney represented an indigent client in a civil rights claim matter on a pro bono basis. The client had trouble paying rent or affording basic living expenses. The attorney and the client had recurring disagreements about litigation strategy and tactics, and at one point the client informed the lawyer that he wanted to terminate the representation and find another lawyer to drop the matter. The attorney thought the client was feeling discouraged, so he offered to provide some financial assistant to the client - just enough to cover rent, food, and other basic living expenses until the case concluded. The client agreed to continue with the representation. Was the attorney's conduct improper?

Yes, even under the exception that allows lawyers to provide financial assistance to an indigent client in a pro bono matter, a lawyer may not promise, assure or imply the availability of such gifts prior to retention or as an inducement to continue the client- lawyer relationship after retention.

An attorney works for a firm that handles mortgage lenders in foreclosure actions; she handles foreclosure matters in mediation and at trial. Some close friends of the attorney form a real estate investment company, which buys properties in foreclosure, and sells the properties later at a profit. The friends include the attorney as a passive partner, so she receives a small share of the company's net revenues. The attorney's role in mediation conferences puts her in a position to speed up or slow down foreclosure proceedings, because she negotiates with the defaulting mortgagee for a loan modification. Delayed or failed modifications provide an opportunity for other interested investors, such as her friends' company, to purchase the property at a short sale. The lenders, who are the attorney's clients, often agree to these delays because a short sale may yield a better payoff for the lender than a loan modification. Would it be impermissible for the attorney to drag out the loan modification negotiations with the owner- in-default so that prospective buyers may have the chance to purchase the properties at a short sale?

Yes, even with the consent of the attorney's clients, this is an impermissible dilatory litigation tactic with no substantial purpose other than to delay or prolong the proceeding.

An attorney had a dispute with her nonlawyer siblings about the guardianship of their elderly mother. One of the siblings filed a petition for the appointment as the mother's legal guardian, which the court granted. The attorney then filed an appearance on behalf of her mother in the matter to contest the guardianship. Bitter fighting between the siblings continued, and at one point the attorney filed a motion to withdraw, which the court granted, even though it was not clear that the attorney had ever had a client-lawyer relationship with her mother. As the moths went by, the attorney grew increasingly concerned about how her sibling was treating their elderly mother, and began filing various motions, petitions, and appeals on the mother's behalf seeking judicial relief and the appointment of a different legal guardian. Could the attorney be subject to discipline, given these facts?

Yes, filing petitions and appeals on behalf of someone no longer legally one's client imposes unnecessary delays in court proceedings.

An attorney represents a client in a guardianship proceeding. The client is an adult with Down Syndrome and has an IQ far below average, in the "mental retardation" range of the Diagnostic and Statistical Manual of Mental Disorders (DSM-IV). The client's family is trying to have the client institutionalized involuntarily, and the client is fighting this, wanting instead to live semi-independently in a group home. With the help of a social worker, the client has hired the attorney to defend him against the legal proceedings to have the client institutionalized permanently. Having researched this type of case, the attorney knows that case precedents give the client a small chance of prevailing if the attorney can change the venue to family court or probate court. The attorney has not discussed with the client his decision to seek a change of venue that would be more favorable to the client under that jurisdiction's recent appellate decisions. Switching venue, however, will mean traveling much further (more than an hour) to the proceedings. Is it proper for the attorney to leave the client out of this decision entirely?

Yes, fully informing the client according to the usual ethical standards may be impracticable, because the client suffers from diminished capacity

Attorney Stevenson is a partner in ABC law firm, and Lawyer Best formerly was a partner. A new client has sought to retain Attorney Stevenson to file suit on behalf of the client against Conglomerate Corporation. Before joining the ABC firm, Lawyer Best had represented Conglomerate Corporation at an earlier stage of the current dispute. Lawyer Best has now resigned from the ABC firm, disclosed no confidential information about Conglomerate Corporation relevant to the matter to other lawyers in ABC, left no files at ABC that relate to the proposed suit, and will not share in fees derived by the ABC firm from the representation of the new client. Given that Lawyer Best represented Conglomerate Corporation in the same matter, and then worked for ABC law firm in between (but has recently left the firm), is it proper for Attorney Stevenson to represent the new client in the matter against Conglomerate?

Yes, given Lawyer Best's departure and the fact that nobody else at the firm learned confidential information about Conglomerate Corporation, there is no remaining imputation of Best's conflict of interest.

An elderly retiree was reading the newspaper one morning, and he noticed an advertisement by a local attorney offering to write simple wills for $500. The attorney's name was unfamiliar, but the retiree called the phone number in the ad and asked the attorney to write a simple will for him, and the attorney agreed. Neither party, however, mentioned the advertisement or discussed the attorney's fees. The lawyer drafted the will, met with the client for signing, and then sent a bill for $1500. Under these circumstances, is the client entitled to pay only $500?

Yes, given that the attorney advertised for that amount and the client had seen the ad, the parties have an implicit contract under which the attorney must write the will in exchange for $500.

An attorney worked for a small plaintiffs' firm in Dallas, Texas. The firm undertook the representation of the victim, who suffered severe injuries in a traffic accident with a large truck, allegedly due to the truck driver's negligence. The attorney was not involved in the case at all; another associate at the firm represented the victim in the lawsuit. Big Firm, which has offices in several states, is defending the trucking company in the personal injury lawsuit brought by the victim. The attorney's small firm has a single office and a computer network that allows the five lawyers there to share documents and files from all their cases with each other. Any lawyer in the firm could access all of the other lawyers' documents, which saved time as lawyers could copy and paste from various motions and pleadings that other lawyers had drafted previously on unrelated matters. Every Thursday afternoon, there was a mandatory meeting of the lawyers in the firm, in which they discussed how the pending litigation of each lawyer was proceeding. The lawyers exchanged advice and suggestions for one another's cases. The attorney did not make partner at the small firm, so he left and went to the Kansas satellite office of Big Firm instead. Big Firm assigned the attorney to work on the trucking company case, the same case in which his previous firm represented the opposing party. The attorney had not worked previously on the case and had heard about it only in passing during the weekly litigation meetings at his previous firm, and now remembers almost nothing from the conversations. Should the attorney be subject to disqualification from defending the trucking company?

Yes, if a lawyer has general access to files of all clients of a law firm and regularly participates in discussions of their affairs, it creates an inference that such a lawyer in fact is privy to all information about all the firm's clients, and the burden of proof should rest upon the firm whose disqualification is sought.

A certain attorney was a solo practitioner with many years of experience. For the last few years, the attorney represented a local cupcake shop, jointly owned by Susan and Diane. Susan was in a traffic accident while doing a personal errand, but she was driving the delivery van of the cupcake shop. Susan was co-owner of the shop and was therefore free to use the shop's vehicle for occasional person errands. There is a dispute among the parties involved in the accident about who was at fault. The attorney did not do personal injury litigation, so Susan asked him to refer her to a personal injury lawyer who could represent her at trial. At the same time, Susan insisted that the attorney who handled the business transactional work for the cupcake shop should receive a referral fee, and the attorney is willing to accept joint responsibility for the matter but will not assist in the litigation. The other driver has already filed a claim against Susan and the cupcake shop, and the attorney can see that Susan's interests in the suit are adverse to the cupcake shop's interests. Even though the attorney does not plan to represent the cupcake shop in the lawsuit, the cupcake shop will continue to be the attorney's client for business and transactional matters. Can the attorney make the referral and accept a referral fee, under these circumstances?

Yes, if the attorney obtains the informed consent, confirmed in writing, of both the cupcake shop and Susan as potential co- defendants, and otherwise meets the requirements of Model Rule 1.7(b).

A certain attorney was a solo practitioner with many years of experience. For the last few years, the attorney represented a local cupcake shop, jointly owned by Susan and Diane. Susan was in a traffic accident while doing a personal errand, but she was driving the delivery van of the cupcake shop. Susan was co-owner of the shop and was therefore free to use the shop's vehicle for occasional person errands. There is a dispute among the parties involved in the accident about who was at fault. The attorney did not do personal injury litigation, so Susan asked him to refer her to a personal injury lawyer who could represent her at trial. At the same time, Susan insisted that the attorney who handled the business transactional work for the cupcake shop should receive a referral fee, and the attorney is willing to accept joint responsibility for the matter but will not assist in the litigation. The attorney expects the other driver in the accident to file a claim against Susan, and eventually against the cupcake shop as well, as the owner of the vehicle. In that case, the attorney's duty of loyalty to Susan and the cupcake shop could be in tension, and the attorney could have a material limitation in the representation. Can the attorney make the referral and accept a referral fee, under these circumstances?

Yes, if the attorney obtains the informed consent, confirmed in writing, of both the cupcake shop and Susan as potential co- defendants, and otherwise meets the requirements of Model Rule 1.7(b).

An attorney has a private practice in a large rural township, and she specializes in commercial real estate transactions, such as the sale and lease of farmland, stables, granaries, and mills. As the only lawyer in the township with expertise in this area, she has represented most of the parties who buy and sell commercial real estate there. As a result, most of her clients pose potential conflicts of interest with other current, former, or future clients, so the attorney has a standard "waiver of future conflicts" form that explains conflicts of interest that typically arise in commercial real estate transactions, and she asks every client to sign it at the commencement of representation. The client is a major landholder in the township, who inherited extensive tracts of farmland from his family, who in previous generations were some of the original settlers in the area. Over the years, the client has sold off dozens of small parcels of farmland to neighboring farmers or small businesses such as honey processors, taxidermists, a hardware store, and a veterinarian. The client has also bought properties at times that were adjacent to his existing landholdings. The client has always used other lawyers for these transactions in the past, and in each previous instance, the other party had separate counsel. The client now wants to hire the attorney to sell a parcel to a real estate developer. Buyer (the developer) is also a client of the attorney on unrelated matters, but the Buyer has hired another lawyer to handle this certain matter. The client and Buyer have had a good working relationship in the past and have consummated a few transactions that went smoothly. When the client meets with the attorney to review and sign a retainer for this representation, the attorney includes with the retainer her standard "waiver of future conflicts" form, without additional oral explanation except to mention that she represents Buyer in an unrelated matter. The client reads the form and signs it. As the negotiations for the sale to the developer proceed, a new conflict arises between the client's interests and the unrelated matters for which the attorney has represented the developer, as one will significantly affect the road traffic for the other. This was an unexpected development, though not unusual - such situations were familiar and routine for the attorney and the parties. Is the attorney's standardized "waiver of future conflicts," signed by the client, likely to be effective in this situation?

Yes, if the client agrees to consent to a specific type of conflict with which the client is already familiar, then the consent ordinarily will be effective regarding that type of conflict.

An attorney represented a client in her divorce and custody case. The client's husband had been abusive, so she asked the attorney to obtain a temporary restraining order against her ex-husband. The application for the temporary restraining order is an ex parte proceeding, so opposing counsel is not present. The attorney knows that the ex-husband has not been physically abusive to the client in over two years, and that he has been faithfully attending an anger-management support group during that time that appears to have produced genuine results. At the same time, the client is fearful that the ongoing custody battle will push her ex-husband over the edge, and that the abuse she endured in the past will resume. At the hearing for the temporary restraining order application, does the attorney have an affirmative duty to disclose the length of time since the last abuse occurred and the ex-husband's faithful participation in an anger management program?

Yes, in an ex parte proceeding, a lawyer shall inform the tribunal of all material facts known to the lawyer that will enable the tribunal to make an informed decision, whether or not the facts are adverse.

Two brothers work together in a family landscaping business, and each is a named defendant in a lawsuit over a broken sewage pipe on a client's property where the brothers were digging holes to plant new trees. The two brothers hire their family's attorney to represent them. Though the brothers get along reasonably well, there are several topics they avoid discussing, especially related to family matters and the inheritance, and who is to blame for some lost clients and damaged equipment in the recent past. Then the attorney explains the potential for conflicts of interest in the common representation and asks if they are willing to sign a waiver to the conflicts. One asks the lawyer privately about the issue of confidentiality and privileged information, because it is possible that litigation could emerge within the family later over various issues - the inheritance, control of the business, liability for business losses, and even a marital dispute. Does the common representation have implications for the attorney- client privilege?

Yes, regarding the attorney-client privilege, the prevailing rule is that, as between jointly represented clients, the privilege does not attach, and lawyers should assume that if litigation eventuates between the clients, the privilege will not protect any such communications.

A producer of popular energy drinks and the owner of a popular chain of video-rental kiosks wanted to undertake a joint venture to distribute energy drinks and DVD rentals through the same kiosks. They approached a certain attorney to work out the details of the joint venture and draft the necessary legal documents. The attorney would provide common representation to both as clients in the matter. As part of obtaining informed consent from the clients regarding potential conflicts, the attorney explains that all information would be available to the other client, even information that otherwise would have been confidential information in a normal representation with a single client. Then the attorney explains he will have to withdraw if one client insists that the attorney keep certain information from the other, if the information was relevant and material to the representation. The energy drink maker, however, has a secret formula for the drinks, and the DVD kiosk owner has a trade-secret method of tracking the distribution and stocking of the DVDs in the kiosks minute-by-minute. Neither wanted the other to discover their trade secrets, but the attorney may eventually possess the secrets as part of his document review for the joint venture. Neither client clearly needs to know the trade secrets of the other, however, to proceed with the joint venture. Eventually, the attorney concludes that failure to disclose one client's trade secrets to another client would not adversely affect the representation in this case and agrees to keep that information confidential with the informed consent of both clients. Is the attorney's conduct proper?

Yes, in limited circumstances like this, it would be appropriate for the lawyer to proceed with the representation when the clients have agreed, after receiving adequate disclosures, that the lawyer will keep certain information confidential.

An attorney represented a client in a lawsuit over a traffic accident. The client told the attorney about a certain eyewitness who had been present at the scene and who had said at the time that the client was not at fault. The attorney tracked down this witness, but soon discovered that the eyewitness did not want any involvement in the litigation or trial. The witness was necessary to corroborate the client's version of what happened in the accident, so the attorney offered to pay $500 honorarium in cash if the witness would testify at trial. The eyewitness was out of work needed the cash, so he begrudgingly agreed. Would the Model Rules prohibit the attorney from paying the eyewitness an honorarium to testify at trial?

Yes, it is impermissible for a lawyer to pay an eyewitness to attend and testify at a hearing or trial.

An attorney licensed in Texas represented a group of plaintiffs in a foreign court - a third- world dictatorship with no enforcement of lawyer licensing requirements. The lawsuit claimed that a former United States President was personally responsible for international terrorism, colonial imperialism, climate change, the worldwide malaria epidemic, human trafficking, and narcoterrorism. The local court in the third-world dictatorship found the former President liable on all charges, even though he was not present or aware of the proceedings and awarded damages of ten billion dollars to the local plaintiffs. The attorney then filed an action in the United States jurisdiction where the former U.S. President had a ranch and a personal bank account, seeking to execute on the foreign judgment. The state court immediately dismissed the action with prejudice, and the attorney appealed this decision, still hoping to execute the billion-dollar judgment against the former President. Is the attorney subject to discipline for bringing a frivolous action and appeal?

Yes, it was impermissible for the attorney to bring the action for executing the judgment, and to appeal the dismissal, as there was basis in law or fact for doing so.

Conglomerate Corporation became the subject of an enforcement action by the Department of Labor for violating certain wage- and-hour laws protecting workers' rights. Conglomerate's general counsel interviewed many of the company's employees, in groups of eight or ten at a time, and explained that there was litigation pending with the Department of Labor that could hurt their employer in the long run. General counsel then asked each groups of workers that they decline to discuss the case with anyone, especially lawyers from the government. Did general counsel violate the Model Rules by asking the employees not to talk to the other party?

Yes, it was not reasonable for the general counsel to believe that refraining from giving such information would not impinge on the employees' interests.

Big Firm bills most of its clients on an hourly-billing basis, measured in fifteen-minute increments. Most of the firm's clients are large corporations. Big Firm's associates have burdensome billable hour requirements, so they spend as many hours as possible on every case, working every angle possible, taking an exhaustive approach to research memoranda, depositions of potential witnesses, and daily written updates to the corporate clients about their matters. The managing partners at Big Firm assign a dozen or more associates to every matter, no matter how small, even if that means some associates are merely double-checking or proofreading the work of other associates. The corporate clients and their insurers pay for these services, and whenever the clients prevail in litigation, they seek attorney's fees from the losing party. Could Big Firm (or its managing partners) be subject to discipline for charging unreasonable fees?

Yes, lawyers should not exploit fee arrangements based primarily on hourly charges by using wasteful procedures.

Big Firm raises its hourly billing rate for all clients annually, on the first day of the year, by two percent. The initial engagement documents at the outset of representation explain this practice clearly, but Big Firm does not inform clients in writing each time the annual rate increase occurs. Is it proper for Big Firm to handle its billing and rate increases in this manner?

Yes, periodic, incremental increases in a lawyer's regular hourly billing rates are permissible if a client understands and accepts such practice at the commencement of the client-lawyer relationship, and the periodic increases are reasonable under the circumstances.

The Office of the Attorney General in Texas ordered administrative suspensions of driver's licenses for parents who failed to pay child support, pursuant to state statutes. An attorney worked for the State Office of Administrative Hearings (SOAH), the agency that adjudicated license suspensions like this one. When he decided to leave the SOAH, the attorney surreptitiously copied a database of individuals facing license suspensions and used the names to solicit clients as he started his own firm. The attorney represented clients who wanted to appeal their license suspensions in court, though he did not represent anyone whose case he had personally worked on during his time at the SOAH. Could the attorney be subject to discipline in the cases in which he represents clients appealing their license suspensions?

Yes, the attorney had access to confidential government information from his time working for the state

During a lunchtime recess of a case, the attorney representing the plaintiffs walked with his expert witness to a nearby delicatessen, which full. By coincidence, the server seated the attorney and his expert at the adjoining table to two of the jurors in the case. The attorney recognized the two women from the jury and greeted them, and they chatted for a few minutes about the weather, their favorite sandwiches, and how long the case was taking. They did not discuss the merits of the case itself. Two secretaries from opposing counsel's firm were also eating at the delicatessen and observed this conversation, which they promptly reported to their supervising attorney, who reported it to the judge. The judge ordered a mistrial, dismissed the jurors, and ordered the attorney to reimburse the county for the jurors' fees. Could the attorney also be subject to disciplinary sanctions for running into the two jurors at lunch and making friendly conversation?

Yes, regardless of the attorney's intentions, the conversation violated the prohibition on ex parte communication with jurors.

An attorney was a judge but has left and joined Big Firm. Another lawyer at Big Firm represents the client in a case on the docket at the same court where the attorney worked as a judge. In fact, as a judge, the attorney ruled on some of the pretrial motions in the case, mostly evidentiary motions. The firm has screening measures in place to screen the attorney from any participation in the matter. The attorney will receive no part of the fee from the matter, and timely notice went to the parties about the screening measures in place. The other parties, however, did not provide informed written consent to Big Firm's representation of the client. Is it proper for the other lawyer at Big Firm to continue representing the client in this matter?

Yes, so long as Big Firm also provides timely notice to the appropriate tribunal as well, so that the tribunal may ascertain compliance with screening measures

An attorney is a partner in a law firm, and owns $100,000 worth of stock in Conglomerate Corporation, the named defendant in a new antitrust suit. The attorney's total compensation from the firm is around $15million per year, including bonuses, and his net worth is around $500 million. His home is worth about $7 million and the attorney inherited it, so the property is unencumbered by any mortgage or liens. The attorney works in a specialized area of law at the firm and does not have much interaction with the other lawyers, except at parties and occasional partners' meetings. Another lawyer in the firm seeks to represent the plaintiffs in the antitrust action against Conglomerate Corporation, which is not a client of the firm. Would it be proper for the firm to represent the plaintiffs in litigation against Conglomerate Corporation?

Yes, so long as the attorney is not involved in the representation, there will be no imputation of the attorney's conflict of interest to the other lawyers in the firm, because it is the attorney's personal interest and does present a significant risk of materially limiting the representation of the client by the remaining lawyers in the firm

A certain client hired an attorney to represent him in civil litigation. The client's own testimony at trial would be crucial to the case, and the client was concerned that his embittered former business partner would testify against him as a negative character witness to impeach his credibility. The two had been quite close early in their partnership, but then had a falling out and were no longer on speaking terms. The attorney approached the former partner privately, explained the client's situation, and offered the client's former partner several thousand dollars not to testify or even talk to the opposing counsel in the case. The former partner jumped at the chance to make some easy money by doing nothing, and he accepted the attorney'soffer. Could the attorney be subject to discipline for his actions, as described here?

Yes, the Model Rules do not permit lawyers to offer witnesses money to refrain from testifying or providing information about the matter, with exceptions that do not apply here.

An experienced attorney represented a client in commercial litigation. During a deposition, the client gave answers that the attorney knew to be false, regarding a matter of great relevance to the case. The attorney sat silently and permitted the client to give these answers in the deposition. At the subsequent trial, opposing counsel submitted convincing evidence showing that the client had lied during the deposition. It was evident from the circumstances that the attorney must have known that the client's statements were untruthful at the time. Opposing counsel then filed a grievance against the attorney for allowing the client to give false testimony and failing to rectify it. When the attorney filed a response to the grievance, he explained that alleged ethical violation took place during a deposition, long before the trial, so the duty of candor to the tribunal was inapplicable at that point. Is the attorney subject to discipline?

Yes, the Model Rules require a lawyer to take remedial measures when a client offers false statements even during a deposition.

An attorney represents a small business in a contract dispute with one of its suppliers. The attorney meets eight the employees of his client, in groups of four or five at a time, and explains that there is litigation pending, that Big firm is representing the supplier, and that they should simply decline to discuss the case with anyone, especially lawyers from Big Firm. Was it proper for the attorney to ask the employees not to talk to the other party?

Yes, the Rules of Professional Conduct permit a lawyer to advise employees of a client to refrain from giving information to another party, for the employees may identify their interests with those of the client

Business Manager and Shift Supervisor, who worked at a customer service call-center, became co-defendants in a lawsuit by a disgruntled former employee. The plaintiff claimed to have been the victim of gender discrimination in the form of a hostile work environment, as well as intentional and negligent infliction of emotional distress related to the same factual allegations about her treatment at the workplace. Business Manager hired a certain attorney to represent both himself and the Shift Supervisor, who had been the plaintiff's direct superior. Based on Business Manager's initialinvestigation and review of the personnel files of the plaintiff and the Shift Manager, he believes the allegations are baseless and that the suit will end in a dismissal or summary judgment before trial. Shift Supervisor had a spotless work history, but the plaintiff had numerous interpersonal conflicts with her peers, was frequently late for work or missed work completely, and was the subject of several customer complaints. From his consultations with the defendants, the attorney understood that the complaints targeted the Business Manager and Shift Supervisor equally. Business Manager and Shift Supervisor both gave the attorney written informed consent to the potential conflicts of interest in having the attorney represent both. Business Manager obtained tentative permission to have the business cover the legal fees for the attorney. Near the end of the discovery phase, however, plaintiff produced numerous inappropriate love letters to her from Shift Supervisor, many with explicit sexual overtures, and a few that sounded threatening based on her lack of response to previous letters. In addition, several co-workers of plaintiff gave depositions explaining that they had witnessed Shift Supervisor engaging in inappropriate and unwanted touching of plaintiff on many occasions. Several also testified that Shift Supervisor would often accost her for ten or fifteen minutes outside, before she could reach her workstation, and that this was the cause of her tardiness for work. Business Manager had never heard about any of these problems before. Moreover, during depositions the plaintiff explained that she always had little contact with Business Manager and had no direct complaints about his treatment of her, and she acknowledged that she had never complained to Business Manager about Shift Supervisor's harassment of her. She disclosed that Business Manager was a co-defendant only because her attorney believed it was necessary to name someone from upper management in the lawsuit to trigger the legal protections of Title VII and other antidiscrimination laws. Business Manager then revoked his consent to the conflict of interest, explaining that he wanted separate representation from Shift Supervisor. Trial was due to begin two weeks later. Would it be proper for the attorney to continue representing either Shift Supervisor or Business Manager, but withdraw from representing the other?

Yes, the attorney can potentially continue representing Shift Supervisor but not Business Manager, given the nature of the conflict, the fact that Business Manager revoked consent because of a material change in circumstances, the expectations of Shift Supervisor, and so on.

An attorney agreed to represent a client who wanted to contest the will of her recently deceased aunt. The matter turned out to be much more complicated than the attorney imagined, however, and he already had an overwhelming number of cases for other clients. The attorney received interrogatories from the opposing parties in the matter regarding the will, and he put them off, and then put them off again, as he was busy with other cases. After several months without a response to the interrogatories, the court dismissed the client's case. The client planned to file a malpractice action against the attorney, but the evidence in her case and the relevant law meant she had been unlikely to succeed on her original claim. Could the attorney be subject to discipline for failing to expedite the proceedings, if the court already punished the attorney by dismissing the client's case?

Yes, the attorney did not make reasonable efforts to expedite the litigation consistent with the interests of the client.

A litigation attorney represented Conglomerate Corporation as the defendant in a personal injury lawsuit. Proceedings were underway, and the discovery phase was nearing conclusion. Cross motions for summary judgment were pending. One day, the attorney received a phone call from the judge presiding over the matter, asking to meet the attorney for coffee. When the attorney met with the judge, the judge asked the attorney how much harm it would cause if Conglomerate if the company had to go to trial instead of winning at summary judgment. The attorney explained that the trial would cost his client millions of dollars in expert witness fees, and that settlement seemed impossible at this point, so summary judgment was the only way for his client to avoid a major financial setback that could affect their share price and solvency. Opposing counsel was not present and the two agreed not to mention their meeting to her. The next day, the judge granted summary judgment in favor of Conglomerate Corporation, the attorney's client. Could the attorney be subject to discipline for his conduct in this situation?

Yes, the attorney had an impermissible ex parte communication with the judge presiding over the attorney's case.

An attorney worked for several years for a federal government agency in regulatory enforcement. Big Firm then hired the attorney for a much higher salary, and the attorney accepted the position and left her government position. One of the attorney's first assigned cases at Big Firm was a new action by the client against Conglomerate Corporation. The attorney had worked on an enforcement against Conglomerate Corporation and learned confidential government information about the entity during the litigation. The government agency gives its informed consent, confirming in writing to the representation. Is the attorney nevertheless subject to disqualification in the client's matter against the attorney's former employer?

Yes, the attorney has confidential government information about a person acquired while working for the government agency, and therefore may not represent a private client whose interests are adverse to that person in a matter in which the information could be used to the material disadvantage of that person

An attorney represented a criminal defendant facing extortion charges. The state's star witness against the defendant was a former co-conspirator who had agreed to testify in exchange for a plea deal. To establish the witness' reliability and knowledge of the conspiracy, the prosecution planned to introduce a recorded conversation of an intercepted conference call, from a wiretap, in which the defendant, the witness, and other co-conspirators discussed and planned the conspiracy. The attorney was also part of the recorded conversation, at least at the beginning, though left the call before the later part when the participants agreed to commit their crimes. Even though the attorney was not facing charges as a co-conspirator, his voice would be among others in the recorded conversation when it played at the trial. Given this situation, does the attorney have an ethical duty to have another lawyer represent the defendant at the trial?

Yes, the attorney is in the uncomfortable position of being a potential witness and an advocate in the same trial.

An attorney regularly represented clients in transactional matters. While she was representing a certain client in negotiating and drafting a contract, the client asked the attorney to represent her in a lawsuit as well. The attorney felt nervous because she rarely did litigation work, so she asked the client to sign a waiver of potential malpractice claims that could arise from the litigation work. She orally advised the client to talk to another lawyer about the waiver before signing it, but the client felt that she already had legal representation, as this attorney was handling her transactional matters. The client readily agreed to the waiver. The attorney competently handled the litigation matter, and the case settled before trial with a favorable result for the client. Could the attorney be subject to discipline for obtaining a malpractice waiver from the client?

Yes, the attorney is making an agreement prospectively limiting the lawyer's liability to a client for malpractice, and the client does not have independent representation in making the agreement.

A plaintiff who had prevailed at trial needed representation for the appeal, because the defendant in the case appealed the verdict. Plaintiff's counsel did only trial work, not appellate work, and referred the client to an appellate attorney nearby. The trial lawyer even offered to accompany the plaintiff to the initial consultation with the appellate attorney to help facilitate the transition and to safeguard his client's interests in retaining new counsel.Instead, the plaintiff fired the trial lawyer, terminating the representation, and then went alone to the consultation with the appellate attorney. The appellate attorney asked the plaintiff to sign an agreement waiving potential malpractice claims against the appellate attorney, because the appellate attorney did not want to be responsible for the trial lawyer's mistakes. Theappellate attorney did not inform the plaintiff fully about the risks or downsides of waiving future malpractice claims, nor did he advise the plaintiff of the desirability of seeking the advice of independent legal counsel in connection therewith. Could the attorney be subject to discipline, based on these facts?

Yes, the attorney made an agreement prospectively limiting his liability to a client for malpractice.

Asylum Now is a nonprofit organization that advocates for refugees and immigrants from poor countries. The Board of Directors for Asylum Now wants to bring a test case in federal court to challenge the constitutionality of detaining refugees who enter the country under duress without a visa. Asylum Now has offered to pay an attorney to seek the release of a certain refugee currently in federal detention, and to use this case to challenge current federal laws and regulations that mandate such detentions. The refugee consents to the representation, as well as the payment of legal fees by Asylum Now, and agrees to have his case be the test case that might benefit others. During the representation, the attorney meets several times with the directors of Asylum Now to discuss how to frame their argument in the case in a way that would shape public policy in the right direction. Is it permissible for the attorney to undertake the representation, given this arrangement?

Yes, the attorney may accept payment by Asylum Now and may agree to make contentions that Asylum Now wishes to have tested by the litigation.

During a trial recess, the judge asked the lawyers for both parties to meet with him briefly in chambers. Once there, the judge explained that he planned to retire from the bench soon and was wondering if either of their firms were hiring litigation attorneys, as he might be interested. Could the judge be subject to discipline under the Model Rules of Professional Conduct for making this inquiry?

Yes, under the Model Rules, a lawyer shall not negotiate for employment with any person who is involved as a party or as lawyer for a party in a matter in which the lawyer is participating as a judge personally and in a substantial way.

An attorney worked as in-house counsel for Conglomerate Corporation for eight years, dealing with every aspect of management and corporate affairs, in addition to regulatory compliance. Conglomerate then hired a new CEO, who promptly fired the attorney and replaced him with another lawyer who was a nephew of the CEO. Giant Company has asked the attorney who formerly worked for Conglomerate to represent Giant in an antitrust matter against Conglomerate Corporation. The contemplated lawsuit focuses on specific incidents that took place after the attorney left Conglomerate, but the lawsuit involves sweeping charges of longstanding anti-competitive market strategy and pricing practices by Conglomerate, which would include the time when attorney worked there. Would it be improper for the attorney to represent Giant Company in this matter against Conglomerate?

Yes, the breadth of confidential client information of Conglomerate previously accessible to the attorney during the prior representation, and the breadth of issues open in Giant Company's contemplate lawsuit, creates a substantial risk that the information would materially prejudice Conglomerate in the upcoming litigation.

An attorney was a judge for several years. Near the end of her tenure as a judge, she functioned in the role of the chief administrative judge in that court, assigning cases to the other judges and supervising their work, and had only a limited docket of her own trials. The attorney then left the bench and opened her own law practice. The attorney agrees to represent the client in a matter in the same courthouse where the attorney formerly served as a judge. The attorney even remembers the case, but only the names of the parties and the nature of the action, because she assigned it to the trial judge who currently has the case on his docket, but the attorney had no other involvement in the matter. The client's previous lawyer in the matter was subject to disqualification at the motion of the opposing party due to a conflict of interest. Is it proper for the attorney to represent the client in this matter?

Yes, the fact that a former judge exercised administrative responsibility in a court does not prevent the former judge from acting as a lawyer in a matter where the judge had previously exercised remote or incidental administrative responsibility that did not affect the merits

An attorney represented a man twenty years ago in a divorce with the client's first wife. The client was a well-known local celebrity, a retired professional athlete who became a semi-successful actor and an outspoken advocate of a radical political cause. Recently, former client's third wife approached the same attorney seeking representation in obtaining a divorce from the former client. There are no children from the marriage--their children from previous marriages are now adults--and the distribution of assets will follow the terms of a carefully drafted prenuptial agreement between the former client and his third wife. The former client (that is, the husband) long ago provided written informed consent for future conflicts of interest if the attorney represented another party with adverse interest. The attorney does not believe that any confidential information learned from representing the husband twenty years ago in his first divorce will be relevant to the pending third divorce. On the other hand, there is regular media coverage of the husband's trysts and on-and-off sexual relationships with various actresses and female socialites in the area, and marital infidelity could trigger certain exception clauses in the prenuptial agreement. Would it be proper for the attorney to use the information about husband's recent indiscretions in representing the third wife?

Yes, the fact that a lawyer has once served a client does not preclude the lawyer from using generally known information about that client when later representing another client.

A federal judge hired a clerk for the first two years after the clerk graduated from law school. During his second year as a clerk, he began applying for associate positions at local law firms, to secure a job that would begin immediately after his clerkship ended. A few of the firms to which he applied had pending matters before the same judge, and these were among the firms that interviewed the clerk for an associate attorney position. During the interviewing process, the clerk refrained from mentioning he knew about their pending matters on his judge's docket, though the interviewers always mentioned the fact that their firms regularly appeared before the judge in whose chambers the applicant was then clerking. Each firm that interviewed the clerk received a letter from the judge recommending the applicant to prospective legal employers. The judge did not know where the clerk applied, or which firms were interviewing the clerk; the recommendation letter was a general letter that opened with "To Whom It May Concern." Was it improper for the clerk to apply for positions at firms that have pending matters before the judge for whom she was clerking?

Yes, the fact that the judge did not have notice of where the clerk applied, or which firms were interviewing the clerk.

A client hired an attorney to represent her in business litigation, as the plaintiff, for a set hourly rate for the fees. By agreement, the fees were not due until the conclusion of the matter and the end of the representation. During the pleading phase of the lawsuit, however, the other party unexpectedly impleaded a third party, which made the case far more complicated and time-consuming for the attorney. The attorney explained the problem to the client, and the two agreed to shift to a contingent-fee arrangement. The attorney carefully explained the tradeoffs involved in the different fee arrangements, and offered to continue, on an hourly basis, but both the client and attorney thought that contingent fees were now more appropriate. The attorney fully complied with the written notice requirements of Rule 1.8(a) for changing fees mid-representation. The following day, in another unexpected development, the opposing party offered to settle for a generous sum, more than the parties thought the case was worth, and the client immediately accepted. Must the client now pay the contingent fee to the attorney, even though the client would have paid significantly less under the original hourly fee agreement?

Yes, the fee change was reasonable under these circumstances, and the attorney followed the notice requirements of the Model Rules.

An attorney represented a client who was a stockbroker in a boundary dispute with the client's neighbor. Before the conclusion of the representation, the attorney also made some personal investments using the same client's brokerage services, receiving the same terms, services, and fee waivers that other customers of the brokerage firm received. The attorney did not advise the client to seek the opinion of independent legal counsel for this transaction, and did not obtain signed, written consent from the client about the attorney's role in the transaction. The terms of the brokerage services agreement were in writing, as usual. Based on these facts, were the attorney's actions proper in this transaction?

Yes, this is a standard commercial transaction between the attorney and the client for a service that the client normally would market to others.

A family law attorney represented a client in a child custody dispute. The divorced parents lived in neighboring states, and the dispute involved allegations of child abuse by the client's ex-husband, the opposing party in the case. The case was complex and involved related petitions in two separate courts. The client received an unfavorable preliminary ruling regarding custody in the initial stages of the proceedings. With her client's consent, the attorney then took her zealous advocacy online, using Twitter and other social media platforms to denounce the injustice of the unfavorable preliminary custody ruling, to urge the judges to uphold the law, and to urge readers to write to the judges in the case or advocate for thechildren's safety themselves through Twitter. The attorney also created online petitions on websites like Change.org, with names like "Demand Justice for These Children!" The judges presiding over the various petitions in the case received hundreds of letters, emails, and phone calls in response to the attorney's efforts. Many of these communications by concerned citizens to the judges were hostile and vulgar. Could the attorney be subject to disbarment for such zealous advocacy online?

Yes, the lawyer was attempting to communicate with the judges and potential jurors through public commentary.

Mr. Burns, the chief executive officer of Conglomerate Corporation, now faces criminal charges of discussing prices with the president of a competing firm. If found guilty, both Mr. Burns and Conglomerate Corporation will be subject to civil and criminal penalties under state and federal antitrust laws. An attorney has been representing Conglomerate Corporation. She has conducted a thorough investigation of the matter, and she has personally concluded that no such pricing discussions occurred. Both Conglomerate Corporation and Mr. Burns plan to defend on that ground. Mr. Burns has asked the attorney to represent him, as well as Conglomerate Corporation, in the proceedings. The legal and factual defenses of Conglomerate Corporation and Mr. Burns seem completely consistent at the outset of the matter. Would the attorney need to obtain informed consent to a conflict of interest from both Mr. Burns and a separate corporate officer at Conglomerate Corporation before proceeding with this dual representation?

Yes, the likelihood of conflicting positions in such matters as plea bargaining requires the attorney to obtain the informed consent of both clients before proceeding with the representation.

A family law attorney represented a client in a divorce proceeding. Early in the representation, before the client's spouse had retained counsel, the attorney advised her client to meet with other lawyers in the area for the sole purpose of creating a conflict of interest, that is, so that the client's spouse would be unable to retain the other lawyers for representation in the divorce. The client did so; the client scheduled consultations with several other divorce attorneys in a "taint shopping" campaign, but he never intended to retain any of their services. Could one of the other lawyers permissibly represent the spouse anyway, if they were lucky enough to have evidence to show that the original consultation was merely taint-shopping?

Yes, the person was not genuinely seeking legal representation, so the lawyer would have no duty to protect the confidentiality of the information disclosed and no conflict of interest.

A prosecutor in New York is engaged in plea bargain negotiations with a defendant and defense counsel. The defendant offers to confess to a much more serious crime, committed several years ago in California, if the prosecutor will drop the current charges, which will put the defendant in danger of retaliation from his gang once he is in prison. The prosecutor agrees, and the defendant confesses to a notorious armored car robbery in California ten years earlier that made national news, and for which another wrongfully convicted man was serving his sentence. The defendant describes the crime with enough detail that the prosecutor doubts that he could be fabricating the story. Does the prosecutor have any ethical duties about what to do with this information?

Yes, the prosecutor must promptly disclose that evidence to an appropriate court or authority

An attorney is representing a defendant in a personal injury trial between a celebrity plaintiff and a famous hotel, where the plaintiff claims to have suffered injuries due to unsafe conditions. On his way into the courthouse on the day of jury selection, reporters gather around the attorney hoping for comments. The attorney explains that his client has already made renovations to the hotel to ensure that no accidents happen in the future, even though they do not admit liability in the present case. He also explains that if his client loses, his insurance company will simply pay the damages, and lawsuits like this make everyone's insurance premiums go up. The client had previously given the attorney permission to talk to the media. Opposing counsel is standing nearby waiting for his turn to talk, and he expresses no objection to the first attorney giving interviews like this, or to the lawyer's comments. Are the attorney's statements proper?

Yes, the rules about trial publicity permit lawyers to talk about defenses in the case, and the client's mitigation efforts and public policy concerns over skyrocketing insurance rates could be the defendant's main arguments to the jury

Husband hired a certain attorney to represent him in a divorce; the husband and wife had three adult children. Husband was quite upset when he met with the attorney, because his wife had filed for divorce and he felt deeply betrayed. The couple had a prenuptial agreement that clearly delineated the division of assets in case of divorce, and child custody is not an issue as the children are in their twenties. As part of his routine consultation questions, the attorney asked if there had been any marital infidelity on the part of either the husband or wife. Husband admitted to the attorney that he once had an affair many years ago, that the wife never discovered, and that he wanted to keep secret, if possible. He then speculated that he had no idea if his wife had ever had an affair, then became very emotional as he considered the possibility. Within minutes, he had convinced himself that his wife had been having affairs with other men for years, though he never knew it, and that the three children were unlikely to be his offspring. The attorney had already looked at Husband's photograph of his children, and their resemblance to their father (Husband) was remarkable. the attorney finds repugnant the idea of subjecting the adult children to paternity tests, which would traumatize them unnecessarily, regardless of the result. The attorney also believes that accusing the wife of infidelity would be imprudent, as it will ensure that the family would discover Husband's previous affair, which otherwise might not happen. Without the accusations of infidelity, all the issues of the divorce would come under the prenuptial agreement and not be in dispute. Then the attorney insists on limiting his representation to the divorce and wants to include in the retainer agreement that there will be no accusations of infidelity or paternity testing of the children, unless the other side initiates in this regard. After Husband calms down, he agrees to the attorney's conditions of representation. Is it proper for the attorney to insist on such conditions of representation?

Yes, the terms of the representation agreement may exclude specific means that might otherwise serve used to accomplish the client's objectives, such as actions that the lawyer regards as repugnant or imprudent.

A wealthy heiress hired an attorney to represent her and her family in a complex federal case involving the family business and charges of securities fraud and racketeering. Early in the representation, the attorney and the client start dating and become sexually involved, to the consternation of the rest of the family. The client and her attorney rationalize the relationship by saying that they each have "needs" that their new romantic partner meets, and they have even discussed marriage as an eventuality. Is the attorney subject to discipline for this relationship?

Yes, unless the sexual relationship predates the beginning of legal representation, the lawyer absolutely cannot represent a client with whom he has such a relationship

An attorney represented a wealthy elderly widow whose mental condition was deteriorating. The client was beginning to have traumatic hallucinations and violent outbursts. During one of her lucid moments, she encouraged her attorney to take steps to ensure that her granddaughter, who lived next door and was her primary caregiver, would have legal guardianship over her and would be able to manage her affairs. The granddaughter, who was married and had two children, spoke with the attorney, and urged him to pursue such measures. At the same time, the granddaughter had borrowed a large sum fifteen years earlier to purchase the home adjacent to the widow's, so that she could live close enough to provide daily care for her, and this loan was coming due with a large balloon payment at the end that the granddaughter could not afford. The client would not agree to extend the loan, because she did not expect to live much longer, and no longer understood the problem with the upcoming balloon payment. She also began to miss payments on some of her bills because of her mental confusion. After an episode of violent hallucinations that necessitated the hospitalization and involuntary medication of the client, the attorney decided to proceed, obtained a psychiatric evaluation of the client, and petitioned in court for the granddaughter's appointment as legal guardian of the client. The attorney believed in good faith that such stepswere necessary to protect the client's financial interests and physical well-being. When the client learned that the attorney had filed for a guardianship, she objected to it, and accused the granddaughter of trying to avoid repayment of her loan. Were the attorney's actions proper, given these facts?

Yes, when a lawyer has reason to believe that the client has diminished capacity, is at risk of substantial physical or financial harm unless action is taken, the lawyer may take necessary protective action, including seeking the appointment of a guardian or conservator.

After law school, an attorney worked for the local City Attorney's office in a mid-sized municipality, working mostly on enforcement of anti-pollution and anti-littering ordinances.After five years, the attorney left the position at the municipality and went to work for the federal Environmental Protection Agency (EPA). In some cases, the EPA intervenes in litigation over pollution in which the same municipality is also a party. In that situation, may the EPA ignore the usual screening requirements that would apply to a lawyer moving to a private firm?

Yes, when a lawyer is employed by a city and subsequently is employed by a federal agency, the latter agency does not have to screen the lawyer.

Three years after prosecuting a defendant and obtaining a conviction for murder, another individual comes to the police station and confesses to committing the very murder for which the defendant is already serving time. The defendant always maintained his innocence and the basis of his conviction was an identification (in a lineup) by a single eyewitness. The person now confessing to the crime also fits the description given by the eyewitness and had a plausible motive for committing the murder. Does the prosecutor have a duty report this to the convicted defendant's lawyer?

Yes, when a prosecutor knows of new, credible and material evidence creating a reasonable likelihood that a convicted defendant in his jurisdiction did not commit an offense of which the defendant was convicted, the prosecutor shall promptly disclose that evidence to the defendant unless a court authorizes delay, and undertake further investigation, or make reasonable efforts to cause an investigation, to determine whether the defendant was convicted of an offense that the defendant did not commit.

An insurer retained an attorney to represent it in a matter, and requested a retainer agreement that limited the representation to matters related to the insurance coverage. The insurance was a homeowner's policy for damage to the policyholder's residential real estate, and included a rider for premises liability. The incident that triggered the claim, however, involved the brutal murder of a woman and her two young children across the street from the house in a neighbor's driveway. Due to the limited scope of his representation, however, the attorney ignored the horrific deaths and the fact that the known killer had escaped conviction on a technicality. In a cool and calculated matter, the attorney focused his work exclusively on the property damage from the incident and the premises liability, and obtained a favorable outcome for the insurer. Was it proper for the attorney to limit the scope of his representation in this way?

Yes, when an insurer retains a lawyer to represent an insured, the representation may be limited to matters related to the insurance coverage; a limited representation may be appropriate because the client has limited objectives for the representation

An attorney is representing a defendant in a highly publicized criminal trial. On his way into the courthouse on the day of jury selection, reporters gather around the attorney hoping for comments. The attorney explains that the prosecutor already held a press conference in which she shared that the defendant had refused to take a polygraph test, that DNA tests had confirmed the defendant's guilt, and that the defendant had refused several offers of guilty pleas. To set the record straight before trial, the defense attorney explains that his client had already agreed to take a polygraph test but that none had occurred. He adds that defense experts would testify about problems with the DNA tests, and that the plea offers had all been the same (a life sentence instead of the death penalty) and were unacceptable to the client. Were the defense attorney's statements proper?

Yes, when others have publicly made prejudicial statements, responsive statements may have the salutary effect of lessening any resulting adverse impact on the adjudicative proceeding

A certain attorney represents a client in a transactional matter, a complex business merger. The parties have agreed in advance, by contract, to engage in good-faith negotiations, but that if an agreement does not emerge within six months, either party can abandon the deal and cease negotiations. Three months into the negotiations, the parties are close to a final agreement. The attorney has been conducting the negotiations without the client present, checking in with the client from time to time. One day, the other party presents a detailed proposal that would resolve all remaining issues. This proposal would give each side most of what it wants, but also requires a few concessions from each party. The attorney calls the client immediately and gives a brief overview of the new proposal, hitting most of the highlights and carefully explaining the bottom-line concerning the final buyout price to complete the merger. The client gives the attorney consent to consummate the agreement. Could the attorney be subject to discipline for how he handled the final agreement?

Yes, when there is time to explain a proposal made in a negotiation, the lawyer should review all important provisions with the client before proceeding to an agreement, and the facts suggest that the attorney did not necessarily explain all the concessions that the client would have to make.

Three individuals hire an attorney to represent them as co-defendants in a tort action. At the outset, the attorney tells them that there could be a potential conflict of interest if he represents all three of them, and that they will need to sign informed consent forms, which they do. The three individuals have common goals and interests in the litigation, so they do not hesitate to sign the forms or inquire further about the implications of the potential conflicts. No further discussion occurs about the issue, and the attorney proceeds with the representation. Could the attorney end up having a duty to withdraw from representation later in the litigation, if the clients gave written consent to the shared representation at the outset?

Yes, when undertaking representation of multiple clients in a single matter, the information must include the implications of the common representation, including potential effects on loyalty, confidentiality and the attorney-client privilege and the advantages and risks involved.

A major city has large district attorney's office with many prosecutors. In most cases, several prosecutors share responsibility for parts of a single criminal case, so different prosecutors have responsibility for investigating the matter, presenting the indictment, and trying the case. Inevitably, some less important or immaterial information learned by the prosecutor conducting the investigation, or the grand jury presentation, does not pass along to the other prosecutor in the subsequent proceedings. As a result, the prosecutor handling the trial does not know certain minor details that might be helpful to the defendant's case, and therefore cannot disclose the information to defense counsel. Does it constitute a violation of the Model Rules for cases to pass from prosecutor to prosecutor, with the defendant and the prosecutor being unaware of some minor details (whether inculpatory, exculpatory, or mitigating) lost in the process?

Yes, within the district attorney's office, supervisory lawyers must establish procedures to ensure that each prosecutor involved has the exculpatory evidence to disclose.


Ensembles d'études connexes

ECON 2020-Final Practice Questions

View Set